You are on page 1of 121

15 oct, 2017

10/25/2017 Test Information

Test Information

Test Name DAMS CBT 2017 Total Questions 300

Test Type Examination

Total Marks 1200 Duration 180 minutes

Test Question Language:- ENGLISH

1. Which of the following structures in the scapula is palpable in the infraclavicular fossa?

a. A

b. B

c. C

d. D

Solution. C

Ref: Read the text below

Sol :

The markings are as below:

A-Infraglenoid tubercle

B-Acromian process

C-Coracoid process

D-Inferior angle of scapula

The infraclavicular fossa (deltopectoral triangle)

Its a small depression located inferior to the clavicle

Its located between the clavicular origins of pectoralis major and deltoid, at the junction of the convex medial and concave lateral portions of the
clavicle

The apex of the coracoid process lies approximately 2.5 cm below the clavicle immediately lateral to this fossa, and is covered by the anterior fibres
of deltoid. (Can be palpated in this fossa, the tip of coracoid process to be exact)

LANDMARKS OF SCAPULA

Coracoid process: The apex of the coracoid process lies approximately 2.5 cm below the clavicle.

Acromian process: It lies subcuateously forming the top of shouder. (not in the infraclavicular fossa)

Infraglenoidtuerbcle: It gives origin to long head of triceps.

Inferior angel of scapula: It is found at level of T 7 spine

Answer. c

http://cbt.damsdelhi.com/damscbtadmin/admin/index.php?pageName=test/export&id=1710001
xxxxxxxxxxxxxxxxxxxxxxxxxxxxxxxxxxxxxxxxxxxxxxxxxxxxxxxxxxxxxxxxxxxxxxxxxxxxxxxxxxxxxx 1/121
15 oct, 2017
10/25/2017 Test Information
2. In Pemphigus vulgaris, autoantibodies are formed against which cell adhesion molecule:

a. Selectin

b. Cadherin

c. Integrin

d. IgSF CAM

Solution. (b) Cadherin.

Ref: Read the text below

Sol : in Pemphigus vulgaris (Intraepithelial), autoantibodies are formed against Desmosomes(Cadherin) whereas in Bullous pemphigoid(Subepithelial),
autoantibodies are formed against Hemidesmosomes

Answer. b

3. Development of diaphragm begins at 4 weeks of gestation. Diaphragm has contributions from different structures. Central tendon of diaphragm
develops from?

a. Dorsal mesooesophagus

b. Septum transversum

c. Pleuro peritoneal membrane

d. Cervical myotomes

Solution. (b) Septum transversum

Ref.: Read the text below

Sol :

Development of Diaphragm

- Diaphragm develops from

- Septum transversum : it is a mass of mesodermal tissue arising from coelomic wall at the 4rd week of gestation. It forms the central tendon of
diaphragm.

- Dorsal mesooesophagus forms the median portion of diaphragm & the crurae.

- Cervical myotomes (from C3-C5 somites) of lateral body wall form the peripheral parts of diaphragm.

- Pleuro-peritoneal membranes.

Chromosome 15q plays a major role in diaphragm development

- During the initial period of development, it lies at a higher level (cervical) and then migrates caudally and hence the nerve supply is from C3, C4, C5
(Phrenic nerve).

Answer. b

4. A 29-year-old man presents with duodenal peptic ulcer and complains of cramping epigastric pain. Which of the following structures harbors the cell
bodies of abdominal pain fibers?

a. Lateral horn of the spinal cord

http://cbt.damsdelhi.com/damscbtadmin/admin/index.php?pageName=test/export&id=1710001
xxxxxxxxxxxxxxxxxxxxxxxxxxxxxxxxxxxxxxxxxxxxxxxxxxxxxxxxxxxxxxxxxxxxxxxxxxxxxxxxxxxxxx 2/121
15 oct, 2017
10/25/2017 Test Information
b. Anterior horn of the spinal cord

c. Dorsal root ganglion

d. Sympathetic chain ganglion

Solution. (c)

Dorsal root ganglion

Reference Read the text below

Sol:

Cell bodies of the abdominal pain fibers are located in the dorsal root ganglion. The lateral horn of the spinal cord contains cell bodies of sympathetic
preganglionic nerve fibers; the anterior horn contains cell bodies of general somatic efferent (GSE) fibers.

The sympathetic chain ganglion contains cell bodies of sympathetic postganglionic fibers, which supply blood vessels, sweat glands, and hair
follicles.

The celiac ganglion contains cell bodies of sympathetic postganglionic fibers, which supply the visceral organs such as stomach and intestine

Answer. c

5. The musculophrenic arteries give rise to ?

a. Anterior intercostal arteries for intercostals spaces 79

b. Posterior intercostals arteries for intercostals spaces 311

c. Subcostal artery

d. Inferior phrenic artery

Solution. (a)

Anterior intercostal arteries for intercostals spaces 79

Reference Read the text below

Sol:

The musculophrenic artery gives rise to anterior intercostal arteries for intercostal spaces 79.

Answer. a

6. A 35-year-old woman presents with abdominal tenderness and acute pain. On examination, her physician observes that an abdominal infection has
spread retroperitoneally Which of the following structures is most likely affected?

a. Stomach

b. Transverse colon

c. Jejunum

d. Descending colon

Solution. (d)

Descending colon

Reference Read the text below

Sol:

The descending colon is a retroperitoneal organ. The rest of the organs are surrounded by peritoneum.

Answer. d

7. Which of the following bones articulates with the fibula, tibia, calcaneus, and navicular?

a. Talus

b. Lateral cuneiform

http://cbt.damsdelhi.com/damscbtadmin/admin/index.php?pageName=test/export&id=1710001
xxxxxxxxxxxxxxxxxxxxxxxxxxxxxxxxxxxxxxxxxxxxxxxxxxxxxxxxxxxxxxxxxxxxxxxxxxxxxxxxxxxxxx 3/121
15 oct, 2017
10/25/2017 Test Information

c. Intermediate cuneiform

d. Medial cuneiform

Solution. (a)

Talus

Reference Read the text below

Sol:

The talus articulates with the fibula, calcaneus, navicular, and tibia.

Answer. a

8. A 29-year-old man comes in with a stab wound, cannot raise his arm above horizontal, and exhibits a condition known as winged scapula. Which of the
following structures of the brachial plexus would most likely be damaged?

a. Medial cord

b. Posterior cord

c. Lower trunk

d. Roots

Solution. (d)

Roots

Reference Read the text below

Sol:

Winged scapula is caused by paralysis of the serratus anterior muscle that results from damage to the long thoracic nerve, which arises from the roots of
the brachial plexus (C5-C7).

Answer. d

9. Which type of gland is depicted here :

a. Apocrine glands

b. Merocrine glands

c. Holocrine glands

d. Endocrine glands

Solution. (c)

Holocrine glands

Ref: Read the text below

Sol :

The given slide in the figure is taken from a section of skin, showing sebaceous glands.

In holocrine glands, the secretions are produced in the cytoplasm of the cell and released by the rupture olf the plasma membrane,which destroys
the cell and results in the secretion of the product into the lumen.
http://cbt.damsdelhi.com/damscbtadmin/admin/index.php?pageName=test/export&id=1710001
xxxxxxxxxxxxxxxxxxxxxxxxxxxxxxxxxxxxxxxxxxxxxxxxxxxxxxxxxxxxxxxxxxxxxxxxxxxxxxxxxxxxxx 4/121
15 oct, 2017
10/25/2017 Test Information
Examples; Sebaceous gland (skin), meibomian glands (eyelid).

Answer. c

10. Example of traction epiphysis is :

a. Os trigonum of talus

b. Coracoid process of scapula

c. Head of femur

d. Tibial tuberosity

Solution. (d)

Tibial tuberosity

Ref: Read the text below

Sol :

Coracoid process & Os Trigonum of Talus are eg. of Atavastic epiphysis.

Attached to the tip are the origins of short head of biceps & coracobrachialis.

Attached to the medial border is the insertion of pectoralis minor.

Greater tubercle & tibial tuberosity are examples of traction epiphysis, head of femur is a type of Pressure epiphysis.

Answer. d

11. All of the following statements correctly apply to a bronchopulmonary segment except

a. It is separated from adjacent segments by connective tissue septa.

b. It is the largest subdivision of a lobe.

c. It is not resectable.

d. It is named according to the segmental bronchus supplying it.

Solution. (c)

It is not resectable.

Reference Read the text below

Sol:

A bronchopulmonary segment is a pyramidshaped segment of the lung with its apex facing the lung root and its base at the pleural surface.

It is the largest subdivision of a lobe and is sepa-rated from adjacent segments by connective tissue.

It is surgically resectable

Answer. c

12. Dorsal pancreatic bud does not form one of the following -

a. Head

b. Neck

c. Body

d. Uncinate process

Solution. (d)

Uncinate process

Ref: Read the text below

http://cbt.damsdelhi.com/damscbtadmin/admin/index.php?pageName=test/export&id=1710001
xxxxxxxxxxxxxxxxxxxxxxxxxxxxxxxxxxxxxxxxxxxxxxxxxxxxxxxxxxxxxxxxxxxxxxxxxxxxxxxxxxxxxx 5/121
15 oct, 2017
10/25/2017 Test Information
Sol :

The dorsal bud forms the upper part of head, neck & body of pancreas.

The ventral bud forms the lower part of head & uncinate process.

Answer. d

13. Inferior rectal artery is a branch of -

a. Anterior division of internal iliac

b. Inferior mesentric

c. Internal pudendal

d. Posterior division of internal iliac

Solution. (c)

Internal pudendal

Ref: Read the text below

Sol:

Internal pudendal artery is given in the perineum.

It is a branch of internal pudendal which is a branch of anterior division of internal iliac.

Inferior mesentfric is ventral branch from abdominal aorta at the level of L3.

Answer. c

14. Medial crest of fibula is related to:

a. Posterior tibial artery

b. Anterior tibial artery

c. Popliteal artery

d. Peroneal artery

Solution. (d)

Peroneal artery

Ref: Read the text below

Sol :

Medial crest of fibula is related to peroneal artery, branch of posterior tibial.

Popliteal artery is a continuation of Femoral after it pierces the adductor magnus muscle.

It divides into an Anterior tibial ( Anterior compt ) & Posterior tibial ( Posterior Compt ) of leg.

Answer. d

15. Extra embryonic mesoderm is derived from:

a. Primary yolk sac

b. Secondary yolk sac

c. Epiblast

d. Hypoblast

Solution. (a)

Primary yolk sac > d. Hypoblast > c. Epiblast

http://cbt.damsdelhi.com/damscbtadmin/admin/index.php?pageName=test/export&id=1710001
xxxxxxxxxxxxxxxxxxxxxxxxxxxxxxxxxxxxxxxxxxxxxxxxxxxxxxxxxxxxxxxxxxxxxxxxxxxxxxxxxxxxxx 6/121
15 oct, 2017
10/25/2017 Test Information
Ref: Read the text below

Sol :

The outer layer of cells from the primary umbilical vesicle (yolk sac) forms a layer of loosely arranged connective tissue, the extraembryonic
mesoderm.

The origin of the extraembryonic mesoderm is by no means clear: it may arise from several sources, including the caudal region of the epiblast, the
parietal hypoblast: trophoblast or a new germinal population which is yet to be established (Grays Anatomy ed 41).

Answer. a

16. Which of the following is least useful in bupivacaine induced cardiac toxicity

a. 20 % lipid emulsion

b. Atropine

c. Epineprine

d. Amiodarone

Solution. (d) Amiodarone


Ref: Read the text below
Sol:
Bupivacine forms irreversible complexes with receptors of heart.
Treatment of CVS toxicity is 20% intralipid emulsion, adrenaline,atropine. lignocaine and amiodarone should not be used routinely.
CNS toxicity is manifested by circumoral numbness, paresthesia of tongue, lightheadedness, diziness,muscle twitching,tremors,convulsions.
Tt is small dose of propofol/thiopentone,secure airway,BZD,anticonvulsants.

Answer. d

17. Which of following is not used topically

a. Coccaine

b. Benzocaine

c. Chlorprocaine

d. Tetracaine

Solution. (c) Chlorprocaine


Ref: Read the text below
Sol:
Topically used LA are coccaine,benzocaine,tetracaine,lignocaine,dibucaine,prlocaine
Chlorprocaine is c/I in spinal anaesthesia as it causes neurotoxicity.

Answer. c

18. Induction agent of choice in patient with known history of malignant hyperthermia is

a. Propofol

b. Etomidate

c. Thiopentone

d. Ketamine

Solution. (a) Propofol


Ref: Read the text below
Sol:
Choice of anaesthesia is TIVA which is combination of propofol and remifentanil. Inhalational agents should be and scoline should not be used.
Most initial symptoms of MH is masseter muscle spasm.

Answer. a

19. Afferent nerves most sensitive to L.A blockade are

http://cbt.damsdelhi.com/damscbtadmin/admin/index.php?pageName=test/export&id=1710001
xxxxxxxxxxxxxxxxxxxxxxxxxxxxxxxxxxxxxxxxxxxxxxxxxxxxxxxxxxxxxxxxxxxxxxxxxxxxxxxxxxxxxx 7/121
15 oct, 2017
10/25/2017 Test Information

a. A alpha

b. A beta

c. A delta

d. C fibres

Solution. (c) A delta


Ref: Read the text below
Sol:
A alpha fibres mediate motor, a beta afferent from skin and joints, A gamma are efferent to muscle spindles , A delta are afferent sensory nerves mediate
cold temp,touch,pain.
B fibres are preganglionic autonomic fibres and perform various autonomic functions
C fibres are 2 types
Postganglionic autonomic fibres and perform various autonomic functions, dorsal root fibres mediate warm temp,pain, touch.
LA sensitivity for A fibres are A gamma>A delta>A beta>A alpha
B>C>A

Answer. c

20. Baroreflex mechanism is altered by

a. Propofol

b. Ketamine

c. Thiopentone

d. Midazolam

Solution. (a) Propofol


Ref: Read the text below
Sol:
Propofol blunts carotid body receptor response so compensatory tachycardia may not occur.it may cause more fall in blood pressure.
Propofol is antiemetic, anticonvulsant,antipruritic, antioxident.

Answer. a

21. Not correct about desflurane is

a. It is fluorinated methyl ethyl ether

b. Blood gas solubility is less than nitrous oxide

c. MAC is lower than isoflurane

d. When used in high concentration causes hypertension and tachycardia

Solution. (c) MAC is lower than isoflurane


Ref: Read the text below

Sol:
Desflurane is most pungent smelling inhational agent.
Its B/G coff is .42 which is lowest among fluronated inhational agent.
It forms CO with dry CO2 absorbent,minimally metabolised, maximum green house effects, MAC is 6.6 in 100% O2, and2.3 in 66% nitrous oxide.
Sudden increase causes sympathetic stimulation .

Answer. c

22. Shortest acting local anaesthetic for subarachnoid injection is

a. Lignocaine

b. Procaine

c. Mepivacaine

d. Bupivacaine

Solution. (b) Procaine

http://cbt.damsdelhi.com/damscbtadmin/admin/index.php?pageName=test/export&id=1710001
xxxxxxxxxxxxxxxxxxxxxxxxxxxxxxxxxxxxxxxxxxxxxxxxxxxxxxxxxxxxxxxxxxxxxxxxxxxxxxxxxxxxxx 8/121
15 oct, 2017
10/25/2017 Test Information
Ref: Read the text below
Sol:
Procaine is LA of choice of in pts with history of MH, lignocaine and mepivacaine are intermeidiate acting while bupivacaine is long acting.

Answer. b

23. Choose most appropriate statement regarding Post dural puncture headache

a. Headache is more in supine position than sitting

b. Headache is usually temporal> frontal

c. Diplopia and tinitus may occur

d. Bed rest and analgesics is treatment of choice

Solution. (c) Diplopia and tinitus may occur


Ref: Read the text below
Sol:
PDPH occurs due to leak of CSF from dural puncture site.
The leak starts 24-48 hrs after puncture and lasts for 7 days usually.
Low pressure headache.
Headache increases in sitting and standing position and decreases in lying down position.it is usually occipital >frontal. neurological symptoms may
occur,it can be prevented by using pencil tip needles, higher gauge needles, correct dehydration.
Most definitive tt is epidural blood patch.

Answer. c

24. An infant on non human milk , NPO period is

a. 2 hrs

b. 4 hrs

c. 6 hrs

d. 8 hrs

Solution. (c) 6 hrs


Ref: Read the text below
Sol:
NPO period is in breast feeding infant 4hrs , infant on non human milk and formula feed is 6 hrs.
In adults NPO is 6 hrs for solids and 4 hrs clear liquids.

Answer. c

25. In pts with advanced airway breath rate during CPR is

a. 6-8/min

b. 8-10/min

c. 10-12/min

d. 12-14/min

Solution. (b) 8-10/min


Ref: Read the text below
Sol:
CPR rate is 100/min, compression to ventilation ratio is adults 30:2 in adults, in children and infants 30:2 single rescuer, 15:2 double rescuer, drug of
choice is adrenaline 1:10000, 1 mg iv, hyperventilation should be avoided rate is 8-10/min if pt is intubated.

Answer. b

26. True regarding cytoplasmic m RNA?

a. It is chiefly translated from nuclear DNA

b. The sugar is deoxyribose

http://cbt.damsdelhi.com/damscbtadmin/admin/index.php?pageName=test/export&id=1710001
xxxxxxxxxxxxxxxxxxxxxxxxxxxxxxxxxxxxxxxxxxxxxxxxxxxxxxxxxxxxxxxxxxxxxxxxxxxxxxxxxxxxxx 9/121
15 oct, 2017
10/25/2017 Test Information
c. Thymine is present in place of uracil

d. Its MW is greater than heterogenous RNA

Solution. (a) It is chiefly translated from nuclear DNA


Ref: Read the text below
Sol:
mRNA is chiefly transcribed from nuclear DNA
MW of cytoplasmic mRNA is lesser than the heterogenous RNA .
mRNA contain only uracil not the thymine .
Sugar is ribose moiety, not the deoxyribose moiety .

Answer. a

27. All of the following types of porphyrias are autosomal dominant except -

a. Acute intermittent porphyria

b. Porphyria cutanea tarda

c. CEP (Congenital erythropoetic porphyria)

d. Hereditary coproporphyria

Solution. (c) CEP (Congenital erythropoetic porphyria)


Ref: Read the text below
Sol:
CEP is inherited in Autosomal recessive pattern.
All other inherited porphyrias are transmitted in autosomal dominant pattern

Answer. c

28. The first step in livers metabolism of fructose is ?

a. Isomerization of glucose

b. Phosphorylation to fructose 1, 6 bisphosphate by ATP

c. Phosphorylation to fructose 6 phosphate by ATP

d. Phosphorylation to fructose 1 phosphate by ATP

Solution. (d) Phosphorylation to fructose 1 phosphate by ATP


Ref: Read the text below
Sol:
- This reaction is catalysed by Fructokinase.
- Deficiency of fructokinase leads to Essential Fructosuria.

Answer. d

29. Cabbage odour urine is found in ?

a. Leucine catabolism defect

b. Valine catabolism defect

c. Tyrosinemia type 1

d. None

Solution. (c) Tyrosinemia type 1


Ref: Read the text below
Sol:

- Tyrosinemia type 1 is due to defect of hydrolase enzyme in this disorder there occurs cabbage odour urine.

Answer. c

30. The predominant isoenzyme of LDH in the cardiac muscle is?

http://cbt.damsdelhi.com/damscbtadmin/admin/index.php?pageName=test/export&id=1710001
xxxxxxxxxxxxxxxxxxxxxxxxxxxxxxxxxxxxxxxxxxxxxxxxxxxxxxxxxxxxxxxxxxxxxxxxxxxxxxxxxxxxxx 10/121
15 oct, 2017
10/25/2017 Test Information
a. LDH 1

b. LDH 2

c. LDH 3

d. LDH 5

Solution. (a) LDH 1


Ref: Read the text below
Sol:
- In normal plasma LDH-2 is more in concentration than LDH 1.
 -In myocardial infarction level of LDH 1 increases and this leads to altered ratio of LDH.
- It means LDH 1 > LDH 2.
 -This altered ratio of the LDH is known as flipped pattern.

Answer. a

31. Aminoacyl t- RNA is required for all except ?

a. Hydroxyproline

b. Methionine

c. Cystine

d. Lysine

Solution. (a) Hydroxyproline


Ref: Read the text below
Sol:
- Hydroxyproline is a derived amino acid and there is no t-RNA available to transport
derived amino acids: hydroxyproline and hydroxylysine are two examples of derived
amino acid for which there is no t-RNA available.

Answer. a

32. Modifications to the nucleotides of the pre-tRNAs, pre-rRNAs and pre-mRNAs occur

a. Postprandially

b. Postmitotically

c. Pretranscriptionally

d. Postranscriptionally

Solution. (d) Postranscriptionally


Ref: Read the text below
Sol:
As the modification is taking place after the transcription is complete of the above RNAs, it is a posttranscriptional modification.
Site of posttranscriptional modification of various RNA

pre-tRNAs: cytosol
pre-rRNAs: nucleolus
pre-mRNAs: nucleus

Answer. d

33. Ubiquitin breaks down of proteins in

a. Lysosomes

b. Proteasomes

c. Macrophages

d. Emdoplasmic reticulum

Solution. (b) Proteasomes


Ref: Read the text below
Sol:
Ubiquitin breaks down certain misfolded proteins, proteins having short half life by
http://cbt.damsdelhi.com/damscbtadmin/admin/index.php?pageName=test/export&id=1710001
xxxxxxxxxxxxxxxxxxxxxxxxxxxxxxxxxxxxxxxxxxxxxxxxxxxxxxxxxxxxxxxxxxxxxxxxxxxxxxxxxxxxxx 11/121
15 oct, 2017
10/25/2017 Test Information
ubiquinating them.
The organelle is Proteasome which is multisubunit barrel shaped organelle.

Answer. b

34. Reading of codon on mRNA is

a. 5-3 direction

b. 3-5 direction

c. Direction nonspecific

d. None

Solution. (a) 5-3 direction


Ref: Read the text below
Sol:
Reading of codon on mRNA occurs in 5-3 direction
Protein is synthesized in N terminal to C terminal direction.

Answer. a

35. The first reaction involved in degradation of most of the amino acid involves the participation of

a. NAD+

b. Thiamine pyrophosphate(TPP)

c. Pyridoxal phosphate (PLP)

d. FAD+

Solution. (c) Pyridoxal phosphate (PLP)


Ref: Read the text below
Sol:
The first reaction involved in degradation of most of the amino acid involves the
participation of
Transaminase enzyme. Cofactor for transaminase enzyme is PLP.

Answer. c

36. Select the statement which is not correct

a. The side chain of cysteine and methionine absorb light at 280 nm

b. Glycine is often present at a region where a polypeptide is forming a sharp bend reversing the direction of a polypeptide

c. The C,N,H and O atoms of a peptide bond are coplanar

d. A linear pentapeptide contain four peptide bond

Solution. (a) The side chain of cysteine and methionine absorb light at 280 nm
Ref: Read the text below
Sol:
Amino acids which absorb uv light at 280nm are:
Tryptophan
Tyrosine
phenylalanine

Answer. a

http://cbt.damsdelhi.com/damscbtadmin/admin/index.php?pageName=test/export&id=1710001
xxxxxxxxxxxxxxxxxxxxxxxxxxxxxxxxxxxxxxxxxxxxxxxxxxxxxxxxxxxxxxxxxxxxxxxxxxxxxxxxxxxxxx 12/121
15 oct, 2017
10/25/2017 Test Information
37. Which of the following statement is not correct?

a. Metabolic disorder are only infrequently associated with defects in catabolism of purines

b. Immune dysfunction are associated both with defective adenosine deaminase and defective purine nucleoside phosphorylase

c. Xanthine lithiasis can be due to severe defect in xanthine oxidase

d. Hyperuricemia can result from conditions such as cancer characterized by enhanced tissue turnover

Solution. (a) Metabolic disorder are only infrequently associated with defects in catabolism
of purines
Ref: Read the text below
Sol:
Metabolic disorder are only infrequently associated with defects in catabolism of
purines
There are number of metabolic disorders which are associated with defect in purine catabolism. To name they are as follows:
Immune dysfunction are associated both with defective adenosine deaminase and defective purine nucleoside phosphorylase

Answer. a

38. The initiator tRNA is placed within the active 80S complex at which of the three canonical ribosomal sites dueing protein synthesis

a. E site

b. I site

c. P site

d. A site

Solution. (c) P site


Ref: Read the text below
Sol:
The initiator tRNA is placed at P site within the active 80S complex during protein
synthesis.

Answer. c

39. Select the most appropriate statement about isoenzyme

a. Different km value

b. Act on different substrate

c. Consist of multimeric complex

d. Same electrophoretic mobilityl

Solution. (a) Different km value


Ref: Read the text below
Sol:
Isoenzyme have different physical characteristics.
Following are the physical characteristic which differ in isoenzymes
a. Km Value
b. Structure
c. Tissue distribution
d. Electrophoretic mobility

http://cbt.damsdelhi.com/damscbtadmin/admin/index.php?pageName=test/export&id=1710001
xxxxxxxxxxxxxxxxxxxxxxxxxxxxxxxxxxxxxxxxxxxxxxxxxxxxxxxxxxxxxxxxxxxxxxxxxxxxxxxxxxxxxx 13/121
15 oct, 2017
10/25/2017 Test Information
e. Immunological characteristics
f. Substrate affinity

Answer. a

40. The enzyme avidin inactivates.. in egg

a. Pyriodoxine

b. Riboflavin

c. Biotin

d. Thiamine

Solution. (c) Biotin


Ref: Read the text below
Sol:
Avidin and biotin they tend to bind to make an irreversible complex which impedes the absorption of biotin.
Avidin is heat labile.

Answer. c

41. A child is diagnosed having psoriasis associated with streptococcal sore throat. The lesions are likely to be:

a. Violaceous with white criss-crossing lines on surface

b. Small red papules with mild scaling distributed over trunk

c. Annular plaques with central crusting

d. Photo distributed plaques with pigmented borders and carpet tack scales

Solution. (b) Small red papules with mild scaling widely distributed over trunk.

Ref:Bolognia Dermatology, 3rd edition; p 135-139.

Sol :

Psoriasis has a complex etiopathogenesis.

Both genetic as well as external factors are at play. In a genetically predisposed individual, external triggering factors may precipitate psoriasis.

Amongst the genetic factors HLA-Cw6 has strongest association. Amongst the external precipitating factors drugs, infections and trauma are
important. The classical drugs known to precipitate psoriasis include lithium, -blockers, antimalarials and salicylates.

Psoriasis is classically well defined erythematous plaques with silvery white scales.

Guttate psoriasis is strongly associated with streptococcal sore throat in children. Guttate psoriasis is characterized by small red papules widely
scattered over trunk with little scaling.

Violaceous plaques with criss-crossing white lines on surface (Wickhams striae) are seen in lichen planus.

Centrally crusted annular lesions may be seen in leishmaniasis. Carpet tack scales are seen in discoid lupus erythematosus.

Answer. b

42. Which test is based on the principle of type 4 delayed type hypersensitivity:

a. Patch test

b. Prick test

c. Radioallergen sorbent test

d. IgE level estimation

Solution. (a) Patch test

Ref:Bolognia Dermatology, 3rd edition; p 236.

Sol :

http://cbt.damsdelhi.com/damscbtadmin/admin/index.php?pageName=test/export&id=1710001
xxxxxxxxxxxxxxxxxxxxxxxxxxxxxxxxxxxxxxxxxxxxxxxxxxxxxxxxxxxxxxxxxxxxxxxxxxxxxxxxxxxxxx 14/121
15 oct, 2017
10/25/2017 Test Information
Allergic contact dermatitis is confirmed with patch test.

Patch test is aimed for identification of allergens by applying them on skin under occlusion for 48 hours to elicit delayed type hypersensitivity.

IgE and radioallergen sorbent test as well as prick test are used for acute type I hypersensitivity, whereas allergic contact dermatitis is a type IV
hypersensitivity.

Answer. a

43. The obstruction in miliaria is due to

a. Propionibacterium

b. Staphylococcus aureus

c. Staphylococcus epidermidis

d. Streptococcus viridans

Solution. (c) Staphylococcus epidermidis

Ref:Andrews Diseases of The Skin,11th edition; p 19-20.

Sol :

Miliaria, the retention of sweat is a result of occlusion of eccrine sweat ducts. Staphylococcus epidermidis, which produces an extracellular
polysaccharide substance, induces miliaria. This polysaccharide substance may obstruct the delivery of sweat to the skin surface produces an eruption that
is common in hot, humid climates, such as in the tropics and during the hot summer months in temperate climates.

Depending on the level of the injury to the sweat gland or duct, several different forms are recognized- Miliaria crystalline, miliaria rubra (prickly
heat) and miliaria profunda

Answer. c

44. Which of these biologic is anti CD20 antibody:

a. Secukinumab

b. Ustekinumab

c. Etanercept

d. Rituximab

Solution. (d) Rituximab


Ref: Bolognia Dermatology, 3rd edition; p 152, 53.
Sol :
- Rituximab is anti-CD20 Ab.
- Thus its target is B cells and hence it is useful where immunoglobulins are involved.
- Psoriasis is cell mediated mainly by TH17 and Th1 cells. Etanercept, infliximab,
adalimumab are classic TNF alpha inhibitors used in psoriasis.
- Ustekinumab and secukinumab work on IL23 and IL17 respectively and hence used in psoriasis.
- Rituximab in dermatology is used for immuno bullous diseases like pemphigus.

Answer. d

45. A patient presents with asymptomatic, gradually expanding, infiltrated plaque with central scarring as shown in image. Likely diagnosis is:

a. Lupus vulgaris

http://cbt.damsdelhi.com/damscbtadmin/admin/index.php?pageName=test/export&id=1710001
xxxxxxxxxxxxxxxxxxxxxxxxxxxxxxxxxxxxxxxxxxxxxxxxxxxxxxxxxxxxxxxxxxxxxxxxxxxxxxxxxxxxxx 15/121
15 oct, 2017
10/25/2017 Test Information
b. Tinea corporis

c. Discoid lupus erythematosus

d. Fixed drug eruption

Solution. (a) Lupus vulgaris

Ref:Read the text below

Sol :

Lupus vulgaris is likely diagnosis.

Tinea has no scarring and is itchy.

DLE has carpet tack scales, follicular plugging and pigmented borders.

Fixed drug eruption is pigmented and may have blisters in center.

Answer. a

46. Erythema nodosum is type of

a. Panniculitis

b. Mastocytosis

c. Dermatitis

d. Vasculitis

Solution. (a) Panniculitis

Ref:Andrews Diseases of The Skin,11th edition; p 143.

Sol :

Eythema nodosum is a classic panniculitis that is inflammation of subcutaneous fat.

It is septal panniculitis by histology and may be associated with drugs like OCP, sulfonamides etc., pregnancy, TB, streptococcal infections, other
infections including histoplasmosis, coccidiodocomycosis, sarcoidosis, Behcets syndrome etc.

Answer. a

47. Target lesions are seen in

a. Erythema annulare centrifugum

b. Erythema marginatum

c. Erythema toxicum neonatorum

d. Erythema multiforme

Solution. (d) Erythema multiforme

Ref:Bolognia Dermatology, 3rd edition; p 307- 323.

Sol :

Erythema annulare centrifugumis characterized by annular or polycyclic lesions that grow slowly (23 mm/day). Characteristically, there is a
trailing scale at the inner border of the annular erythema

Erythema marginatum is seen in rheumatic fever as asymptomatic reticular erythema.

Erythema toxicum neonatorum is physiological in day 2-3 old child. Characteristically, the broad erythematous flare is much more prominent than
the small follicular papule or pustule it surrounds and heals on its own by day 10.

Erythema multiformeis associated with HSV infections most commonly. However, other infections like mycoplasma may also be associated. The
lesions are known as target lesions or iris lesions or bulls eye lesions. These are made of concentric rings of varying colors from purplish in center, whitish
in middle and red in outer ring.

Answer. d

http://cbt.damsdelhi.com/damscbtadmin/admin/index.php?pageName=test/export&id=1710001
xxxxxxxxxxxxxxxxxxxxxxxxxxxxxxxxxxxxxxxxxxxxxxxxxxxxxxxxxxxxxxxxxxxxxxxxxxxxxxxxxxxxxx 16/121
15 oct, 2017
10/25/2017 Test Information
48. Which is correctly matched:

a. Nikolsky sign: Bullous pemphigoid

b. Auspitz sign: Lupus vulgaris

c. Apple jelly nodules: Psoriasis

d. Carpet tack sign: Discoid Lupus Erythematosus

Solution. (d) Carpet tack sign: Discoid Lupus Erythematosus.

Ref:Read the text below

Sol :

Nikolsky sign is negative in bullous pemphigoid and positive in pemphigus. Auspitz sign is seen in psoriasis while apple jelly nodules are seen on
diascopy in lupus vulgaris. Dimpling sign is typical of dermatofibroma.

A list of cutaneous signs is given below:

a. Diascopy (VITROPRESSION) consists of pressing a transparent slide over a skin lesion. Examiner will find this of special value to distinguish
erythema or purpura. It is useful to detect the yellow-brown appearance of papules in sarcoidosis, tuberculosis(Apple Jelly Nodules) and other granuloma.

b. Darier's sign is positive when a brown macular or papular lesion of urticaria pigmentosa (cutaneous mastocytosis)becomes palpable wheal after being
rubbed with the blunt end of an instrument.

c. Grattage test & Auspitzs sign is positive when slight scratching or curetting of a scaly lesion reveals initially fine candle wax scales followed by red
Berkleys membrane; which then gets removed to reveal punctate bleeding points(Auspitz's sign) within the lesion which suggests of psoriasis.

d. Nikolsky's sign is positive when a new blister is generated with ease by applying shearing force to skin or epidermis is dislodged ( pemphigus, SSSS,
TEN)

e. Asboe Hansen (Bulla spread) sign: Spread of bulla to normal skin by vertical pressure over in Pemphigus (~ to Nikolskys)

f. Carpet Tack Sign: or carpet en tack sign: removal of adherent scales in DLE reveals downward projection of scales which are follicular plugs.

g. Buttonholing sign: Neurofibroma (with central vertical pressure the lesion disappears under the skin)

h. Dimpling sign: dermatofibroma and dermatofibrosarcoma protuberans.

i. Koebners phenomenon: Spread of lesions of same morphology (isomorphic phenomenon) at the site of trauma. Characteristically seen in Lichen
planus, psoriasis, vitiligo and by auto-inoculation in verruca, molluscum contagiosum
etc..

Answer. d

49. An elderly patient has developed a slowly enlarging lesion as shown in the image. Likely diagnosis is:

a. Squamous cell carcinoma

b. Melanoma

c. Cutaneous T cell lymphoma

d. Basal cell carcinoma

Solution. (d) Basal cell carcinoma.

Ref:Read the text below

Sol :

Chronic ulcer with rolled borders is typical of basal cell carcinoma.

Pigmentation is common in people with pigmented skin like Indians. Then it is called pigmented BCC.

http://cbt.damsdelhi.com/damscbtadmin/admin/index.php?pageName=test/export&id=1710001
xxxxxxxxxxxxxxxxxxxxxxxxxxxxxxxxxxxxxxxxxxxxxxxxxxxxxxxxxxxxxxxxxxxxxxxxxxxxxxxxxxxxxx 17/121
15 oct, 2017
10/25/2017 Test Information
Answer. d

50. The pigmentation in half and half nail is mainly due to

a. Vasodilation

b. Melanin deposition

c. Albumin deposition

d. Porphyrin deposition

Solution. (b)Melanin deposition.

Ref:Read the text below

Sol :

Important nail changes

1) HALF AND HALF NAILS: Proximal portion is white and distal portion is brown with a sharp demarcating line. The pigmentation is a result of
melanin deposition (and capillary proliferation). Seen in renal diseases with azotemia.

2) TERRYS NAILS: Distal 1-2 mm of nail is pink and the rest is white. Seen in cirrhosis, CHF, Diabetes.

3)MEES LINES: Single or multiple white transverse bands on nail seen in chronic arsenic poisoning,septicemia, chemotherapy, renal failure.

4)MUEHRECKES NAILS:Narrow white bands occurring in pairs seen in chronichypo albuminemia.

5) Longitudinal white lines with V-shaped nicks at free margin of nails: Dariers disease.

Answer. b

51. Effective method for treating Recurrent juvenile nasopharyngeal angiofibroma -

a. Transnasal endoscopic

b. Transpalatal

c. Lateral rhinotomy

d. Transmaxillary

Solution. (a) Transnasal endoscopic


Ref: Read the text below
Sol:
Recurrent JNAs are residual disease resulting from incomplete removal of the primary tumor.
Transnasal endoscopic sinus surgery is an effective method for treating recurrent JNA.

Answer. a

52. Most severe complication of maxillary sinus lavage and insufflation-

a. Facial nerve injury

b. Air embolism

c. Epistaxis

d. Coagulopathy

Solution. (b) Air embolism


Ref: Read the text below
Sol:
Antral lavage is a surgical procedure in which a cannula is inserted into the opening of the maxillary sinus via the inferior meatus to allow irrigation and
drainage of the sinus.
Venous air embolism has been known to be a major hazard during maxillary sinus lavage and insufflation

Answer. b

53. All of the following are causes of Rhinolalia Clauses except-

http://cbt.damsdelhi.com/damscbtadmin/admin/index.php?pageName=test/export&id=1710001
xxxxxxxxxxxxxxxxxxxxxxxxxxxxxxxxxxxxxxxxxxxxxxxxxxxxxxxxxxxxxxxxxxxxxxxxxxxxxxxxxxxxxx 18/121
15 oct, 2017
10/25/2017 Test Information

a. Nasopharyngeal carcinoma

b. Hypertrophied adenoids

c. Allergic Rhinitis

d. Velopharyngeal insufficiency

Solution. (d) Velopharyngeal insufficiency


Ref: Read the text below
Sol:

Answer. d

54. The best tool for hearing screening for NICU or high risk babies

a. OAE

b. AABR

c. ECOG

d. Impedance Audiometry

Solution. (b) AABR


Ref: Read the text below
Sol:

http://cbt.damsdelhi.com/damscbtadmin/admin/index.php?pageName=test/export&id=1710001
xxxxxxxxxxxxxxxxxxxxxxxxxxxxxxxxxxxxxxxxxxxxxxxxxxxxxxxxxxxxxxxxxxxxxxxxxxxxxxxxxxxxxx 19/121
15 oct, 2017
10/25/2017 Test Information

Answer. b

55. Which of the following sinus is not visible on Waters View

a. Maxillary

b. Sphenoid

c. Frontal

d. Posterior ethmoid

Solution. (d) Posterior ethmoid


Ref: Read the text below
Sol:

http://cbt.damsdelhi.com/damscbtadmin/admin/index.php?pageName=test/export&id=1710001
xxxxxxxxxxxxxxxxxxxxxxxxxxxxxxxxxxxxxxxxxxxxxxxxxxxxxxxxxxxxxxxxxxxxxxxxxxxxxxxxxxxxxx 20/121
15 oct, 2017
10/25/2017 Test Information

Answer. d

56. Chose most inappropriate statement regarding Audiometry:

a. Speech frequencies are 500,1000,2000 Hz

b. Air Bone Gap is feature of conductive deafness

c. Carhart Notch is at 2000 Hz in Bone conduction graph

d. Noise induced trauma is at 4000 Hz also known as U shape audiogram.

Solution. (d) Noise induced trauma is at 4000 Hz also known as U shape audiogram.
Ref: Read the text below
Sol:
1.Speech frequencies are 500,1000,2000 Hz
2.Air Bone Gap is feature of conductive deafness
3.Carhart Notch is at 2000 Hz in Bone conduction graph
4.Noise induced trauma is at 4000 Hz also known as acoustic Dip or Boiler's Notch
5. 'U' Shape audio gram is feature of congenital deafness.

Answer. d

57. Study the given image and comment on the best possible combination:

a. A-Rhinophyma B-Rhinoscleroma C-Rhinosporodiosis

b. B-Rhinophyma A-Rhinoscleroma C-Rhinosporodiosis

c. C-Rhinophyma B-Rhinoscleroma A-Rhinosporodiosis

d. A-Rhinophyma C-Rhinoscleroma B-Rhinosporodiosis

Solution. (a) A-Rhinophyma B-Rhinoscleroma C-Rhinosporodiosis


Ref: Read the text below
Sol:

Answer. a

58. Fetal nasal bone assessment is a non-invasive procedure that helps provide even greater assurance to patients undergoing their first trimester risk
assessment for aneuploidies, The gestational period must be

a. 11 to 13 wks

http://cbt.damsdelhi.com/damscbtadmin/admin/index.php?pageName=test/export&id=1710001
xxxxxxxxxxxxxxxxxxxxxxxxxxxxxxxxxxxxxxxxxxxxxxxxxxxxxxxxxxxxxxxxxxxxxxxxxxxxxxxxxxxxxx 21/121
15 oct, 2017
10/25/2017 Test Information
b. 2 to 3 wks

c. 3 to 5 wks

d. 5 to 9 wks

Solution. (a) 11 to 13 wks


Ref: Read the text below
Sol:

Fetal nasal bone assessment is a non-invasive procedure that helps provide even greater assurance to patients undergoing their first trimester risk
assessment for aneuploidies
The gestational period must be 11 to 13+6 weeks - the nasal bones first appear at a crown-rump length of 42 mm.

Answer. a

59. Which of the following structure is not removed during Vertical Partial Laryngectomy

a. Ventricle

b. Paraglottic space

c. Vocal cord

d. Arytenoid

Solution. (d) Arytenoid


Ref: Read the text below
Sol:

Answer. d

60. Epipharynx is also called-

a. Nasopharynx

b. Oropharynx

c. Laryngopharynx

d. Hypopharynx

Solution. (a) Nasopharynx


Ref: Read the text below
Sol:
The nasopharynx is the space above the soft palate at the back of the nose and connects the nose to the mouth, which allows a person to breathe through
the nose.
The soft palate separates thenasopharynx from the oropharynx, which sits just below the soft palate.

http://cbt.damsdelhi.com/damscbtadmin/admin/index.php?pageName=test/export&id=1710001
xxxxxxxxxxxxxxxxxxxxxxxxxxxxxxxxxxxxxxxxxxxxxxxxxxxxxxxxxxxxxxxxxxxxxxxxxxxxxxxxxxxxxx 22/121
15 oct, 2017
10/25/2017 Test Information

Answer. a

61. Medical examination of accused can be allowed at his request under which section?

a. 53 CrPC

b. 54 CrPC

c. 53(A) CrPC

d. 164(A) CrPC

Solution. (b) 54 CrPC


Ref: Read the text below
Sol:
CrPC - Examination of arrested person by medical practitioner at the request of the arrested person

Answer. b

62. Identify the injury

a. Shotgun entry

b. Multiple Rifled entry

c. Stab wounds

d. Lacerated wounds

Solution. (a) Shotgun entry


Ref: Read the text below
Sol:
Multiple pellet entry wounds are seen.
Range of firing more than 4 mts.

Answer. a

63. Application of medical knowledge in law enforcement is known as

a. Forensic medicine

b. Medical jurisprudence

c. Medical Ethics

d. Medical Etiquettes

Solution. (a) Forensic medicine


Ref: Read the text below

Sol:
Forensic medicine is the application of medical knowledge in administration of law and justice.

Answer. a

64. Which of the following is a feature of Cobra

http://cbt.damsdelhi.com/damscbtadmin/admin/index.php?pageName=test/export&id=1710001
xxxxxxxxxxxxxxxxxxxxxxxxxxxxxxxxxxxxxxxxxxxxxxxxxxxxxxxxxxxxxxxxxxxxxxxxxxxxxxxxxxxxxx 23/121
15 oct, 2017
10/25/2017 Test Information

a. Pit present between eye and nostril

b. 4th infralabial is the largest

c. 3rd supralabial touches the eye and nostril

d. All of the above.

Solution. (c) 3rd supralabial touches the eye and nostril


Ref: Read the text below
Sol:
Pit is present between eye and nostril in Pit viper. 4th infralabial is the largest in Krait.

Answer. c

65. Nystens rule deals with

a. Rigor mortis

b. Hypostasis

c. Algor mortis

d. Putrefaction

Solution. (a) Rigor mortis


Ref: Read the text below
Sol:
Nystens rule is about the sequence of onset of rigor mortis.

Answer. a

66. Sexual gratification obtained by the use enema is known as

a. Mixoscopia

b. Klismaphilia

c. Partialism

d. Urolagnia

Solution. (b) Klismaphilia


Ref: Read the text below
Sol:
Mixoscopia is pleasure obtained by watching people perform intercourse.
Urolagnia is pleasure by sight or smell of urine.

Answer. b

67. Smell resembling carrots is found in which poisoning

a. Hemlock

b. Calotropis

c. Plumbago

d. Blister beetle

Solution. (a) Hemlock


Ref: Read the text below
Sol:
Hemlock is a peripheral nerve poison which causes ascending motor paralysis. Used to kill Socrates.

Answer. a

68. Kronlein shot is

http://cbt.damsdelhi.com/damscbtadmin/admin/index.php?pageName=test/export&id=1710001
xxxxxxxxxxxxxxxxxxxxxxxxxxxxxxxxxxxxxxxxxxxxxxxxxxxxxxxxxxxxxxxxxxxxxxxxxxxxxxxxxxxxxx 24/121
15 oct, 2017
10/25/2017 Test Information
a. Burst injury of skull

b. Placing a bullet in a stab wound

c. Firing a bullet through a stab wound

d. Firing 5 bullets together

Solution. (a) Burst injury of skull


Ref: Read the text below
Sol:
Burst injury of skull is known as kronlein shot.
Placing a bullet in a stab wound to mislead the investigation is Rayalaseema phenomenon.

Answer. a

69. Injury caused by blast wind is

a. 10 blast injury

b. 20 blast injury

c. 30 blast injury

d. 40 blast injury

Solution. (c) 30 blast injury


Ref: Read the text below
Sol:
10 blast injury is due to shock wave.
20 blast injury is due to projectiles of bomb.
30 blast injury is due to blast wind
40 blast injury is due to other miscellaneous factors.

Answer. c

70. Study the picture given below and comment on the posture shown:

a. Rigor mortis

b. Cadaveric spasm

c. Post mortem caloricity

d. Pugilistic attitude

Solution. (d) Pugilistic attitude


Ref: Read the text below
Sol:
The pugilistic attitude is an extremely typical post-mortem body posture that is similar to a
kneeling position but is involuntary. ( MM: "Force Majeure")
The condition generally occurs several hours after death and is caused by a coagulation of muscle proteins when the body is exposed to extremely
high temperatures.

http://cbt.damsdelhi.com/damscbtadmin/admin/index.php?pageName=test/export&id=1710001
xxxxxxxxxxxxxxxxxxxxxxxxxxxxxxxxxxxxxxxxxxxxxxxxxxxxxxxxxxxxxxxxxxxxxxxxxxxxxxxxxxxxxx 25/121
15 oct, 2017
10/25/2017 Test Information
Answer. d

71. All are causes of ambiguous genitalia in XY child at birth except

a. 21 hydroxylase deficiency

b. 5 alpha reductase deficiency

c. Partial AIS

d. 17beta OH dehydrogenase deficiency

Solution. A
Explanation
21hydroxylase deficiency causes ambiguous genitalia in an XX child and not XY
5 alpha reductase deficiency does not allow the formation of active form of testosterone called as DHT and hence the external genitalia do not convert into
male like structures. In partial AIS the androgen receptor is partially sensitive to androgens and hence even in the presence of testosterone the external
genitalia are only partially virilized and hence appear ambiguous
The deficiency of enzyme 17 beta hydroxysteroid dehydrogenase type 3 causes defects in testosterone biosynthesis, the serum testosterone levels in these
patients are towards lower normal which leads to impairment in virilization of external genitalia

Answer. a

72. A study was done on a group of girls with delayed puberty. All have hypergonadotropic hypogonadism, what is next best investigation

a. MRI brain

b. DHEAS

c. Karyotype

d. Prolactin and TSH

Solution. C
Hypergonadotropic hypogonadism indicates cause in the gonadal tissue most commonly gonadal dysgenesis and hence the feedback inhibition is defective
leading to high levels of LH and FSH The best investigation to establish the cause is karyotyping

The most common type of gonadal dysgenesis is turners syndrome or monosomy X Gonadal dysgenesis presents as primary amenorrhoea without
secondary sexual characters The investigation of choice for primary amenorrhoea is Karyotyping

Answer. c

73. All the following may be used for diagnosis of cervical incompetence except

a. Cervical length less than 2.5cm

b. Internal os diameter more than 2 cm in second trimester

c. Passage of number 8 hegars dilator without resistance in premenstrual phase

d. No8 foley catheter with bulb filled with 2cc saline can be pulled out easily

Solution. D
Explanation
The diagnosis of cervical incompetence in pregnancy is cervical length in second trimester less than 25mm is diagnostic of cervical incompetence
An internal os diameter of more than or equal to 1.5cm in first trimester and more than or equal to 2cm in second trimester is diagnostic of cervical
incompetence
Membrane protruding at a distance 20mminto the cervical canal is diagnostic of incompetence Passage of number 8 hegars dilator without resistance
through the internal os is diagnostic of cervical incompetence in non pregnant state and this test has to be done in the premenstrual phase.
Foley catheter no.16 passed into uterine cavity and bulb filled with with 2cc normal saline can be pulled out easily

Answer. d

74. Primigravida with 39 weeks pregnancy has a persistent blood pressure of 150/110 and also has albuminuria, she has no other symptoms. Her blood
pressure was normal in early pregnancy nut had one record of 150/90 at 30 weeks. What is the diagnosis?

a. Gestational hypertension

b. severe preeclampsia

http://cbt.damsdelhi.com/damscbtadmin/admin/index.php?pageName=test/export&id=1710001
xxxxxxxxxxxxxxxxxxxxxxxxxxxxxxxxxxxxxxxxxxxxxxxxxxxxxxxxxxxxxxxxxxxxxxxxxxxxxxxxxxxxxx 26/121
15 oct, 2017
10/25/2017 Test Information
c. mild Preeclampsia

d. Chronic hypertension with superimposed preeclampsia

Solution. B
It is not chronic hypertension as her blood pressure was normal in early pregnancy
It is also not gestational hypertension as she also has albuminuria
It is a case of severe preeclampsia because the diastolic blood pressure is 110 mmhg Criteria to say it is severe preeclampsia areany one or more of the
following
1) Systolic blood pressure greater than or equal to 160mmhg or diastolic blood pressure more than or equal to 110mmhg on two occasions 4 hrs apart
2) Serum transaminases more than twice the upper limit of normal or severe persistent right upper quadrant pain or epigastric pain not responsive to
medication
3) Platelet count less than 1 lakh
4) Serum creatinine greater than 1.1
5) Pulmonary edema
6) New onset cerebral or visual symptoms

Answer. b

75. Which of the following is not a sling operation for prolapse

a. Manchester

b. Shirodkar

c. Khanna

d. Abdominocervicopexy

Solution. A
EXPLANATION
Manchester is not a sling surgery, it is a conservative surgery which preserves the menstrual functions in a patient of prolapse. The basic surgical principle
of Manchester repair also called a sfothergills repair is to maintain anteversion of uterus by placating the cardinal ligaments in front of cervix so that the
cervix is pulled upwards and backwards along with amputation of cervix. After these steps anterior colporrhaphy and pelvic floor repair may be done in
order to correct
cystocoele, rectocoele and perineal defect.
Rest of the threes surgeries shirodkar, Khanna and abdominocervicopexy are sling surgeries done for prolapse. The most common sling surgery that is
done is modified shirodkars abdominal sling surgery.

Answer. a

76. Which of the following is not true about physiological changes of pregnancy

a. Compression of IVC causes supine hypotension

b. Hyperinsulinemia

c. Diaphragmatic excursions increase

d. TSH increases

Solution. D
EXPLANATION
In pregnancy the TSH levels reduce in the first trimester and in the second and third trimester the TSH value may gradually return to nonpregnant normal
range.
TBG levels increase in pregnancy. In general pregnancy is considered a euthyroid state.
Compression of IVC by the gravid uterus causes reduction of venous return and hence cardiac output decreases. This is called as supine hypotension
syndrome
Pregnancy is characterized by hyperinsulinemia because of insulin resistance which is primarily caused by HPL
The movement of diaphragm increases in pregnancy which is the diaphragmatic excursions. Diaphragm rises in pregnancy by 4cm. The total lug capacity
slightly decreases but the vital capacity remains unchanged .

Answer. d

77. All the following are true about estrogens except

a. Ferning of cervical mucus

b. Beta receptor is present on uterus

c. Initiation of LH surge

d. Estrone increases in PCOS


http://cbt.damsdelhi.com/damscbtadmin/admin/index.php?pageName=test/export&id=1710001
xxxxxxxxxxxxxxxxxxxxxxxxxxxxxxxxxxxxxxxxxxxxxxxxxxxxxxxxxxxxxxxxxxxxxxxxxxxxxxxxxxxxxx 27/121
15 oct, 2017
10/25/2017 Test Information

Solution. B
EXPLANATION
The ERalpha is found in endometrium, breast cancer cells, ovarian stroma cells, and hypothalamus
The ERbeta has been documented in ovarian granulosa cells, kidney brain,bone, heart, lungs Prostate and endothelial cells
The estrogen receptors are intranuclear receptors
High levels of estrogens are required to initiate the LH surge the minimum requiremen for 48 hrs.
Estrogens bring about ferning of cervical mucus
In PCOS the E1 increases while total E2 is usually in normal range. The ratio of E2/E1 is reversed.

Answer. b

78. What is the capacity of the syringe shown below

a. 40ml

b. 60ml

c. 80ml

d. 100ml

Solution. B
EXPLANATION
This is a MVA syringe which has a capacity of 60 ml and is used for MTP upto 12 weeks It is an alternative to suction evacuation when electronic
machine is not available

As per the MTP act, MTP can be done upto 20 weeks.


The medical outpatient abortion with mifepristone and misoprostol can be done upto 7 weeks

Answer. b

79. What is the contraception of choice in a woman with menorrhagia and hypertension

a. OCP

b. Transdermal patch

c. Vaginal ring

d. mirena

Solution. D
Since the patient is a known case of hypertension combined E+P compounds are avoided and only progestational agents are preferred so mirena is the
contraception of choice here also it will reduce the menstrual blood loss as it brings about endometrial atophy.
Transdermal patch and vaginal ring are also E+P agents and hence they have the same absolute contraindications as OCP.

Answer. d

80. Wertheims hysterectomy is done for which of the following stage of cancer cervix

a. Stage 1a2

b. Stage 1b1

http://cbt.damsdelhi.com/damscbtadmin/admin/index.php?pageName=test/export&id=1710001
xxxxxxxxxxxxxxxxxxxxxxxxxxxxxxxxxxxxxxxxxxxxxxxxxxxxxxxxxxxxxxxxxxxxxxxxxxxxxxxxxxxxxx 28/121
15 oct, 2017
10/25/2017 Test Information
c. Both A and B

d. Stage 1A1

Solution. C
Wertheims hysterectomy or type 2 is done in stage 1a2 and 1b1 if the tumour is less than 2 cm For tumours larger than 2 cm we will do radical
hysterectomy or type 3 hysterectomy For tumours larger than 4 cm chemoradiation is the preferred primary treatment For stage 1a1 simple hysterectomy
is done if family is complete

Answer. c

81. All the following are first trimester findings of downs except

a. Decreased AFP

b. Decreased PAPPA

c. Increased HCG

d. Increased nuchal translucency

Solution. A
Serum AFP is a marker of second trimester downs screening not first trimester
The other three tests are done as first trimester screening and when done together are called as combined test.
Serum beta hcg and PAPPA are called as dual test.

Answer. a

82. Which of the following is the most common site of endometriosis

a. Broad ligament

b. POD

c. Rectovaginal septum

d. Uterosacral ligaments

Solution. B
The most common site of endometriosis is ovary followed by the POD.
Endometriosis is typically seen in the age group of 25-35 years and may present with any combination of symptoms of pain, infertility and adnexal mass.
The IOC is laparoscopy and it also helps to confirm the diagnosis by taking a biopsy from the endometrial deposits

Answer. b

83. Name the instrument shown below

a. Artery forceps

b. Needle holder

c. Kochersforcep

d. Green armytage

http://cbt.damsdelhi.com/damscbtadmin/admin/index.php?pageName=test/export&id=1710001
xxxxxxxxxxxxxxxxxxxxxxxxxxxxxxxxxxxxxxxxxxxxxxxxxxxxxxxxxxxxxxxxxxxxxxxxxxxxxxxxxxxxxx 29/121
15 oct, 2017
10/25/2017 Test Information
Solution. A

Answer. a

84. The following ultrasound image of ovary is characteristic to

a. Dermoid

b. Endometrioma

c. Ovarian cancer

d. Follicular cyst

Solution. A

http://cbt.damsdelhi.com/damscbtadmin/admin/index.php?pageName=test/export&id=1710001
xxxxxxxxxxxxxxxxxxxxxxxxxxxxxxxxxxxxxxxxxxxxxxxxxxxxxxxxxxxxxxxxxxxxxxxxxxxxxxxxxxxxxx 30/121
15 oct, 2017
10/25/2017 Test Information

EXPLANATION
The ultrasound image is characteristic of a dermoid cyst also called as mature cystic teratoma The cyst shows a white structure at one edge and this is
called as rokitanskyprotruberance Dermoid is the most common ovarian tumour in reproductive age woman and is usually unilateral although it may be
bilateral in 10%cases.

Answer. a

85. Which of the following is not true about HELLP syndrome

a. Blood pressure may be normal

b. Nulliparity is not a risk factor

c. LDH is not a part of criteria for diagnosis

d. Elevated conjugated bilirubin

Solution. D
HELLP syndrome although is a feature of severe preeclampsia but in upto15% females the blood pressure may be normal, it is more common in
multigravida. It is characterized by pain which is classically epigastric or right upper quadrant pain.
It is also characterized by hemolysis and it is the unconjugated bilirubin which is raised not conjugated. Although there is haemolysis LDH is not a
component of diagnostic criteria .
The diagnostic criteria include
1) Schistocytes on peripheral smear
2) Total serum bilirubin grater than or equal to 1.2mg/dl
3) AST greater than or equal to 70IU
4) Platelet count less than 1 lakh

Answer. d

86. All are soft marker of aneuploidy except

a. Echogenic bowel foci

b. Nuchal translucency

c. Hypoplastic nasal bone

d. Single palmar crease

Solution. B
EXPLANATION
Soft markers are second trimester ultrasound findings that may be seen in normnal babies but their presence increases the risk of aneuploidy

The most important soft markers are absent nasal bone and increased nuchal fold the thickness The other markers of aneuploidy on ultrasound
2) Abnormal Doppler markers like reversed end diastolic flow , abnormal flow in ductus venosus and tricuspid regurgitation.
3) structural anomalies
4) fetal growth restriction
5) single umbilical artery

Answer. b

87. All are true about puberty except


http://cbt.damsdelhi.com/damscbtadmin/admin/index.php?pageName=test/export&id=1710001
xxxxxxxxxxxxxxxxxxxxxxxxxxxxxxxxxxxxxxxxxxxxxxxxxxxxxxxxxxxxxxxxxxxxxxxxxxxxxxxxxxxxxx 31/121
15 oct, 2017
10/25/2017 Test Information

a. Central precocious puberty has high levels of LH and FSH

b. First sign in boys is testicular enlargement

c. First visible sign in girls is thelarche

d. Estrogen brings about pubarche

Solution. D
EXPLANATIONS
Appearance of pubic and axillary hair although are secondary sexual characters in females but are not dependent on estrogen they are brought about by
androgens
Precocious puberty is defined as appearance of the secondary sexual characters in females before the age of 8 years and in boys before the age of 9 years
In central precocious puberty the cause is premature activation of hypothalamic pituitary axis and hence the levels of LH and FSH are high while in
peripheral precocious puberty the level of gonadotropins is low.

Answer. d

88. A woman taking OCPs is at an increased risk stroke with which of the following

a. Smoking

b. Hypertension

c. Diabetes mellitus

d. Hypercholesterolemia

Solution. B
EXPLANATION
Although all the mentioned options are increasing the risk of stroke in an OCP user the highest association is with hypertension

The following are the absolute contraindications of OCP for contraception


1) VTE
2) CAD
3) Stroke
4) Known case of breast cancer
5) Hypertension blood pressure more than 150/100
6) Diabetes with vasculopathy
7) Focal migraine
8) Suspected pregnancy
9) Deranged liver enzymes
10) Age more than 35 years and smoker

Answer. b

89. Important parameters for semen analysis are all except

a. Motility

b. Semen fructose

c. Concentration

d. Volume

Solution. B
The most important semen analysis parameters are sperm concentration and sperm motility The most important parameter for IVF is motility while the
most important parameter for ICSI is morphology
The following are WHO parameters for semen analysis
1) Volume more than 1.5ml
2) Ph more than 7.2
3) Total count more than 39 million per ejaculate
4) Sperm concentration more than equal to 15million per ml
5) Total motility greater than equal to 40%
6) Forward motility greater than equal to 32%
7) Sperm morphology more than equal to 4%
8) Sperm vitality more than equal to 58%
9) Leucocyte count less than 1 million /ml

Answer. b

http://cbt.damsdelhi.com/damscbtadmin/admin/index.php?pageName=test/export&id=1710001
xxxxxxxxxxxxxxxxxxxxxxxxxxxxxxxxxxxxxxxxxxxxxxxxxxxxxxxxxxxxxxxxxxxxxxxxxxxxxxxxxxxxxx 32/121
15 oct, 2017
10/25/2017 Test Information
90. True statement regarding partial mole is

a. Fertilization of empty ovum by two sperms

b. It is monospermic

c. Is associated with early onset preeclampsia

d. Placenta can show enlarged cystic spaces on ultrasound

Solution. D
The placenta of partial mole on ultrasound shows focal hydropic change which are seen as cystic spaces along with fetal tissue in the uterus
While in complete mole there is no fetal tissue and complete hydropic change of placenta The partial moles are dispermic and triploid and 90% of them
have karyotype of 69XXY The medical complications like preclampsia, thyroid storm, hyperemesis, pulmonary embolism are seen in complete mole only
not in partial mole

Answer. d

91. A 26 year old lady for her infertility evaluation has bilateral cornual block on HSG what is the most appropriate management for this patient

a. IVF

b. GIFT

c. Laprohysteroscopy

d. IUI

Solution. C
Bilateral blocks on HSG need confirmation which is done by laparoscopy and we will do the therapeutic part with it as a single procedure which is
hysteroscopic cannulation.
Hsg is the IOC for tubal patency while the best investigation for tubal assessment is is diagnostic laparoscopy .
HSG is to be done in the postmenstrual phase and can be done anytime between day 5 to day 11 of the cycle
Out of all the blocks on hysteroscopy the corneal block has best prognosis as many cases are physiological.
Suspected pregnancy and active pelvic infection are absolute contraindications to do the procedure

Answer. c

92. A primigravida presents at 8 weeks with pain and spotting, on examination the os is closed and ultrasound shows a intrauterine sac with cardiac
activity. What did you want to next for the patient

a. Give misoprostol

b. Dilatation and curettage

c. Bed rest

d. Send serial beta hcg

Solution. C
She is a case of threatened abortion and should be managed by rest and progesterone therapy and follow up with ultrasound
Closest differential diagnosis is missed abortion but in it the cardiac activity is absent on ultrasound, missed abortion needs MTP
While in inevitable abortion the os is open and no products of conception have been expelled

Answer. c

93. A Patient presents with post hysterectomy vault prolapse, this represents what level loss of support

a. Level 3

b. Level 2

c. Level 1

d. Both a and b

Solution. C
Vault prolapse is considered as level 1 defect.
Vault prolapse is seen in post hysterectomy patients and can be prevented by uterosacral suspension if done at the time of hysterectomy
The best surgery of vault prolapse treatment is abdominal sacrocolpopexy

http://cbt.damsdelhi.com/damscbtadmin/admin/index.php?pageName=test/export&id=1710001
xxxxxxxxxxxxxxxxxxxxxxxxxxxxxxxxxxxxxxxxxxxxxxxxxxxxxxxxxxxxxxxxxxxxxxxxxxxxxxxxxxxxxx 33/121
15 oct, 2017
10/25/2017 Test Information
Enterocoele and uterocervical descent are also considered as level 1 defect
While cystocoele is level 2 and rectocoele is a level 3 defect

Answer. c

94. All the following are pathological in pregnancy except

a. Increase in respiratory rate

b. Progressive dyspnoea

c. Cardiomegaly

d. Increased diaphragmatic excursions

Solution. D
Respiratory rate shows no change in pregnancy
Although exrertional dyspnea is physiologic but progressive dyspnea represents worsening oh NYHA grade and is pathological
Cardiac silhouette on Xray appears big because eof rotation oh heart but there is no cardiomegaly.
Cardiomegaly is pathologic
Diaphragm rises in pregnancy by 4cm and the excursions increase.

Answer. d

95. All are components of AMTSL except

a. Oxytocin 10IU i/m bolus

b. Delayed cord clamping

c. Controlled cord traction

d. Uterine massage

Solution. D
Uterine massage is done for treatment of PPH not prophylaxis
The component of AMTSL is intermittent uterine tone assessment instead
Oxytocin is the drug of choice for AMTSL when injectables cannot be given then misoprostol 600mcg per oral can be given
Early cord clamping is done when the baby needs resuscitation and in rh negative pregnancy Else the universal recommendation is delayed cord clamping.

Answer. d

96. Which of the following statement is incorrect regarding Jobs syndrome :

a. Pneumatoceles

b. Facial dysmorphism

c. Recurrent skin infections

d. Autosomal recessive inheritance

Solution. (d) Autosomal recessive inheritance


Ref: Read the text below
Sol:
1. Pattern of inheritance is Autosomal Dominant
Jobs syndrome
Also known as Hyper IgE syndrome
Autosomal dominant pattern of inheritance
Caused by mutation in the gene coding for the transcription factor STAT3
Characterised by recurrent skin and lung infections that can be complicated by
pneumatoceles

Answer. d

97. Each of the following is an indication for surgical intervention in those patients who present with endocarditis except

a. Organism isolated

b. Recurrent embolism

http://cbt.damsdelhi.com/damscbtadmin/admin/index.php?pageName=test/export&id=1710001
xxxxxxxxxxxxxxxxxxxxxxxxxxxxxxxxxxxxxxxxxxxxxxxxxxxxxxxxxxxxxxxxxxxxxxxxxxxxxxxxxxxxxx 34/121
15 oct, 2017
10/25/2017 Test Information

c. Extravalvular infection

d. Aortic regurgitation

Solution. (d) Aortic regurgitation


Ref: Read the text below
Sol:
Antibiotics remain the mainstay for the treatment of endocarditis, yet occasionally surgery is required.
Patients needing surgery include those with fungal isolates, extravalvular infection such as a valve ring abscess or purulent pericarditis, recurrent
embolism, CHF despite medical theraphy, prosthetic valve dehiscence or obstruction, and persistent bacteremia or recurrence of infection despite
appropriate antibiotic theraphy.
Aortic regurgitation is not a primary reason for surgery if it is well-tolerated, but some patients need surgical repair when the infectious process is
cleared.

Answer. d

98. All of the following are features of Juvenile Myoclonic Epilepsy except :

a. Myoclonus on awakening

b. GTCS

c. Automatism

d. Absence seizures

Solution. (c) Automatism


Ref: Read the text below
Sol:
Automatism is a feature of complex partial seizure
JUVENILE MYOCLONIC EPILEPSY
- Age group Early adolescence
 -C/F bilateral myoclonic jerks
 -Most frequent in the morning after awakening
 -Provoked by sleep deprivation
 -CONSCIOUSNESS IS PRESERVED usually
 -Other seizure types GTCS, asbence
 -Treatment response is good
 -Complete remission is uncommon

Answer. c

99. Deep brain stimulation for drug-resistant epilepsy targets which of the following structures?

a. Anterior nucleus of the thalamus

b. Centromedian nucleus of the thalamus

c. Cingulate gyrus

d. Hippocampus

Solution. (a) Anterior nucleus of the thalamus


Ref: Read the text below
Sol:
The anterior nucleus of the thalamus is part of the Papez circuit and is therefore believed to be a relay station for information passing from the
amygdala and hippocampus to the cerebral cortex.
Data have shown that inhibition of the anterior nucleus may result in prevention or cessation of seizures

Answer. a

100. Cerebral ischemia occurs when cerebral blood flow is less than :

a. 10ml/100 g/min

b. 20ml/100g/min

c. 40ml/100g/min

d. 50 ml/100g/min

Solution. (b) 20ml/100g/min

http://cbt.damsdelhi.com/damscbtadmin/admin/index.php?pageName=test/export&id=1710001
xxxxxxxxxxxxxxxxxxxxxxxxxxxxxxxxxxxxxxxxxxxxxxxxxxxxxxxxxxxxxxxxxxxxxxxxxxxxxxxxxxxxxx 35/121
15 oct, 2017
10/25/2017 Test Information
Ref: Read the text below
Sol:
Resting cerebral blood flow is 54ml/100 g/min
Decrease in cerebral blood flow to zero causes death of brain tissue within 4-10 min
Values <16-18 ml/100 g tissue per minute cause infarction within an hour

Values <20 ml/100 g tissue per minute cause ischemia without infarction unless prolonged for several hours or days

Answer. b

101. A 60-year-old woman develops generalized seizure activity lasting 10 minutes; seizure activity appears to arrest after administration of 4 mg of
IV lorazepam. She has chronic kidney disease but is otherwise in good health. Which of the following is the best next pharmacological step in
management?

a. Continuous midazolam infusion

b. Continuous propofol infusion

c. IV fosphenytoin

d. IV ketamine

Solution. (c) IV fosphenytoin


Ref: Read the text below
Sol:
Regardless of whether prolonged seizure activity stops after the administration of an appropriate dose of a benzodiazepine, rapid administration of a
longer-acting anticonvulsant is generally recommended.
This allows for prevention of additional seizures as the effect of the benzodiazepine wears off over the course of several hours.
Of the options listed, fosphenytoin is the preferred option.
A continuous infusion of propofol or midazolam is not indicated in this setting unless clinical or EEG evidence of ongoing seizures (ie, refractory status
epilepticus) exists.
If the patient does not wake up rapidly, an urgent EEG should be obtained

Answer. c

102. A child was being evaluated by an endocrinologist for precocious puberty. His mother was also giving history of psychomotor delay and
uncontrollable laughing episodes in the child. An MRI was ordered. What is the likely diagnosis ?

a. Craniopharyngioma

b. Prolactinoma

c. Hypothalamic hamartoma

d. Pinealoma

http://cbt.damsdelhi.com/damscbtadmin/admin/index.php?pageName=test/export&id=1710001
xxxxxxxxxxxxxxxxxxxxxxxxxxxxxxxxxxxxxxxxxxxxxxxxxxxxxxxxxxxxxxxxxxxxxxxxxxxxxxxxxxxxxx 36/121
15 oct, 2017
10/25/2017 Test Information
Solution. (c) Hypothalamic hamartoma
Ref: Read the text below
Sol:
Hypothalamic hamartoma presents with gelastic seizures/laughing associated seizures

Answer. c

103. All of the following antiarrhythmic agents interact predominantly with the fast sodium channels except

a. Procainamide

b. Flecainide

c. N-Acetyprocainamide (NAPA)

d. Lidocaine

Solution. (c) N-Acetyprocainamide (NAPA)


Ref: Read the text below
Sol:

Antiarrhythmic agents can be broadly grouped according to their electrophysiologic effect on the heart, known as the Vaughan Williams
classification.
Class I drugs are those that act predominantly on the fast sodium channels, including lidocaine, mexiletine, tocainide, procainamide, quinidine,
disopyramide, flecainide , encainide, or propafenone.
Class II drugs are B-blockers. Class III drugs are those that prolong repolarization by increasing the action potential duration, including bretylium,
amiodarone , sotalol, and NAPA.
Class IV drugs are calcium channel blockers.
This classification is very simplistic and has multiple flaws. Several attempts to reclassify antiarrhythmic agents have been proposed, but at the
present time none have been widely adopted.

Answer. c

104. Most common cause of GHRH mediated acromegaly ?

a. Pancreatic islet cell tumor

b. Pheochromocytoma

c. Carcinoid tumor

d. Medullary thyroid cancer

Solution. (c) Carcinoid tumor


Ref: Read the text below
Sol:
- GH hypersecretion is usually the result of somatotrope adenoma.
 -Rarely ectopic GHRH secretion can also mediate acromegaly.
 -The most common cause of GHRH mediated acromegaly is a chest or abdominal carcinoid tumor.

http://cbt.damsdelhi.com/damscbtadmin/admin/index.php?pageName=test/export&id=1710001
xxxxxxxxxxxxxxxxxxxxxxxxxxxxxxxxxxxxxxxxxxxxxxxxxxxxxxxxxxxxxxxxxxxxxxxxxxxxxxxxxxxxxx 37/121
15 oct, 2017
10/25/2017 Test Information

Answer. c

105. Facial angiofibromas and collagenomas are seen in :

a. MEN 1

b. MEN 2A

c. MEN 2B

d. MEN 3

Solution. (a) MEN 1


Ref: Read the text below
Sol:
MEN 1

- Wermers syndrome. Menin gene chromosome 11q
 -Triad of PPP......Parathyroid, Pancreatic islets, ant Pituitary
 -Most common feature Primary hyperparathyroidism
 -Most common enteropancreatic tumor associated Gastrinoma
 -Most common ant pituitary tumor associated - Prolactinoma
 -Other features
- Adrenal cortical tumors
- Carcinoid tumors

- Facial angiofibromas
- Collagenomas
- Lipomas

Answer. a

106. A patient presented with a history of prolonged fever of 3 weeks duration. Given below are the fundus finding and a general examination
finding. What is the diagnosis ?

http://cbt.damsdelhi.com/damscbtadmin/admin/index.php?pageName=test/export&id=1710001
xxxxxxxxxxxxxxxxxxxxxxxxxxxxxxxxxxxxxxxxxxxxxxxxxxxxxxxxxxxxxxxxxxxxxxxxxxxxxxxxxxxxxx 38/121
15 oct, 2017
10/25/2017 Test Information

a. Tuberculosis

b. Syphilis

c. Infective Endocarditis

d. HIV

Solution. (c) Infective Endocarditis


Ref: Read the text below
Sol:
Given are the images of Roth spots and Janeway lesions seen in Infective Endocarditis
- Roth spots - They are retinal haemorrhages with small clear centres. (shown in white arrows) Janeway lesions-They are erythematous,non-tender
nodular lesions found in palms and soles

Answer. c

107. Constrictive pericarditis is associated with each of the following except-

http://cbt.damsdelhi.com/damscbtadmin/admin/index.php?pageName=test/export&id=1710001
xxxxxxxxxxxxxxxxxxxxxxxxxxxxxxxxxxxxxxxxxxxxxxxxxxxxxxxxxxxxxxxxxxxxxxxxxxxxxxxxxxxxxx 39/121
15 oct, 2017
10/25/2017 Test Information
a. Rheumatic heart disease

b. Radiaion theraphy for Hodgkins disease

c. Bacterial (Purulent) pericarditis

d. Hypothyroidism

Solution. (d) Hypothyroidism


Ref: Read the text below
Sol:
Constrictive pericarditis is a condition in which the filling of the heart is limited by the compressive nature of the pericardium.
The pericardium acts as a rigid shell around the heart, imposing continuous positive pressure.
The filling is, therefore, compromised specially in diastole when filling is at its maximum. This results in an early diastolic descent followed by a rise
and plateau (dip and plateau or square root sign).
There are several disease processes that have been attributed to constrictive pericarditis including rheumatic heart disease, radiation theraphy,
tuberculosis, and purulent pericarditis.
Hypothyroidism, however causes pericardial effusion.

Answer. d

108. All of the following are typical features of mitral stenosis found on physical examination except

a. Presystolic murmur

b. Loud S1

c. Soft S3

d. Mid-to late diastolic murmur

Solution. (c) Soft S3


Ref: Read the text below
Sol:
The ausculatory findings of mitral senosis (MS) are directly related to the level of pulmonary vascular resistance and cardiac output.
A loud first heart sound and an OS are the hallmarks of MS. An accentuated second heart sound, specifically the aortic component suggests pulmonary
hypertension.
A presystolic murmur occurs as a result of LA systole, delaying the closure of the merges with the first heart sound.
The typical diastolic murmur heard with mitral stenosis is loudest in mid-diastole. As the severity of the valve increases, or with a increased heart
rate, the murmur may extend into late diastole.
The filling of the left ventricle in MS is normal or slow due to the obstruction; therefore, an S3 would be unlikely or would suggest a regurgitant
component to the lesion.

Answer. c

109. Identify the finding which is reversible on physical examination?

a. Rheumatoid nodules

b. Heberdenss nodes

c. Jaccouds arthropathy

d. Pseudogout tophi

Solution. (c) Jaccouds arthropathy


Ref: Read the text below
Sol:
In Jaccouds arthropathy, the deformities are reducible and result mainly from soft-tissue abnormalities, such as laxity of ligaments, fibrosis of the
capsule, and muscular imbalance, rather than from destruction of the bone of joints, as occurs in rheumatoid arthritis.

http://cbt.damsdelhi.com/damscbtadmin/admin/index.php?pageName=test/export&id=1710001
xxxxxxxxxxxxxxxxxxxxxxxxxxxxxxxxxxxxxxxxxxxxxxxxxxxxxxxxxxxxxxxxxxxxxxxxxxxxxxxxxxxxxx 40/121
15 oct, 2017
10/25/2017 Test Information
Jaccoud's arthropathy was initially described in relation to rheumatic fever, but now more cases are seen in association with SLE, other connective
tissue diseases and neoplasias.

Answer. c

110. Which of the following is the best initial treatment for symptomatic hypercalcemia?

a. Diuresis with a loop diuretic such as furosemide

b. Mithramycin (plicamycin)

c. Hydration with dextrose

d. Hydration with isotonic saline

Solution. (d) Hydration with isotonic saline


Ref: Read the text below
Sol:
Patients with hypercalcemia generally present with volume depletion and must be hydrated before a diuretic is given.

It is best to hydrate a patient with isotonic saline, because saline stays in the vascular system
longer than other solutions, normalizes the glomerular filtration rate, and increases renal tubular sodium and calcium clearance.
Mithramycin (plicamycin) and calcitonin can be used to decrease serum calcium but should be used only if hydration and dieresis are not effective.
A diuretic should only be used after the patient is well-hydrated , and it should be a loop diuretic.
Mithramycin can take up to 48 hours for its maximum effect.

Answer. d

111. Which of the following statements regarding diabetes insipidus is true?

a. It presents with low plasma sodium

b. The plasma and urine osmolatily are equal.

c. Plasma osmolality is greater than 295 mOsm/kg

d. Treatment is fluid restriction

Solution. (c) Plasma osmolality is greater than 295 mOsm/kg


Ref: Read the text below
Sol:
In patients with intact thirst mechanism, the plasma sodium is normal to slightly increased.
The urine osmolality is less than plasma osmolality, less than 295 mOsm/kg. the plasma osmolality is greater than 295 mOsm/kg. urine volume is high,
generally more than 3L per 24 hour period. DDAVP (desmopressin acetate) is the best treatment for chronic central diabetes insipidus.

Answer. c

112. A patient with which of the following conditions should not be placed on a ketogenic diet?

a. Diabetes

b. Hyperlipidemia

c. Pancreatitis

d. Rett syndrome

Solution. (c) Pancreatitis


Ref: Read the text below
Sol:
Ketogenic diets are contraindicated in patients with pancreatitis, hepatic failure, primary carnitine deficiency, carnitine palmitoyl transferase I and II
deficiency, carnitine translocase deficiency, beta-oxidation defects, pyruvate carboxylase deficiency, and porphyria.
In the intensive care setting where diet therapy is being considered for treatment of refractory status epilepticus, the ketogenic diet is also contraindicated
in patients who cannot tolerate enteral feeds, including those with ileus, who are receiving a propofol infusion (to avoid fatal propofol infusion syndrome),
and in patients who have metabolic,hemodynamic, or cardiorespiratory instability

Answer. c

113. In addition to a risk of major congenital malformations, which of the following antiepileptic drugs is also associated with an elevated risk of autism
spectrum disorders and lower IQ at age 6

http://cbt.damsdelhi.com/damscbtadmin/admin/index.php?pageName=test/export&id=1710001
xxxxxxxxxxxxxxxxxxxxxxxxxxxxxxxxxxxxxxxxxxxxxxxxxxxxxxxxxxxxxxxxxxxxxxxxxxxxxxxxxxxxxx 41/121
15 oct, 2017
10/25/2017 Test Information

a. Carbamazepine

b. Lamotrigine

c. Phenobarbital

d. Valproate

Solution. (d) Valproate


Ref: Read the text below
Sol:
Women treated with valproate (either monotherapy or in combination with other
antiepileptics) are at elevated risk of giving birth to children with major congenital malformations.
In addition, of the medications listed above, valproate is also associated with a higher risk of neurodevelopmental effects in children exposed during
pregnancy, as measured by lower IQ at age 6 and a higher risk of autism spectrum disorders.

Answer. d

114. All of the following cardiac abnormalities should receive endocarditis prophylaxis for the appropriate indication except

a. Mitral valve prolapse with a regurgitation murmur

b. Hypertrophic cardiomyopathy

c. Isolated secundum atrial defect

d. Prosthetic cardiac valve

Solution. (c) Isolated secundum atrial defect


Ref: Read the text below
Sol:
Endocarditis can be a significant health problem.
The best treatment for endocarditis is to prevent it.
The American Medical association along with the ACC/ AHA have set guidelines concerning who should be considered for antibiotic prophylaxis
and which antibiotic is recommended for treatment.
Low or negligible risk lesions include isolated atrial septal defect (secundum type), mitralnary artery disease.
Those lesions that are considered intermediate to high risk include prosthetic heart valves, previous history of endocarditis, stenotic and regurgitant
lesions of the aortic and mitral valve, ventricular septal defect, coarctation of the aorta, and mitral valve prolapsed with a regurgitation murmur.

Answer. c

115. Human ehrlichosis is characterized by all of the following features except:

a. There are few asymptomatic cases.

b. Common symptoms include fever, headache, and malaise.

c. Rash is present in approximately 20% of patients.

d. Leucopenia and thrombocytopenia are usually present.

Solution. (a) There are few asymptomatic cases.


Ref: Read the text below
Sol:
Human ehrlichiosis is a tick-borne illness.
Serologic studies show that most cases are asymptomatic.
The illness when symptomatic, is characterized by fever, headache, malaise, anorexia, myalgias, chills, nauses and vomiting, weight loss, and
arthralgias.
It is similar to Rocky Mountain spotted fever, but it is less severe; a rash is a much less common finding.
Leucopenia and thrombocytopenia are commonly present during illness.

Answer. a

116. All of the following features are seen in myelopathies except

a. Facial sensory impairment

b. Brisk jaw jerks

c. Brisk pectoral jerks

d. Urgency and incontinence of micturition

http://cbt.damsdelhi.com/damscbtadmin/admin/index.php?pageName=test/export&id=1710001
xxxxxxxxxxxxxxxxxxxxxxxxxxxxxxxxxxxxxxxxxxxxxxxxxxxxxxxxxxxxxxxxxxxxxxxxxxxxxxxxxxxxxx 42/121
15 oct, 2017
10/25/2017 Test Information
Solution. (b) Brisk jaw jerks
Ref: Read the text below
Sol:
Trigeminal/Vth cranial nerve has its nucleus in pons as well has a spinal nucleus which extends down through the lower pons and medulla into spinal
cord as far as C3/C4.Through which sensory fibres will pass thus carrying sensations of the face.Hence due to high cervical cord lesion till C3/ Facial
sensory impairement will occur.
Jaw jerk-Afferent impulse of this reflex are carried through sensory portion of the trigeminal nerve the efferent impulse through trigeminal nerve with its
reflex centre in PONS.So this jerk like any other jerk will become brisk in a UMN esion/supranuclear lesion OR a lesion above pons,never in a lesion
with the spinal cord.So normal jaw jerk happens in cases of myelopathy

NEVER BRISK JAW JERK OCCURS.


Bladder is innervated from autonomic nervous system with relay via spinal cord.It is very evident that any disease of spinal cord/myelopathy will
obviously affect the bladder.

PECTORALIS REFLEX-With patients arm in midposition between abduction and adduction,the examiner places her finger as nearly as possible on the
tendon of pectoralis major muscle near its insertion on the greater tuberosity of the humerus.Tapping the finger causes adduction and slight internal
rotation of the arm of the shoulder.The contraction of the muscle may be felt but usually not seen in normal indivisual.In patients with cervical
spondylotic myelopathy ,a hyperactive/brisk pectoralis reflex indicates spinal cord compression at C2-3 and C3-4 levels.This reflex is mediated by
the medial and lateral(anterior thoracic) nerves (C5-T1).

Answer. b

117. Most common cause of brain tumor is :-

a. Astrocytoma

b. Glioblastoma

c. Meningioma

d. Metastatic.

Solution. (d) Metastatic.


Ref: Read the text below
Sol:
Most common brain tumors are metastases. The most common primary brain tumors are gliomas. The most common gliomas are astrocytomas.
Meningiomas are the second most common primary brain tumors after gliomas.
Tumors frequently associated with hemorrhage include high-grade gliomas, metastatic melanoma, and choriocarcinoma.

Answer. d

118. In high dose dexamethasone test cortisol production will be unchanged in :-

a. ACTH secreting pituitary tumor

b. Ectopic ACTH production

c. Both of these.

d. Exogenous steroid use.

Solution. (b) Ectopic ACTH production


Ref: Read the text below
Sol:
Suppression of cortisol production after high dose dexamethasone i.e 2 mg i/v for 2 days occurs in Pitutary adenoma and hypothalamic disorders while
no suppression occurs in ACTH secreting ectopic tumors as well as adrenal tumors causing cushing syndrome.

Answer. b

119. A 17-year old girl who was evaluated for short height was found to have an enlarged pituitary gland. Her T4 was low and TSH was increased.
Which of the following is the most likely diagnosis?

a. Pituitary adenoma

b. TSH-secreting pituitary tumour

c. Thyroid resistance

d. Primary hypothyroidism

Solution. (d) Primary hypothyroidism


Ref: Read the text below
Sol:
http://cbt.damsdelhi.com/damscbtadmin/admin/index.php?pageName=test/export&id=1710001
xxxxxxxxxxxxxxxxxxxxxxxxxxxxxxxxxxxxxxxxxxxxxxxxxxxxxxxxxxxxxxxxxxxxxxxxxxxxxxxxxxxxxx 43/121
15 oct, 2017
10/25/2017 Test Information
In Pitutary adenoma/TSH secreting pituitary tumor increased TSH with increased T3/4 will be seen.
In primary Hypothyroidism due to defect in the gland itself high TSH with low T4 will be seen.

Answer. d

120. Rapid, repetititve, coordinated and stereotyped movements, most of which can be mimicked are known as:

a. Chorea

b. Ballism

c. Athetosia

d. Tics

Solution. (d) Tics


Ref: Read the text below
Sol:
AKATHASIA: A subjective feeling of inner restlessness that is relieved by movements. (e.g. crossing and uncrossing legs, rocking back and forth and
pacing).TOC-Beta blockers.
ASTERIXIS: Sudden periods of cessation of muscle contraction, best seen when the patients arms are extended in front.
ATHETOSIS: Slow, sinuous, writhing movements, usually of the distal parts of the
limbs.Lesion Globus Pallidus(GAP)
BALLISMUS: wild flinging, movements that represent large amplitude proximal choreiform movements. Ballismus is often unilateral
(Hemiballismus).Lesion-STN.
CHOREA: Semipurposeful flowing movements that flit from one part of the body to another in a continuous and random pattern.Lesion-cuadate
nucleus.
DYSKINESIA: A general term for any excessive movement. The term dyskinesia is often used as an abbreviation for Tardive dyskinesia (repetitive
oral movements often seen in patients taking certain psychiatric medications).
DYSTONIA: Twisting movements that are often sustained for variable periods of time.
MYOCLONUS: Sudden, brief shock-like jerks.
TICS: Repetitive, stereotypic movements or sounds that are suppressible and that relieve a feeling of inner tension.
TREMOR: Regular, oscillatory movements that may be present at rest ore with action.

Answer. d

121. Identify the virus

a. Adenovirus

b. Small Pox virus

c. Orthomyxovirus

d. Rabies virus

Solution. (d) Rabies virus


Ref: Read the text below
Sol:
The rabies virus is bullet shaped with one end rounded or conical and the other planar or concave.
The lipoprotein envelope carries knob like spikes composed of glycoprotein G. These spikes do not cover the planar end.

Answer. d

http://cbt.damsdelhi.com/damscbtadmin/admin/index.php?pageName=test/export&id=1710001
xxxxxxxxxxxxxxxxxxxxxxxxxxxxxxxxxxxxxxxxxxxxxxxxxxxxxxxxxxxxxxxxxxxxxxxxxxxxxxxxxxxxxx 44/121
15 oct, 2017
10/25/2017 Test Information
122. A teenaged boy suffered a foot laceration while swimming in a polluted water area in a river. He did not seek medical treatment, and the wound
developed a foul-smelling exudate. One of the bacteria isolated from the abscess exudate was missing superoxide dismutase, catalase, and a
peroxidase. Which of the following statements best describes this microorganism

a. It is a capnophile

b. It is a facultative anaerobe

c. It is a microaerophile

d. It is an anaerobe

Solution. (d) It is an anaerobe


Ref: Read the text below
Sol:
Obligate anaerobes lacks Superoxide dismutase and catalase enzymes.
Foul smelling pus in anther indicator of anaerobic infections
Capnophile grows in high CO2 environment
Micro aerohile grows in low oxygen tension

Answer. d

123. Resazurin test is done to

a. Detect bacterial contamination in milk

b. Estimate water content in milk

c. Detect chemical impurities in the food

d. Detect pollution in the air

Solution. (a) Detect bacterial contamination in milk


Ref: Read the text below
Sol:
In the Resazurin test the dye resazurin on reduction passes through a series of colour changes from blue to pink after incubation with milk for a
particular period of time depending on the degree of contamination.

Answer. a

124. All of the following immunodeficiency diseases are due to chromosomal instability or defective repair of genes except?

a. Ataxia telangiectasia

b. Bloom syndrome

c. Chediak-Hegashi syndrome

d. Seckel syndrome

Solution. (c) Chediak-Hegashi syndrome


Ref: Read the text below
Sol:
It is due to Hereditary metabolic defects.

Answer. c

125. A hospitalized patient developed severe diarrhoea and pseudomembranous colitis within five days after antibiotic therapy was initiated. These
occurred as a result of which of the following:

a. Collagenase

b. Vacuolating toxin

c. Toxin A and B

d. Lecithinase

Solution. (c) Toxin A and B


Ref: Read the text below
Sol:

http://cbt.damsdelhi.com/damscbtadmin/admin/index.php?pageName=test/export&id=1710001
xxxxxxxxxxxxxxxxxxxxxxxxxxxxxxxxxxxxxxxxxxxxxxxxxxxxxxxxxxxxxxxxxxxxxxxxxxxxxxxxxxxxxx 45/121
15 oct, 2017
10/25/2017 Test Information
Clostridium difficile produces two toxins A and B.
Both toxins are present in stool samples.
Toxin A is enterotoxin causing the severe diarrhea, whereas toxins B is cytotoxic leading to the destruction of enterocytes resulting in
pseudomembranous colitis.

Answer. c

126. Which of the following best describes the characteristics of the mycoplasma?

a. Absence of a cell wall

b. Belonging to the class of Eukaryotes

c. Often evoke an IgM autoantibody response leading to human erythrocyte agglutination.

d. Typically colonize the gastrointestinal tract

Solution. (a) Absence of a cell wall


Ref: Read the text below
Sol:
Mycoplasma are the smallest living organisms, and they do not have cell walls but rather only have cell membranes.
Thus they are typically resistant to antibiotics that interface with cell wall synthesis. Because of their absence of a cell wall, they are not usually
detected on Gram stain.

Answer. a

127. Find the true statement regarding exotoxin:

a. They are very stable and resist most physical and chemical agents

b. Highly antigenic

c. Produced abundantly by gram negative bacilli

d. Lipopolysaccharide protein complex in nature

Solution. (b) Highly antigenic


Ref: Read the text below
Sol:

Answer. b

128. Non hemolytic streptococci is otherwise called

a. alpha hemolytic streptococci

b. beta hemolytic streptococci

c. gamma hemolytic streptococci

d. viridans streptococci

Solution. (c) gamma hemolytic streptococci


Ref: Read the text below
Sol:
Gamma hemolytic streptococci produce no change in the Blood agar
and so nonhemolytic streptococci
Partial hemolysis Alpha
Complete Hemolysis - Beta

Answer. c

129. Identify the Negative sense RNA virus

a. Paramyxovirus

b. Orthomyxovirus

c. Rhabdovirus

http://cbt.damsdelhi.com/damscbtadmin/admin/index.php?pageName=test/export&id=1710001
xxxxxxxxxxxxxxxxxxxxxxxxxxxxxxxxxxxxxxxxxxxxxxxxxxxxxxxxxxxxxxxxxxxxxxxxxxxxxxxxxxxxxx 46/121
15 oct, 2017
10/25/2017 Test Information
d. All of the above

Solution. (d) All of the above


Ref: Read the text below
Sol:
The minus sense RNA virus are paramyxovirus orthomyxovirus and rhabdovirus.
Depending on the method of mRNA transcription single stranded RNA viruses are classified into two categories -positive and negative sense.
In negative sense RNA virus the RNA is antisense with polarity opposite to mRNA.
They possess their own RNA polymerases for mRNA transcription.

Answer. d

130. Which are the special laboratory conditions needed to recover C jejuni?

a. 98.6F aerobic on blood agar plates

b. 98.6F anaerobic on blood agar plates

c. 107.6F microaerophilic on skirrow's medium

d. 107.6F aerobic on skirrow's medium

Solution. (c) 107.6F microaerophilic on skirrow's medium


Ref: Read the text below
Sol:
Campylobacter species are small motile, non spore forming, comma -shaped, gramnegative bacilli, best grown in a microaerophilic environment at
42 C (107.6 F) on skirrow's selective medium

Answer. c

131. Transport medium for Streptococcus pyogenes is

a. Sachs buffered glycerol saline

b. Stuart medium

c. Pikes Medium

d. Cary Blair Medium

Solution. (c) Pikes Medium


Ref: Read the text below
Sol:
Stuart medium is transport medium for Gonococci
Cary Blair Medium is transport medium for Vibrio
Sachs buffered glycerol saline is transport medium for enteric pathogens.

Answer. c

132. The coagulase test shown in the picture detects

a. Free coagulase

b. Bound coagulase

http://cbt.damsdelhi.com/damscbtadmin/admin/index.php?pageName=test/export&id=1710001
xxxxxxxxxxxxxxxxxxxxxxxxxxxxxxxxxxxxxxxxxxxxxxxxxxxxxxxxxxxxxxxxxxxxxxxxxxxxxxxxxxxxxx 47/121
15 oct, 2017
10/25/2017 Test Information

c. Both of the above

d. None of the above

Solution. (b) Bound coagulase


Ref: Read the text below
Sol:
Coagulase test is used to differentiate Staphylococcus aureus (positive) from
Coagulase Negative Staphylococcus (CONS).
Coagulase is an enzyme produced by S. aureus that converts (soluble) fibrinogen to (insoluble) fibrin in plasma.
Staphylococcus aureus produces two forms of coagulase Free & bound.
Tube coagulase test is done to detect free coagulase (Clot reactivating factor - CRF).
Slide coagulase test is done to detect bound coagulase (Clumping factor CF).

Answer. b

133. The most common cause of congenital infections is

a. Cytomegalovirus

b. Herpes simplex virus

c. Parvovirus

d. Rubella virus

Solution. (a) Cytomegalovirus


Ref: Read the text below
Sol:

- Among the choices CMV is the most common cause of congenital infections that can lead to various symptoms in the new-born.

Answer. a

134. An individual is known to have chronic hepatitis disease. Which serological marker should be monitored to determine if the blood contains the
infectious virus?

a. HBcAg

b. HBeAg

c. HBsAg

d. IgG HBs

Solution. (b) HBeAg


Ref: Read the text below
Sol:
Levels of HBeAg are monitored to determine if blood contains infectious HBV.
Chronic hepatitis HBsAg & Anti HBc IgG are positive

Answer. b

135. A 6 month old infant has had watery diarrhoea for 6 days; he vomited a couple of times. The stools have no blood and no pus. He is
dehydrated. What is most likely causing this?

a. Clostridium botulinum toxin

b. Giardia lamblia

c. Rotavirus

d. Staphylococcus aureus enterotoxin

Solution. (c) Rotavirus


Ref: Read the text below
Sol:
The lack of flaccid paralysis leaves out botulinum toxin, and staphylococcal toxin is also unlikely as it is a short, self-resolving illness.
The most likely is rotavirus due to his age and the length of the disease.

Answer. c

http://cbt.damsdelhi.com/damscbtadmin/admin/index.php?pageName=test/export&id=1710001
xxxxxxxxxxxxxxxxxxxxxxxxxxxxxxxxxxxxxxxxxxxxxxxxxxxxxxxxxxxxxxxxxxxxxxxxxxxxxxxxxxxxxx 48/121
15 oct, 2017
10/25/2017 Test Information
136. Yersinia pestis may be transferred by

a. Dermacentor tick bite

b. Human body louse bite

c. Ixodes tick bite

d. Respiratory droplets

Solution. (d) Respiratory droplets


Ref: Read the text below
Sol:

The common mode of transmission is bite of Rat flea (X. cheopsis) that choice not given here
The other route of transmission is through respiratory droplets from patients who have Pulmonic plague.

Answer. d

137. A 36-year-old male from Bihar has a cough that has been bothering him for several weeks. He has also lost 12 Kg weight. Sputum acid fast
stain shown in the picture. Which of the following factors is known to be most important in triggering the granulomatous reaction to wall off and
contain the infection?

a. Cord factor

b. Mycolic acid

c. Purified protein derivative (PPD)

d. Sulfatides

Solution. (a) Cord factor

Ref:Read the text below

Sol:

Cord factor helps trigger the Th1 response, which helps contain the infection.

Answer. a

138. Bacterial antibiotic resistance is frequently conveyed by

a. Temperate bacteriophage

b. R-factor plasmid

c. Replicon

d. Lytic bacteriophage

http://cbt.damsdelhi.com/damscbtadmin/admin/index.php?pageName=test/export&id=1710001
xxxxxxxxxxxxxxxxxxxxxxxxxxxxxxxxxxxxxxxxxxxxxxxxxxxxxxxxxxxxxxxxxxxxxxxxxxxxxxxxxxxxxx 49/121
15 oct, 2017
10/25/2017 Test Information
Solution. (b) R-factor plasmid

Ref:Read the text below

Sol:

R-factor (resistance) plasmidscontain genes for proteins that degrade antibiotics or alter antibiotic transport, thus conferring antibiotic
resistance.

They also carry transfer genes, which facilitate their intercellular transfer to other genomes.

Answer. b

139. Medium shown in the picture (Baird Parker Agar) used to detect the following pathogen

a. Corynebacterium diphtheriae

b. Legionella pneumophila

c. Cryptococcus neoformans

d. Staphylococcus aureus

Solution. (d) Staphylococcus aureus

Ref:Read the text below

Sol:

Baird Parker Agar mediumis formulated on the principle that staphylococci are able to reduce tellurite to tellurium.

Lithium chloride and potassium telluriteacts as inhibitor agent for contaminating microflora.

It appears as black colour colonies.

Answer. d

140. Find the False statement regarding smallpox

a. It has no animal reservoir

b. It has no carriers

c. The source of infection- patient in late phase of the disease

d. Produces synchronous rash

Solution. (c) The source of infection- patient in late phase of the disease

Ref:Read the text below

Sol:

The source of infection was a patient in the early phase of the disease though infectivity extended from the appearance of buccal mucosal
lesions to the disappearance of all the skin lesions.

It produces synchronous rash.

Small pox was an exclusively human infection with no animal reservoir.

There were no carriers as the virus was eliminated from the patient on recovery.

http://cbt.damsdelhi.com/damscbtadmin/admin/index.php?pageName=test/export&id=1710001
xxxxxxxxxxxxxxxxxxxxxxxxxxxxxxxxxxxxxxxxxxxxxxxxxxxxxxxxxxxxxxxxxxxxxxxxxxxxxxxxxxxxxx 50/121
15 oct, 2017
10/25/2017 Test Information
Answer. c

141. Thinnest part of the lens :

a. 2microns

b. 3microns

c. 4microns

d. 5microns

Solution. (c) 4microns


Ref: Read the text below
Sol:
Thinnest part of lens is the capsule at the posterior pole with thickness of 4microns

Answer. c

142. Inverse glaucoma is :

a. Pseudoexfoliation syndrome

b. Malignant glaucoma

c. Neovascular glaucoma

d. Iridocorneal syndrome

Solution. (b) Malignant glaucoma


Ref: Read the text below
Sol:
Inverse glaucoma is glaucoma treated by a mydriatic .
Examples of Inverse glaucoma are malignant glaucoma and Spherophakia

Answer. b

143. One and a half syndrome is :

a. MLF Lesion

b. PPRF Lesion

c. Both

d. None

Solution. (c) Both


Ref: Read the text below
Sol:
It is Lesion of both MLF and PPRF of the same side so that there is defective adduction on one side and on the other side both adduction and
abduction are defective .

Answer. c

144. First symptom of sympathetic ophthalmitis :

a. Photophobia

b. Blepharospasm

c. Decreased vision

d. Decreased accommodation

Solution. (d) Decreased accommodation


Ref: Read the text below
Sol:

- Since the inflammation starts in the retrolental area first ,so the first symptom is difficulty in near vision due to involvement of ciliary muscles .

http://cbt.damsdelhi.com/damscbtadmin/admin/index.php?pageName=test/export&id=1710001
xxxxxxxxxxxxxxxxxxxxxxxxxxxxxxxxxxxxxxxxxxxxxxxxxxxxxxxxxxxxxxxxxxxxxxxxxxxxxxxxxxxxxx 51/121
15 oct, 2017
10/25/2017 Test Information
Answer. d

145. Number of endothelial cells in corneal decompensation

a. "< 700"

b. "< 400"

c. "< 500"

d. "< 600"

Solution. (c) <500


Ref: Read the text below
Sol:
When the number of endothelial cells are less than 500 then compensation is not possible and it leads to corneal edema .

Answer. c

146. Angular conjunctivitis occurs due to

a. Moraxella axenfeld

b. Moraxella catarrhalis

c. Moraxella lacunata

d. All of the above

Solution. (d) All of the above


Ref: Read the text below
Sol:
Other cause is staphylococcus aureus .

Answer. d

147. Maximum goblet cells are seen in

a. Inferonasal conjunctiva

b. Superonasal conjunctiva

c. Inferotemporal conjunctiva

d. Superotemporal conjunctiva

Solution. (a) Inferonasal conjunctiva


Ref: Read the text below
Sol:
Goblet cells form the mucin layer and its maximum density is on the inferonasal conjunctiva

Answer. a

148. Size of pinhole is

a. 1mm

b. 2mm

c. 3mm

d. 4mm

Solution. (a) 1mm


Ref: Read the text below
Sol:
Pin hole neutralises 3D .

Answer. a

http://cbt.damsdelhi.com/damscbtadmin/admin/index.php?pageName=test/export&id=1710001
xxxxxxxxxxxxxxxxxxxxxxxxxxxxxxxxxxxxxxxxxxxxxxxxxxxxxxxxxxxxxxxxxxxxxxxxxxxxxxxxxxxxxx 52/121
15 oct, 2017
10/25/2017 Test Information
149. Crossed diplopia is a feature of

a. LR Palsy

b. MR palsy

c. SR palsy

d. IR palsy

Solution. (b) MR palsy


Ref: Read the text below
Sol:
MR palsy causes divergent squint , in which we get crossed diplopia .
In convergent squint we get uncrossed diplopia .

Answer. b

150. Thinnest part of the retina is :

a. 0.1mm

b. 0.2mm

c. 0.3mm

d. 0.4mm

Solution. (a) 0.1mm


Ref: Read the text below
Sol:
Retina is thinnest at Ora Serrata with the thickness of 0.1mm

Answer. a

151. Identify the muscle being tested in the illustration shown here.

a. Subscapularis

b. Suprascapularis

c. Infraspinatus

d. Supraspinatus

Solution. A
Subscapularis
The Lift Off Test (also knows as Gerbers Test) is commonly used in orthopedic
examinations to test for a tear in subscapularis tendon or subscapularis tendonitis. It can also show scapular instability.
In this position, the patient attempts to move the hand away from the lower back by
extending and further internally rotating the arm. The examiner can also provide
resistance to this movement if the patient is able to complete the movement.

Answer. a

152. Which of the following nerve injury shows the worst prognosis ?
http://cbt.damsdelhi.com/damscbtadmin/admin/index.php?pageName=test/export&id=1710001
xxxxxxxxxxxxxxxxxxxxxxxxxxxxxxxxxxxxxxxxxxxxxxxxxxxxxxxxxxxxxxxxxxxxxxxxxxxxxxxxxxxxxx 53/121
15 oct, 2017
10/25/2017 Test Information

a. Radial Nerve

b. Ulnar Nerve

c. Axillary Nerve

d. Median Nerve

Solution. B
Ulnar Nerve
Best Prognosis : Radial Nerve
Worst Prognosis : Ulnar Nerve
Worst Prognosis despite Surgical Repair : Sciatic Nerve

Answer. b

153. Which of the following is the most common direction of epiphyseal slip in SCFE ?

a. Posteromedial

b. Posterolateral

c. Anteromedial

d. No slip

Solution. D
No Slip
Actually due to Anterolateral disruption of Physis , There is an apparent Posteromedial
Slip. But in true sense , There is no slip.

Answer. d

154. Which of the following is the most common Giant Cell Variant ?

a. Chondroblastoma

b. Aneursymal Bone Cyst

c. Fibrous Dysplasia

d. Non Ossifying Fibroma

Solution. D
Non Ossifying Fibroma
Most Common GC Variant : Non Ossifying Fibroma
Closest GC Variant : ABC

Answer. d

155. Which of the following is associated with maximum shortening ?

a. Posterior Hip Dislocation

b. Fracture Intracapsular Femur

c. Fracture Extracapsular Femur

d. Fracture Shaft of Femur

Solution. A
Posterior Hip Dislocation
Maximum Shortening : Posterior Hip Dislocation
Least Shortening :Fracture Neck Femur

Answer. a

156. Identify the implant

http://cbt.damsdelhi.com/damscbtadmin/admin/index.php?pageName=test/export&id=1710001
xxxxxxxxxxxxxxxxxxxxxxxxxxxxxxxxxxxxxxxxxxxxxxxxxxxxxxxxxxxxxxxxxxxxxxxxxxxxxxxxxxxxxx 54/121
15 oct, 2017
10/25/2017 Test Information

a. Denham's Pin

b. Steinmann's Pin

c. Rush Pin

d. Ender's Pin

Solution. A
Denham's Pin
The central grooves in this image suggest Denham's Pin as these grooves help in
penetrating in cancellous bones like Calcaneum

Answer. a

157. A 25 year old soccer player complained of injury to right knee 6 months ago. Now he says that there is a feeling of sudden giving away of knee joint
with knee in 90 degree flexion which completely disappears when the knee is taken into complete extension. Which of the following ligament needs to be
checked ?

a. Anteromedial bundle of ACL

b. Anterolateral bundle of ACL

c. Anterolateral bundle of PCL

d. Anteromedial bundle of PCL

Solution. A
Anteromedial bundle of ACL
Anteromedial Bundle ACL - KNEE 90 degrees flexion
Posterolateral Bundle ACL - KNEE COMPLETE EXTENSION

Answer. a

158. Most Common Cause of Failure of Conservative treatment of Manipulation and Casting in CTEV is :

a. Inadequate Application of pressure on fulcrum

b. Overcorrection of deformity

c. Early application of POP Cast

d. Non Compliance with Bracing and Shoes

Solution. D
Non Compliance with Bracing and Shoes is the Most Common Cause of Failure of
Conservative treatment of Manipulation and Casting in CTEV

Answer. d

159. Preiser Disease involves :

a. Scaphoid

b. Lunate

http://cbt.damsdelhi.com/damscbtadmin/admin/index.php?pageName=test/export&id=1710001
xxxxxxxxxxxxxxxxxxxxxxxxxxxxxxxxxxxxxxxxxxxxxxxxxxxxxxxxxxxxxxxxxxxxxxxxxxxxxxxxxxxxxx 55/121
15 oct, 2017
10/25/2017 Test Information
c. Talus

d. Navicular

Solution. A
Panner's disease : Capitellum
Kohler's disease : Navicular
Kienbock's disease : Lunate
Preiser disease : Scaphoid

Answer. a

160. Identify the angle shown in the illustration above :

a. Cobb's Angle

b. Baumann's Angle

c. Gissane's Angle

d. Southwick's Angle

Solution. B
Baumann's Angle is the angle between a line along the shaft humerus and the other
along the superior capitellar physis.

Answer. b

161. Smiles at mirror image milestone is achieved at

a. 3 months

b. 4 months

c. 5 months

d. 6 months

Solution. (D) 6 months


Ref.: Read the text below
Sol :
DEVELOPMENT MILESTONES (QUICK REVIEW):
Gross Motor:
3 months Head holding
5 months Sitting with support
6 months Prone to supine, supine to prone
8 months Sitting without support
9 months Crawling
10 months Creeping
1 year Walking few steps independently
2 yr walk up stairs with two feet at each step
3 yr Up stairs with one foot at each step, rides tricycle
4 yrs Hops on one foot
5 yr Skips on two feet
Fine motor

12 weeks Grasp reflex disappears


4 months goes for object

http://cbt.damsdelhi.com/damscbtadmin/admin/index.php?pageName=test/export&id=1710001
xxxxxxxxxxxxxxxxxxxxxxxxxxxxxxxxxxxxxxxxxxxxxxxxxxxxxxxxxxxxxxxxxxxxxxxxxxxxxxxxxxxxxx 56/121
15 oct, 2017
10/25/2017 Test Information
5 months Bidextrous grasp
7 months Transfer object, palmar grasp
9 months Pincer grasp
13 months Casting
15 months Self feed with a spoon
18 months Self feed with a cup
tower of 4 cubes
24 months turns pages of a book one at time
6-7 cubes
3 yrs makes circle, tower of 10 cubes
4 yrs Square, + sign
5 yrs Triangle x sign
Social
2 months Social smile
3 months recognizes mother
6-7 months smiles at mirror image
9 months waves bye-bye
Stranger anxiety
15 months hugs parents

18 months kisses parents


2 yrs Dry by day
3 yrs dry by night
Dress/undress himself
Language
1 month head turns to sound
3 months cooing
6 months mono syllabus (ma,ba)
9 months Bisyllabus (ma-ma, baba)
1 yr 2 words with meaning
18 months vocabulary of ten words
2 yrs simple sentences with 2 words
3 yrs tells age and sex, uses pronouns,handedness
4 yrs tells story
5 yrs knows colors

Answer. d

162. CFTR gene is on

a. 7p

b. 7q

c. 17p

d. 17q

Solution. (b) 7q

Ref.:Read the text below

Sol :

CF is the major cause of severe chronic lung disease in children and is responsible for most exocrine pancreatic insufficiency in early life.

It is also responsible for many cases of salt depletion, nasal polyposis, pansinusitis, rectal prolapse, pancreatitis, cholelithiasis, and insulin-
dependent hyperglycemia.

CF occurs most frequently in white populations of northern Europe, North America, and Australia/New Zealand. The prevalence varies by report
but, in general, approximates 1/3,500 live births.

CF is inherited as an autosomal recessive trait.

The CF gene codes for a protein of 1,480 amino acids called the CF transmembrane regulator (CFTR). CFTR is expressed largely in epithelial cells
of airways, the gastrointestinal tract (including the pancreas and biliary system), the sweat glands, and the genitourinary system.

CFTR has ion channel and regulatory functions that are perturbed variably by the different mutations.

More than 1,500 CFTR polymorphisms are associated with the CF syndrome.

The most prevalent mutation of CFTR is the deletion of a single phenylalanine residue at amino acid 508 (F508). This mutation is responsible for
the high incidence of CF in northern European populations and is considerably less frequent in other populations, such as those of southern Europe and
Israel.

Approximately 50% of individuals with CF who are of northern European ancestry are homozygous for F508, and >70% carry at least one F508
gene.

http://cbt.damsdelhi.com/damscbtadmin/admin/index.php?pageName=test/export&id=1710001
xxxxxxxxxxxxxxxxxxxxxxxxxxxxxxxxxxxxxxxxxxxxxxxxxxxxxxxxxxxxxxxxxxxxxxxxxxxxxxxxxxxxxx 57/121
15 oct, 2017
10/25/2017 Test Information
The membranes of CF epithelial cells are unable to secrete chloride ions in response to cyclic adenosine monophosphate (cAMP)mediated signals
and, at least in the respiratory tract, excessive amounts of sodium are absorbed through these membranes

Another puzzle is the propensity for P. aeruginosa to undergo mucoid transformation in the CF airways. The complex polysaccharide produced by
these organisms generates a biofilm that provides a hypoxic environment and thereby protects Pseudomonas against antimicrobial agents.

Answer. b

163. Gain of weight in a preschool normal child is?

a. 2-2.5 kg per year

b. 3-3.5 kg per year

c. 4-4.5 kg per year

d. 5-5.5 kg. per year

Solution. (a) 2-2.5 kg

Ref.:Read the text below

Sol :

- A gain of approximately 2 kg (4-5 ib) in weight and 7-8 cm (2-3 in) in height per year is seen during pre school years.

Answer. a

164. Failure to pass meconium with 48 hrs of birth in a newborn with no obvious external abnormality should lead to the suspicion of

a. Anal atresia

b. Congenital pouch colon

c. Congenital aganglinosis

d. Meconium ileus.

Solution. (c) Congenital aganglinosis

Ref.:Read the text below

Sol :

Hirschsprung disease (HD) is a motor disorder of the gut, which is caused by the failure of neural crest cells (precursors of enteric ganglion cells) to
migrate completely during intestinal development.

At least eight genetic mutations have been identified in patients with HD

The predominant gene affected is the RET proto-oncogene. RET malfunction accounts for at least 50 percent of familial and 20 percent of sporadic
cases, and is especially seen in patients with long segment disease

Answer. c

165. The commonest cause of death in diphtheric child is ?

a. IIIrd nerve palsy

b. Tonsillitis

c. Myocarditis

d. Septicemia

Solution. (c) Myocarditis

Ref.:Read the text below

Sol :

The most common cause of death in diphtheria is either due to mechanical obstruction of airways in bull neck or due to myocarditis.

Answer. c

http://cbt.damsdelhi.com/damscbtadmin/admin/index.php?pageName=test/export&id=1710001
xxxxxxxxxxxxxxxxxxxxxxxxxxxxxxxxxxxxxxxxxxxxxxxxxxxxxxxxxxxxxxxxxxxxxxxxxxxxxxxxxxxxxx 58/121
15 oct, 2017
10/25/2017 Test Information

166. All of the following are characteristics features of tricuspid atresia except

a. Left axis deviation

b. right ventricular hypoplasia

c. Pulmonary vascularity is diminished

d. Splitting of S2

Solution. (d) Splitting of S2

Ref.:Read the text below

Sol :

Tricuspid Atresia

- In tricuspid atresia, no outlet from the right atrium to the right ventricle is present. The entire systemic venous return enters the left side of the heart by
means of the heart by means of foramen ovale or an associated ASD. Left ventricle blood usually flows into the right ventricle via a VSD.

- If ventricular septum is intact, the right ventricle is completely hypoplastic and pulmonary atresia is present. In such conditions pulmonary blood flow
is totally depends on PDA.

- Left axis deviation and left ventricular hypertrophy are noted on ECG.

Clinical Manifestation :

- Cyanosis is usually evident at birth.

- Holosystolic murmur audible along left sterna border.

- Hypoxic episodes occurs as a result of compromised pulmonary blood flow.

- SI is normal and S2 is single. (No splitting of S2)

- X-ray shows pulmonary under circulation

Answer. d

167. Earliest valvular lesionin a case of acute rheumatic fever

a. MR

b. AR

c. MS

d. AS

Solution. (a) MR

Ref.:Read the text below

Sol :

The valves most affected by rheumatic fever, in order, are the mitral, aortic, tricuspid, and pulmonary valves.

In most cases, the mitral valve is involved with 1 or more of the other 3.

In acute disease, small thrombi form along the lines of valve closure.

In chronic disease, there is thickening and fibrosis of the valve resulting in stenosis, or less commonly, regurgitation.

Answer. a

168. What is the preferred drug for Infantile spasms in children

a. ACTH

b. Phenytoin

c. Valproate

http://cbt.damsdelhi.com/damscbtadmin/admin/index.php?pageName=test/export&id=1710001
xxxxxxxxxxxxxxxxxxxxxxxxxxxxxxxxxxxxxxxxxxxxxxxxxxxxxxxxxxxxxxxxxxxxxxxxxxxxxxxxxxxxxx 59/121
15 oct, 2017
10/25/2017 Test Information

d. Vigabatrin

Solution. (a) ACTH

Ref.:Read the text below

Sol :

Infantile spasms usually begin between the ages of 4 and 8 mo and are characterized by brief symmetric contractions of the neck, trunk, and
extremities. There are at least three types of infantile spasms: flexor, extensor, and mixed.

The EEG that is most commonly associated with infantile spasms is referred to as hypsarrhythmia, which consists of a chaotic pattern of high-
voltage, bilaterally asynchronous, slow-wave activity or a modified hypsarrhythmia pattern.

Infantile spasms are typically classified into two groups:cryptogenic and symptomatic.

Prenatal and perinatal factors include hypoxic-ischemic encephalopathy with periventricular leukomalacia, congenital infections, inborn errors of
metabolism, neurocutaneous syndromes such as tuberous sclerosis, cytoarchitectural abnormalities including lissencephaly and schizencephaly, and
prematurity. Postnatal conditions include CNS infections, head trauma (especially subdural hematoma and intraventricular hemorrhage), and
hypoxic-ischemic encephalopathy.

Infants with cryptogenic infantile spasms have a good prognosis, whereas those with the symptomatic type have an 8090% risk of mental
retardation.

One hypothesis implicates corticotropin-releasing hormone (CRH), a putative neurotransmitter, metabolized in the inferior olive.

Exogenous ACTH and glucocorticoids suppress CRH synthesis, which may account for their effectiveness in treating infantile spasms. In Europe
and Canada, infantile spasms associated with TS are treated with vigabatrin (rather than adrenocorticotropic hormone) with good results.
Vigabatrin is not available in the United States.

Answer. a

169. Which of the following is not a common organism responsible for neonatal sepsis in India?

a. E. coli

b. Staph aureus

c. Gp B Streptococcus

d. Klebsiella

Solution. (c) Gp B Streptococcus

Ref.:AIIMS NICU Protocols(http://www.newbornwhocc.org/pdf/sepsis_innewborn.pdf)

Sol:

Epidemiology of sepsis in India:

The incidence of neonatal sepsis according to the data from National Neonatal Perinatal Database (NNPD, 2002-03) is 30 per 1000 live births.

Among intramural births, Klebsiella pneumoniae was the most frequently isolated pathogen (32.5%), followed by Staphylococcus aureus (13.6%).

Among extramural neonates (referred from community/other hospitals), Klebsiella pneumoniae was again the commonest organism (27%), followed
by Staphylococcus aureus (15%) and Pseudomonas (13%).

Answer. c

170. A 2-day old neonate in the neonatal ICU develops seizures. Which of the following would be the next best investigation for the child?

a. Transcranial ultrasound

b. Skull X-ray

c. CT scan

d. MRI

Solution. (a) Transcranial ultrasound

Ref.:Rumacks Read the text below

Sol:

Seizures in neonates are relatively common, with variable clinical manifestations.

http://cbt.damsdelhi.com/damscbtadmin/admin/index.php?pageName=test/export&id=1710001
xxxxxxxxxxxxxxxxxxxxxxxxxxxxxxxxxxxxxxxxxxxxxxxxxxxxxxxxxxxxxxxxxxxxxxxxxxxxxxxxxxxxxx 60/121
15 oct, 2017
10/25/2017 Test Information
Their presence is often the first sign of neurologic dysfunction, and they are powerful predictors of long-term cognitive and developmental
impairment.

Most seizures in the neonate are focal, although generalized seizures have been described in rare instances.

Subtle seizures are more common in full-term than in premature infants.

Video electroencephalogram (EEG) studies have demonstrated that most subtle seizures are not associated with electrographic seizures.

Examples of subtle seizures include chewing, pedaling, or ocular movements.

Seizures occur when a large group of neurons undergo excessive, synchronized depolarization.

Depolarization can result from excessive excitatory amino acid release (eg, glutamate) or deficient inhibitory neurotransmitter (eg, gamma amino
butyric acid [GABA]).

Preterm neonates <32 gestational weeks and/or <1500 g are highly susceptible for HIE and/or IVH.

Thus, special medical care, including post-labor hospitalization in well equipped special baby care units (SCBU) and routine transcranial
ultrasound (TCUS) screening is recommended for those preterm neonates.

Answer. a

171. In an Autosomal recessive disorder, one parent is normal and the other is carrier and the child is also affected. What is the reason?

a. Germline Mosaicism

b. Genomic imprinting

c. Penetration

d. Uniparental disomy

Solution. (d) Uniparental disomy

Ref.:Read the text below

http://cbt.damsdelhi.com/damscbtadmin/admin/index.php?pageName=test/export&id=1710001
xxxxxxxxxxxxxxxxxxxxxxxxxxxxxxxxxxxxxxxxxxxxxxxxxxxxxxxxxxxxxxxxxxxxxxxxxxxxxxxxxxxxxx 61/121
15 oct, 2017
10/25/2017 Test Information
Sol :

- Uniparental disomy(UPD) occurs when a person receives two copies of a chromosome, or of part of a chromosome, from one parent and no copies
from the other parent.

- UPD can be the result of heterodisomy, in which a pair of non-identical chromosomes are inherited from one parent (an earlier stage meiosis I error) or
isodisomy, in which a single chromosome from one parent is duplicated (a later stage meiosis II error).

- Because it may lead to the duplication of lethal recessive genes, isodisomy is potentially dangerous, while heterodisomy is essentially benign.

Answer. d

172. After 5 days of birth, baby developed poor feeding, convulsions, fever with low protein, low sugar and high chloride (in CSF) is most likely due
to

a. Listeria monocytogenes

b. Mycoplasma pneumoniae

c. T.B.

d. Leptospira

Solution. (a) Listeria monocytogenes

Ref.:Read the text below

Sol :

Most common cause of Neonatal meningitis

E. Coli

Streptococci group b

Listeria monocytogens

Out of the given options only L. Monocytogenes can cause menginitis in 5 days old baby

There is no meningitis which lowers the CSF protein.

Answer. a

173. A child presents with antimongoloid shant, pulmonary stenosis, short stature and undescended testis. The likely diagnosis is :

a. Hypoparathyroidism

b. Noonan syndrome

c. Klinefelter syndrome

d. XYY sex chromosome

Solution. (b) Noonan syndrome

Ref.:Read the text below

Sol :
Noonan syndrome(NS) is a relatively common autosomaldominantcongenital disorderthat affects both males and females equally[1]:550.

It used to be referred to as the male version of Turner's syndrome[2](and is still sometimes described in this way);however,the genetic causes of
Noonan syndrome and Turner syndrome are distinct.

The principal features include congenital heart defect(typically pulmonary valve stenosis) also ASD, hypertrophic cardiomyopathy, short
stature, learning problems, pectus excavatum, impaired blood clotting, and a characteristic configuration of facial features including a webbed neck
and a flat nose bridge. The syndromeis named after Dr. Jacqueline Noonan.

It is a RASopathy, as the syndrome is in the family of RAS-MAPK pathway disorders.

Answer. b

174. The earliest indicator of response after starting iron in a 6 year old girl with iron deficiency

http://cbt.damsdelhi.com/damscbtadmin/admin/index.php?pageName=test/export&id=1710001
xxxxxxxxxxxxxxxxxxxxxxxxxxxxxxxxxxxxxxxxxxxxxxxxxxxxxxxxxxxxxxxxxxxxxxxxxxxxxxxxxxxxxx 62/121
15 oct, 2017
10/25/2017 Test Information

a. Increased reticulocyte count

b. Increased hemoglobin

c. Increased ferritin

d. Increased serum iron

Solution. (a) Increased reticulocyte count

Ref: Read the text below

Sol :

Response to iron therapy :

o With iron therapy, the activity of iron containing enzyme in the cell improve.

o The child become less irritable and his appetite improve within 24 hrs.

o Initial bone marrow response is observed within 48 hrs.

o Rise in reticulocyte count occurs by second to third day.

o This is followed by elevation of hemoglobin level

o Body iron stores are repleted after correction of the hemoglobin level.

o Out of the given options, reticulocyte count increased first by second to third day.

Answer. a

175. Late onset hemorrhagic disease of newborn is characterized by all of the following features except

a. Usually occurs in cow milk fed babies

b. Onset occurs at 4 12 week of age

c. Intracranial hemorrhage can occur

d. Intramuscular vitamin K prophylaxis at birth has a protective role.

Solution. (a) Usually occurs in cow milk fed babies

Ref: Read the text below

Sol :

Hemorrhagic diseaseof newborn is caused due to deficiency of vit.K.

Breast milk is a poor source of vitamin k, and hemorrhagic complications have appeared more frequently in breast fed than in formula fed
infants.

Answer. a

176. A 3-year old boy is brought to the casualty by his mother with progressive shortness of breath for 1 day. The child has history to bronchial
asthma. On examination, the child is blue, gasping and unresponsive, what will you like to do first

a. Intubate

b. Administer 100% O2 by mask

c. Ventilate with bag and mask

d. Administer nebulised salbutamol

Solution. (C) Ventilate with bag and mask

Ref: Read the text below

Sol :

Child has H/O bronchial asthma. Child is blue, gasping and unresponsive suggest, that he is suffering from life threatening asthma.

Other features of life threatening asthma are -

http://cbt.damsdelhi.com/damscbtadmin/admin/index.php?pageName=test/export&id=1710001
xxxxxxxxxxxxxxxxxxxxxxxxxxxxxxxxxxxxxxxxxxxxxxxxxxxxxxxxxxxxxxxxxxxxxxxxxxxxxxxxxxxxxx 63/121
15 oct, 2017
10/25/2017 Test Information
o Cyanosis

o Silent chest

o Poor respiratory efforts.

o Exhaustion and fatigue

o Altered sensorium

o Peak expiratory flow rate < 30% of predicted

o O2 saturation < 90%

Management of life threatening asthma :

o Immediately start on O2 inhalation.

o Subcutaneous injection of terbutaline or adrenaline.

o Inhalation of salbutamol or terbutaline, and ipratropium.

o IV hydrocortisone 10 mg/kg and transfer the patient in ICU.

Answer. c

177. True regarding breath holding spells is all except

a. Antiepileptic treatment is necessary

b. Atropine is sometimes used

c. Attacks of cyanosis can occur

d. Occurs between 6 months to 5 years.

Solution. (a) Antiepileptic treatment is necessary

Ref: Read the text below

Sol :

Answer. a

178. All of the following are considered criteria used to diagnose attention deficit hyperactivity disorder except

a. Inattention

b. Hyperactivity

c. Long-term memory loss

d. Clinical impairment in social settings

Solution. (c) Long-term memory loss

Ref.:Read the text below

Sol :

http://cbt.damsdelhi.com/damscbtadmin/admin/index.php?pageName=test/export&id=1710001
xxxxxxxxxxxxxxxxxxxxxxxxxxxxxxxxxxxxxxxxxxxxxxxxxxxxxxxxxxxxxxxxxxxxxxxxxxxxxxxxxxxxxx 64/121
15 oct, 2017
10/25/2017 Test Information
According to the DSM-IV, the core clinical features of this disorder include impulsivity, distractibility, an inability to sustain attention and/or
concentration, and developmentally inappropriate activity levels.

Impairment from symptoms must be present in two or more settings.

Long-term memory loss has not been associated with ADHD.

Answer. c

179. Webbing of neck, increased carrying angle, low posterior hair line and short fourth metacarpal are characteristics of :

a. Klinefelter syndrome

b. Turner syndrome

c. Cri du chat syndrome

d. Noonan syndrome

Solution. (b) Turner syndrome

Ref: Read the text below

Sol :

Answer. b

180. Increased nuchal fold thickness is a feature of :

a. Paul Bunnel syndrome

b. De-pan syndrome

c. Downs syndrome

d. Cri-du chat syndrome

Solution. (c) Downs syndrome

Ref: Read the text below

Sol :

Fetal ultrasonographymay reveal increase nuchal fold thickness in case of Down syndrome.

Answer. c

181.
All of the following are features of haemolytic anemia except

http://cbt.damsdelhi.com/damscbtadmin/admin/index.php?pageName=test/export&id=1710001
xxxxxxxxxxxxxxxxxxxxxxxxxxxxxxxxxxxxxxxxxxxxxxxxxxxxxxxxxxxxxxxxxxxxxxxxxxxxxxxxxxxxxx 65/121
15 oct, 2017
10/25/2017 Test Information

a. Hemoglobinuria

b. Jaundice

c. Increased haptoglobulin levels

d. Hemosiderinuria

Solution. C

Haptoglobulin levels are decreased when there is haemolytic anemia. When daily turnover of Hb exceeds twice the normal rate, haptoglobulins tend to get
depleted. Low concentrations of haptoglobulins in absence of increased hemolysis may be found in hepatocellular diseaseIncreased. haptoglobulin levels
may be found in pregnancy, chronic infections, malignancy, tissue damage, Hodgkin disease, rheumatoid arthritis, SLE and biliary obstruction and as a
consequence of steroid therapy or the use of OCPs. Under these circumstances normal haptoglobulin does not exclude hemolysis.

Answer. c

182. Philadephia chromosome seen in chronic myeloid leukemia produces a protein called

a. P170

b. P210

c. P190

d. P230

Solution. B

Due to BCR ABL1 fusion in CML, the new protein formed is 210 kD or p210. P190 is seen in ALL.

Answer. b

183. A 62 year old woman presented with headache, chest pain and weak pulses in the upper limb. A biopsy from aortic arch showed granulomatous
inflammation with fragmentation of internal elastic lamina and narrowed lumen. The diagnosis is

a. Takayasu arteritis

b. Giant cell aortitis

c. Polyarteritisnodosa

d. Kawasaki disease

Solution. B

The histological findings and site is consistent with that of large vessel vasculitis and granulomatous inflammation. Since the age of the patient is more
than 50 years, the diagnosis will be Giant cell aortitis.

Answer. b

184. MacCallum plaques are seen in

a. Aorta

b. Right atrium

c. Coronary artery

d. Left atrium

Solution. D

MacCallum plaques are seen in left atrium of a heart damaged by rheumatic heart disease due to valvular dysfunction leading to regurgitation jets.

Answer. d

185. Loss of which of the following protein is implicated in aiding metastasis of tumor cells?

http://cbt.damsdelhi.com/damscbtadmin/admin/index.php?pageName=test/export&id=1710001
xxxxxxxxxxxxxxxxxxxxxxxxxxxxxxxxxxxxxxxxxxxxxxxxxxxxxxxxxxxxxxxxxxxxxxxxxxxxxxxxxxxxxx 66/121
15 oct, 2017
10/25/2017 Test Information
a. Integrins

b. Cadherins

c. Matrix metalloproteases

d. Cathepsin

Solution. B

Integrins are components of ECM which help in binding of cells to ECM. Although implicated in metastasis, E and N cadherins (part of desmosomes) are
more recognised in the mechanism of invasion of metastasis, the former, when lost, playing a role in epithelial-mesenchymal transition. Matrix
metalloproteases and cathepsin help breakdown stroma and aid invasion. Loss of E cadherin is also characteristically noted in Lobular carcinoma breast
and diffuse gastric carcinoma.

Answer. b

186. Ductal carcinoma in situ can be differentiated from invasive ductal carcinoma by all of the following except

a. Nuclear pleomorphism

b. Basement membrane invasion

c. Loss of myoepithelial cells

d. Central areas of necrosis

Solution. A

DCIS shows nuclear pleomorphism just like an invasive carcinoma. But the presence of basement membrane invasion, loss of myoepithelial cells favour
IDC whereas central areas of necrosis are more characteristic of DCIS (called comedo type of DCIS). Hence, nuclear pleomorphism does not help to
differentiate between the two entities.

Answer. a

187. All of the following points are true about Lynch syndrome except

a. Left colon is involved more

b. Endometrial cancer is the most common extra colonic tumor involved

c. Intra tumoral lymphocytes are more

d. Seen in younger age group

Solution. A

Warthin Lynch syndrome or Hereditary non polyposis coli cancer is the most common familial cause of colorectal carcinoma. Patients affected are
younger and usually involve the right colon. Mucinous, signet ring cell features are more common and intra tumoral lymphocytes are increased. Increased
incidence of other cancers like endometrial (most common), gastric, ovarian and transitional cell is also noted.

Answer. a

188. Which of the following is the most reliable feature of malignancy in a paraganglioma?

a. Presence of metastasis

b. Nuclear pleomorphism

c. Vascular or capsular invasion

d. Increased mitotic activity

Solution. A

Unfortunately, it is almost impossible to predict the clinical course of a carotid body tumormitoses, pleomorphism, and even vascular invasion are not
reliable indicators. Hence, presence of metastasis is the only reliable indicator of malignancy.

Answer. a

189. Which of the following is not true regarding genetics of thyroid tumors

http://cbt.damsdelhi.com/damscbtadmin/admin/index.php?pageName=test/export&id=1710001
xxxxxxxxxxxxxxxxxxxxxxxxxxxxxxxxxxxxxxxxxxxxxxxxxxxxxxxxxxxxxxxxxxxxxxxxxxxxxxxxxxxxxx 67/121
15 oct, 2017
10/25/2017 Test Information

a. Driver mutations in KRAS are seen in follicular carcinoma

b. BRAF mutations are the most common mutations seen in papillary carcinoma

c. RET rearrangements are seen in medullary carcinoma

d. PI3K mutations are seen in anaplastic carcinoma

Solution. C

RET mutations are seen in medullary carcinoma and RET/PTC rearrangements are seen in papillary carcinoma.

Answer. c

190. Which of the following is not true for megaloblasticanemia?

a. Giant metamyelocytes are seen

b. Bone marrow examination is important

c. Reticulocytosis is present

d. Unconjugated hyperbilirubinemia

Solution. C

Megaloblasticanemia shows reticulocytopenia, not reticulocytosis.

Answer. c

191. Supravital staining helps to visualise all except

a. Reticulocytes

b. HbH inclusions

c. Howell Jolly bodies

d. Heinz bodies

Solution. C

Howell Jolly bodies are recognised on routine peripheral smearing, no special stain is needed.

Answer. c

192. Reduced beta globin chain production in thalassemia occurs due to which of the following genetic change?

a. Nonsense mutations

b. Gene deletions

c. Splicing mutations

d. DNA methylation

Solution. C

Most common cause of reduced beta chain production in thalassemia is splicing mutations and most common cause of absent beta chain production in
thalassemia is nonsense mutations. Gene deletions are seen most commonly in alpha thalassemia.

Answer. c

193. Most common translocation seen in Burkitt lymphoma is?

a. t(8;22)

b. t(8;14)

c. t(8;21)

http://cbt.damsdelhi.com/damscbtadmin/admin/index.php?pageName=test/export&id=1710001
xxxxxxxxxxxxxxxxxxxxxxxxxxxxxxxxxxxxxxxxxxxxxxxxxxxxxxxxxxxxxxxxxxxxxxxxxxxxxxxxxxxxxx 68/121
15 oct, 2017
10/25/2017 Test Information
d. t(2;8)

Solution. B

In Burkitt lymphoma, c-myc amplification leading to its overexpression is noted. Different translocations involving the gene locus on chromosome 8 have
been identified- t(8;14), t(2;8) and t(8;22)- out of which the first accounts for more than 70% cases.

Answer. b

194.

A child presents with protrusion of eyes, increased thirst and diuresis and bony defects in the calvarium. A biopsy was performed and electron microscopy showed the
following. Which marker would confirm the diagnosis of this condition?

a. S-100

b. MHC II

c. CD 207

d. All of the above

Solution. D

The clinical features and electron microscopy picture are consistent with Hand Schuller Christian triad- which is a multifocal unisystem occurrence of
Langerhans cell histiocytosis. The markers for LCH are all mentioned (CD 207 is Langerin)

Answer. d

195. Thorium induced tumor is

a. Hepatic angiosarcoma

b. Rhabdomyosarcoma

c. Renal cell carcinoma

d. Non small cell carcinoma

Solution. A

Thorium used in contrast dyes and polyvinyl chloride are both implicated in causation of hepatic angiosarcoma

Answer. a

196. All of the following can occur due to asbestos exposure except

a. Mesothelioma

b. Laryngeal carcinoma

c. Ovarian carcinoma

d. Testicular tumors

Solution. D

Mesothelioma (most typical), lung carcinoma (most common), laryngeal, ovarian, colorectal carcinomas have all been implicated in asbestos exposure.

Answer. d

http://cbt.damsdelhi.com/damscbtadmin/admin/index.php?pageName=test/export&id=1710001
xxxxxxxxxxxxxxxxxxxxxxxxxxxxxxxxxxxxxxxxxxxxxxxxxxxxxxxxxxxxxxxxxxxxxxxxxxxxxxxxxxxxxx 69/121
15 oct, 2017
10/25/2017 Test Information

197. A young patient with cough, wheezing, abdominal pain and deranged liver function tests is being evaluated. Liver biopsy stained with a special stain
shows the following. What finding will you expect in the lung?

a. Irreversible enlargement of airspaces distal to terminal bronchiole

b. Hyperplastic mucus glands in the bronchial walls

c. Smooth muscle hypertrophy with eosinophilia in the bronchial walls

d. Smooth muscle and elastic tissue destruction

Solution. A

The picture is that of a PAS stained liver biopsy. PAS positive inclusions can be seen within the hepatocytes. The patient is a young child with respiratory
and hepatic derangement symptoms. It is a case of alpha 1 antitrypsin deficiency leading to panacinar emphysema in the lungs. The histological
description of option A is most consistent with that.

Answer. a

198. All are markers for malignant melanoma except

a. HMB 45

b. MART1

c. SOX 11

d. SOX 10

Solution. C

SOX 11 is a marker for mantle cell lymphoma.

Answer. c

199. Maximum malignant potential is of which of the following conditions?

a. Familial adenomatous polyposis

b. Juvenile polyp

c. PeutzJegher polyp

d. Ulcerative colitis

Solution. A

Familial adenomatous polyposis is an inherited condition with neoplastic polyps, whereas JPS and PJP are classified under non neoplastic hamartomatous
polyps. Ulcerative colitis and Crohns disease comprise the spectrum of inflammatory bowel disease and therefore also carry a risk of colorectal cancer
(equal incidence).

Answer. a

http://cbt.damsdelhi.com/damscbtadmin/admin/index.php?pageName=test/export&id=1710001
xxxxxxxxxxxxxxxxxxxxxxxxxxxxxxxxxxxxxxxxxxxxxxxxxxxxxxxxxxxxxxxxxxxxxxxxxxxxxxxxxxxxxx 70/121
15 oct, 2017
10/25/2017 Test Information
200. A child with delayed growth and history of malabsorption presents with pruritic, grouped vesicular rashes on the extensor surface of elbows and
knees. Immunofluorescence study shows the following. Which of the following findings can be expected if a biopsy from the small intestine is taken?

a. Normal histology

b. Crypt hyperplasia

c. Lipid laden epithelial cells

d. Lipid laden macrophages

Solution. B

The picture above shows immunofluorescence in tips of dermal papillae only. The clinical picture is most consistent with that of dermatitis herpetiformis.
An intestinal association of this condition is celiac disease (gluten sensitive enteropathy) which is characterised by loss of villus architecture but crypt
hyperplasia on biopsy evaluation.

Answer. b

201. Based on the dose response curve, which one of the following statement is true

a. Drug A is more potent and more efficacy than drug C

b. Drug A more potent but less efficacy than drug C

c. Drug A is more efficacy but less potent than drug B

d. Drug A is less efficacy and less potent than drug A

Solution. (a) Drug A is more potent and more efficacy than drug C

Ref:Read the text below

Sol:

http://cbt.damsdelhi.com/damscbtadmin/admin/index.php?pageName=test/export&id=1710001
xxxxxxxxxxxxxxxxxxxxxxxxxxxxxxxxxxxxxxxxxxxxxxxxxxxxxxxxxxxxxxxxxxxxxxxxxxxxxxxxxxxxxx 71/121
15 oct, 2017
10/25/2017 Test Information

Answer. a

202. Which one of the following receptor is mediated through inhibitory adenylcyclase pathway

a. M2

b. M1

c. Nm

d. Alpha 1

Solution. (a) M2

Ref:Read the text below

Sol:

M2 M4 Gi pathway

M1.M3, M5 Gq pathway

Nm- ligand gated (Na, Ca)

Alpha 2 Gi pathway

Alpha1- Gq pathway

Answer. a

203. Interleukin -6 blocker useful in rheumatoid arthritis is

a. Dexamethasone

b. Tocilizumab

c. Reislzumba

d. Anakinra

Solution. (b) Tocilizumab

Ref:Read the text below

Sol:

Anakinra- Analogue of IL 1 receptor Antagonist-

Resilzumab- IL 5 Antagonist- useful in bronchial asthma

Tocilizumab-IL 6 blocker

Steroids-IL, IL 6 blocker

http://cbt.damsdelhi.com/damscbtadmin/admin/index.php?pageName=test/export&id=1710001
xxxxxxxxxxxxxxxxxxxxxxxxxxxxxxxxxxxxxxxxxxxxxxxxxxxxxxxxxxxxxxxxxxxxxxxxxxxxxxxxxxxxxx 72/121
15 oct, 2017
10/25/2017 Test Information
Answer. b

204. Type B adverse reaction is

a. Augmented effect of the drug

b. Allergic effect of the drug

c. Effects seen on chronic use of the drug

d. Delayed effect of the drug

Solution. (b) Allergic effect of the drug

Ref:Read the text below

Sol:

TYPE A- (AUGMENTED )REACTIONS

o Hypoglycemia with insulin

TYPE B- (BIZARRE) REACTIONS

o Penicillin allergy, chloramphenicol- Anemia

TYPE C- (CHRONIC) REACTIONS

o Peptic ulcer- NSAIDs

TYPE D -(DELAYED) REACTIONS

o Teratogenicity -Thalidomide

TYPE E- (END OF USE ) REACTIONS

o Withdrawal response- Morphine

TYPE F FAILURE OF THERAPY

Answer. b

205. Choose the most inappropriate statement regarding Nabumetone

a. It is the only nonacid NSAID

b. It has short half life

c. May cause apseudoporphyria

d. unsafe in renal failure

Solution. (b) It has short half life

Ref:Read the text below

Sol:

- Nabumetone is the only nonacid NSAID in current use; it is given as a ketone prodrug, Its half-life of more than 24 hours permits once-daily
dosing, Renal impairment results in a doubling of its half-life and a 30% increase in the area under the curve, has been associated with pseudoporphyria
and photosensitivity in some patients

Answer. b

206. Valbenazine useful in treatment of

a. Tardive dyskinesia

b. Narcolepsy

c. Attention deficit hyperkinetic disorder

d. Mania

http://cbt.damsdelhi.com/damscbtadmin/admin/index.php?pageName=test/export&id=1710001
xxxxxxxxxxxxxxxxxxxxxxxxxxxxxxxxxxxxxxxxxxxxxxxxxxxxxxxxxxxxxxxxxxxxxxxxxxxxxxxxxxxxxx 73/121
15 oct, 2017
10/25/2017 Test Information
Solution. (a) Tardive dyskinesia

Ref:Read the text below

Sol:

VALBENAZINE-

Vesicular monoamine transporter 2 inhibitor

Valbenazine is used to treat tardive dyskinesia in adults

Valbenazine has not been effectively studied in pregnancy, and it is recommended that women who are pregnant or breastfeeding avoid use of
valbenazine

Answer. a

207. Which of the following is not a hit and run drug?

a. Clorgiline

b. Omeprazole

c. Digoxin

d. Reserpine

Solution. (c) Digoxin

Ref:Read the text below

Sol:

Hit and run drugs: MAO inhibitors, PPIs, Guanethidine, Reserpine, Aspirin

Answer. c

208. ARB having PPAR gamma agonism property is

a. Telmisartan

b. Valsartan

c. Losartan

d. Azilsartan

Solution. (a) Telmisartan

Ref:Read the text below

Sol:

Telmisartan is indicated in the treatment of essential hypertension

Telmisartan is an angiotensin II receptor blocker that shows high affinity for the angiotensin II receptor type 1 (AT1), with a binding affinity 3000
times greater for AT1 than AT2.

In addition to blocking the RAS, telmisartan acts as a selective modulator of peroxisome proliferator-activated receptor gamma(PPAR-), a central
regulator of insulin and glucose metabolism. It is believed that telmisartans dual mode of action may provide protective benefits against the vascular
and renal damage caused by diabetes and cardiovascular disease (CVD).

Telmisartan's activity at the peroxisome proliferator-activated receptor delta (PPAR-) receptor has prompted speculation around its potential as a
sport doping agent as an alternative to GW 501516.

Telmisartan activates PPAR- receptorsin several tissues

Answer. a

209. All of the following are natural anticancer drugs except

http://cbt.damsdelhi.com/damscbtadmin/admin/index.php?pageName=test/export&id=1710001
xxxxxxxxxxxxxxxxxxxxxxxxxxxxxxxxxxxxxxxxxxxxxxxxxxxxxxxxxxxxxxxxxxxxxxxxxxxxxxxxxxxxxx 74/121
15 oct, 2017
10/25/2017 Test Information
a. Topotecan

b. Etoposide

c. L asparaginase

d. Pemetrexed

Solution. (d) Pemetrexed

Ref:Read the text below

Sol:

Natural products

i. Vinca alkaloids: Vincristine, vinblastine, vinorelbine

ii. Taxanes: Paclitaxel, docetaxel

iii. Epipodophyllotoxins: Etoposide, teniposide

iv. Camptothecins: Topotecan, irinotecan

v. Antitumor antibiotics: Anthracyclines (daunorubicin, doxorubicin, epirubicin, idarubicin,mitoxantrone), bleomycin, dactinomycin, mitomycin-C

vi. Enzymes: L-asparaginase

Answer. d

210.

Drug useful in Midgut neuroendocrine tumor

a. Lu-Dotatate

b. Ivacaftor

c. Apafant

d. Anagrelide

Solution. (a) Lu-Dotatate

Ref:Read the text below

Sol:

Lu-Dotatate - Midgut neuroendocrine tumor

Ivacaftor- Cystic fibrosis

Apafant-Acute pancreatitis

Anagrelide-Essential thrombocytosis

Answer. a

211. Histone deaceylase inhibitor useful n cutaneous T cell lymphoma is

a. Vorinostat

b. Aldeslukin

c. Panobinostat

d. Mepolizumab

Solution. (a) Vorinostat

Ref:Read the text below

Sol:

Histone De Acetylase inhibitor

VORINOSTAT, ROMIDEPSIN- useful in Cutaneous T cell lymphoma

http://cbt.damsdelhi.com/damscbtadmin/admin/index.php?pageName=test/export&id=1710001
xxxxxxxxxxxxxxxxxxxxxxxxxxxxxxxxxxxxxxxxxxxxxxxxxxxxxxxxxxxxxxxxxxxxxxxxxxxxxxxxxxxxxx 75/121
15 oct, 2017
10/25/2017 Test Information
Panobinostat- useful in multiple myeloma

Aldeslukin IL2 analogue useful RCC, malignant melanoma

Mepolizumab- IL 5 blocker useful in Hypereosinophiicsynd, churg- strausssynd

Answer. a

212. Monoclonal antibody targeting against CD30 is

a. Brentuximab

b. Gemtuzumab

c. Rituximab

d. Alemtuzumab

Solution. (a) Brentuximab

Ref:Read the text below

Sol:

Brentuximab CD 30 Hodgkin lymphoma, Anaplastic large cell lympoma

Rituximab CD-20 Non-Hodgkin lymphoma

Gemtuzuma- --CD-33 CD-33 positive AML Linked to calicheamicin

Alemtuzumab--CD-52 Low grade lymphomas,and CLL

Answer. a

213. Antibiotic acting by inhibiting Transglycosylase enzyme is

a. Chlormphenical

b. Vancomycin

c. Penicillin

d. Cycloserine

Solution. (b) Vancomycin

Ref:Read the text below

Sol:

Vancomycin inhibits transglycosylase

Chloramphenicol- inhibits protein synthesis

Penicillin inhibits Transpeptidase

Cycloserine inhibits Alanine ligase/racemase

Answer. b

214. ATT safe in hepatic failure are:

a. Pyrazinamide and ethambutol

b. INH and Rifampicin

c. Streptomycin and Ethambutol

d. Rifampicin and Streptomycin

Solution. (c) Streptomycin and Ethambutol

Ref:Read the text below

http://cbt.damsdelhi.com/damscbtadmin/admin/index.php?pageName=test/export&id=1710001
xxxxxxxxxxxxxxxxxxxxxxxxxxxxxxxxxxxxxxxxxxxxxxxxxxxxxxxxxxxxxxxxxxxxxxxxxxxxxxxxxxxxxx 76/121
15 oct, 2017
10/25/2017 Test Information
Sol:

Acute Hepatitis

Patients with acute hepatitis rarely need to be treated for tuberculosis on an urgent basis. Since ATT can be delayed, it should be deferred until acute
hepatitis has resolved.

Once there is evidence of acute hepatitis in a patient receiving ATT, it is essential to immediately stop all potentially hepatotoxic drugs such as INH,
RIF, and PZA till complete clinical and biochemical resolution of hepatotoxicity. In the interim period, at least three non-hepatotoxic drugs viz. EMB,
streptomycin and quinolones such as ofloxacin, levofloxacin, etc. can be used after checking renal function and visual acuity

Most ATD can be safely restarted in a phased manner after complete resolution of transaminitis.

Chronic Hepatitis B Virus Infection

Hepatitis B virus (HBV) infection has been reported to be a significant risk factor for hepatotoxicity related to ATT

Isoniazid monotherapy is safe in patients with HBV infectionwhile multidrug ATT is associated with significant incidence of hepatotoxicity.

Multidrug therapy for tuberculosis is also associated with fulminant disease, increased mortality and later onset of hepatotoxic effects in these patients

Answer. c

215. Crofelemer useful in treating

a. Diarrhoea in HIV

b. Constipation in HIV

c. Neuropathy in HIV

d. Lymphoma in HIV

Solution. (a) Diarrhoea in HIV

Ref:Read the text below

Sol:

Crofelemer-

Crofelemer (USAN, trade name Mytesi) is a botanical drug for the treatment of diarrhoea associated with anti-HIV drugs such as nucleoside
analog reverse transcriptase inhibitors and protease inhibitors.

Other possible uses include diarrhoea in children, acute infectious diarrhoea, and diarrhoea in patients with irritable bowel syndrome

Answer. a

216. The dose of metronidazole for intestinal amebiasis is

a. 400 mg TDS for 5 7 days oral

b. 400 mg TDS for 7 10 days oral

c. 800 mg TDS for 7 10 days oral

d. 15 mg/kg IV over 1 hour followed by 7.5 mg/kg every 6 hours

Solution.

(a) 400 mg TDS for 5 7 days oral

Ref: Read the text below

Sol:

Doses of metronidazole:

Option 1: Intestinal amebiasis

Option 2: Giardiasis and trichomoniasis

Option 3: Extra-intestinal amebiasis

Op on 4: Anaerobic bacterial infec ons

Answer. a
http://cbt.damsdelhi.com/damscbtadmin/admin/index.php?pageName=test/export&id=1710001
xxxxxxxxxxxxxxxxxxxxxxxxxxxxxxxxxxxxxxxxxxxxxxxxxxxxxxxxxxxxxxxxxxxxxxxxxxxxxxxxxxxxxx 77/121
15 oct, 2017
10/25/2017 Test Information

217. Anti emetic having appetite stimulating property is

a. Ondensetran

b. Nabilone

c. Aprepitant

d. Domperidone

Solution. (b) Nabilone

Ref:Read the text below

Sol:

Nabilone cannabinoid agonist- anti emetic as well as appetite stimulant

Aprepitant- NK 1 antagonist, useful for controlling delayed phase vomiting following anti cancer therapy

Answer. b

218. Albendazole is effective against all of the following except:

a. Roundworm

b. Hookworm

c. Tapeworm

d. Pinworm

Solution. (c) Tapeworm

Ref:Read the text below

Sol:

Albendazole is the drug of choice for the treatment of all nematode infestations including cutaneous larva migrans (creeping eruption), visceral
larva migrans

(toxocariasis) and neurocysticercosis except enterobius (mebendazole), wuchereria and brugia (DEC), onchocerca and strongyloides (ivermectin)
and dracunculus (Metronidazole).

Answer. c

219. Antiglaucoma drug causing iris pigmentation is

a. Apraclonidine

b. Latanaprost

c. Dipivefrine

d. Betaxalol

Solution. (b) Latanaprost

Ref:Read the text below

Sol:

Apraclonidine- alpha 2 agonist, causes lid lag

Latenaprost- PGF2 alpha, causes iris pigmentation

Dipivefrine- prodrug of epinephrine- causes cystoid macular edema

Betaxalol- beta blocker- safe in asthma patients with glaucoma

Answer. b

http://cbt.damsdelhi.com/damscbtadmin/admin/index.php?pageName=test/export&id=1710001
xxxxxxxxxxxxxxxxxxxxxxxxxxxxxxxxxxxxxxxxxxxxxxxxxxxxxxxxxxxxxxxxxxxxxxxxxxxxxxxxxxxxxx 78/121
15 oct, 2017
10/25/2017 Test Information
220. Nasopharyngitis is the common side effect of

a. Linagliptin

b. Exenatide

c. Pramlintide

d. Acarbose

Solution. (a) Linagliptin

Ref:Read the text below

Sol:

Linagliptine-DPP4 INHIBITORS, ADR- Nasophargyingitis, URTI

Exenatide-GLP-1 analouge, ADR- gastero intestinal problem like nausea, vomiting and diarrhoea is common, rarely Necrotising pancreatitis

PRAMLINTIDE-Islet Amyloid PolyPeptideanalog, given S/C, useful in both Type 1 & 2 DM

ACARBOSE-aGLUCOSIDASE INHIBITORS, Side effects Flatulence, Abd distension diarrhea

Answer. a

221.

Cardiac index is

a. cardiac output per kg body weight

b. cardiac output per meter height

c. cardiac output per sq m of body surface area

d. cardiac output per unit mass

Solution. C

Cardiac index is defined as cardiac output per square meter of body surface area and it averages 3.2L/min/sq meter.

Answer. c

222. All of the following cause heterometric regulation of cardiac output except

a. increase in venous return

b. increase in end diastolic volume

c. increased ventricular filling

d. increased myocardial contractility

Solution. D

When cardiac output is regulated by changes in cardiac muscle fiber length, this is referred to as heterometric regulation. Conversely, changes in
contractility independent of length is called homometric regulation of cardiac output.

Answer. d

223. During exercise, a man consumes 1.8 L of oxygen per minute. His arterial O2 content is 190mL/ L and his O2 content of his mixed venous blood is
130mL/ L. His cardiac output is approximately

a. 10.0 L/min

b. 20.0 L/min

c. 30.0 L/min

d. 40.0 L/min

http://cbt.damsdelhi.com/damscbtadmin/admin/index.php?pageName=test/export&id=1710001
xxxxxxxxxxxxxxxxxxxxxxxxxxxxxxxxxxxxxxxxxxxxxxxxxxxxxxxxxxxxxxxxxxxxxxxxxxxxxxxxxxxxxx 79/121
15 oct, 2017
10/25/2017 Test Information
Solution. C

To calculate flow or cardiac output (L/min) by the Ficks principle, divide the oxygen consumption in mL/min by the arterio-venous oxygen difference in
mL/L.

Answer. c

224. The work done by the left ventricle is significantly higher than the right ventricle even at rest, because

a. left ventricle has a thicker wall

b. left ventricle has a larger force of contraction

c. left ventricle has a slower contraction

d. afterload for the left ventricle is greater

Solution. D

Work done = stroke volume X MAP

Since the resistance and therefore, the afterload is more for the left ventricle, work done by the left ventricle is substantially higher by the left ventricle.

Answer. d

225. The lung volume at the end of a maximum inspiration is

a. inspiratory capacity

b. forced vital capacity

c. functional residual capacity

d. total lung capacity

Solution. D

Lung volume at the end of maximum inspiration is total lung capacity.

Answer. d

226. All of the following are effects of hyperventilation except

a. arterial Pco2 may fall as low as 15 mm Hg

b. alveolar Po2 increases to 120-140 mm Hg

c. cerebral blood flow reduces by 30% or more

d.

blood pH decreases to 7.3 or 7.2

Solution. D

During voluntary hyperventilation arterial Pco2 may fall as low as 15 mm Hg, alveolar Po2 increases to 120-140 mm Hg and cerebral blood flow reduces
by 30% or more because of the direct vasoconstrictor effect of hypocapnia on cerebral blood vessels. The cerebral ischemia causes light headedness,
dizziness, and paresthesias. Voluntary hyperventilation causes a CO2 washout resulting in a respiratory alkalosis.

Answer. d

227. Lactic acid given intravenously increases rate and depth of ventilation due its action on

a. medullary chemoreceptors

b. carotid bodies

c. central chemoreceptors

d. pulmonary chemoreceptors

http://cbt.damsdelhi.com/damscbtadmin/admin/index.php?pageName=test/export&id=1710001
xxxxxxxxxxxxxxxxxxxxxxxxxxxxxxxxxxxxxxxxxxxxxxxxxxxxxxxxxxxxxxxxxxxxxxxxxxxxxxxxxxxxxx 80/121
15 oct, 2017
10/25/2017 Test Information

Solution. b

H+ stimulates the peripheral chemoreceptors to increase the rate and depth of ventilation. Lactic acid given intravenously will not have any effect on the
central or medullary chemoreceptors because it cannot cross the blood brain barrier

Answer. b

228.

Maximum calcium reabsorption occurs in which part of the nephron?

a. PCT

b. TAL

c. DCT

d. CD

Solution. A

Maximum calcium (65%) is reabsorbed in the PCT. 25-30% of the filtered calcium is reabsorbed in the thick ascending limb. Calcium reabsorption in the
DCT (4-9%) is under the control of parathormone. Less than 1% of the filtered calcium is excreted in the urine.

Answer. a

229. Gastric lipase is produced by which of the following?

a. oxyntic cells

b. parietal cells

c. chief cells

d. ECL cells

Solution. c

Oxyntic or parietal cells secrete HCl and intrinsic factor. Chief cells or peptic cells secrete pepsinogens and gastric lipase. ECL or enterochromaffin like
cells secrete histamine.

Answer. c

230. The largest daily secretion is from which of the following

a. salivary glands

b. stomach

c. bile

d. pancreatic

Solution. B

Daily secretion of saliva is 800-1200mL. Gastric secretion is 2000-2500mL per day. Bile is approximately 500mL per day. Pancreatic secretion is 1500mL
per day.

Answer. b

231. Which of the following neurons of the cerebellum secretes glutamate?

a. Basket cells

b. Granule cells

c. Stellate cells

d. Golgi cells

http://cbt.damsdelhi.com/damscbtadmin/admin/index.php?pageName=test/export&id=1710001
xxxxxxxxxxxxxxxxxxxxxxxxxxxxxxxxxxxxxxxxxxxxxxxxxxxxxxxxxxxxxxxxxxxxxxxxxxxxxxxxxxxxxx 81/121
15 oct, 2017
10/25/2017 Test Information
Solution. B

The neurotransmitter released by purkinje, basket, stellate and golgi cells is GABA whereas granule cells release glutamate.

Answer. b

232. All of the following are true about short chain fatty acids except

a. SCFAs are produced in colon by the action of colonic bacteria on complex carbohydrates, resistant starches and dietary fiber

b. 60% of total is acetate

c. SCFAs are absorbed in the colon

d. SCFAs are absorbed by Na-SCFA co transport

Solution. D

SCFAs are produced in colon by the action of colonic bacteria on complex carbohydrates, resistant starches and dietary fiber and are absorbed in the
colon. 60% of total is acetate, 25% propionate, and 15% butyrate. Absorbed SCFAs are metabolized and make a significant contribution to the total caloric
intake. They exert a trophic effect on the colonic epithelial cells. SCFAs are absorbed by specific transporters present in colonic epithelial cells.

Answer. d

233. Which of the following movements does not occur in the colon?

a. peristalsis

b. segmentation contractions

c. mass action contractions

d. migrating motor complexes

Solution. D

Migrating motor complexes begin as a ring of contraction the body of stomach and terminate in the distal ileum.

Answer. d

234.

Which of the following innervate the extensor alpha motor neuron?

a. Reticulospinal tract

b. Lateral corticospinal tract

c. Rubrospinal tract

d. Vestibulospinal tract

Solution. d

Reticulospinal tracts synapse with A gamma motor neurons to the extensors.

Lateral corticospinal and rubrospinal tracts innervate the A alpha neurons to the flexors. Vestibulospinal tract innervate the A alpha to the extensors.

Answer. d

235. The circuit shown below has an inflow pressure of 120 mm Hg and an outflow pressure of 40 mm Hg. Resistance in each of the four vessels is 2 mm
Hg/mL/min. What is the total peripheral resistance of the circuit?

http://cbt.damsdelhi.com/damscbtadmin/admin/index.php?pageName=test/export&id=1710001
xxxxxxxxxxxxxxxxxxxxxxxxxxxxxxxxxxxxxxxxxxxxxxxxxxxxxxxxxxxxxxxxxxxxxxxxxxxxxxxxxxxxxx 82/121
15 oct, 2017
10/25/2017 Test Information
a. 8 mm Hg/mL/min

b. 4 mm Hg/mL/min

c. 2 mm Hg/mL/min

d. 0.5 mm Hg/mL/min

When vessels are arranged in parallel, the total resistance is the sum total of the reciprocal of the resistance of the individual vessel.

1/R = 1/R1 + 1/R2 +1/R3 +1/R4

Answer. d

236. A smoker states that he has been smoking for 6 years. In the first year he was taking up to 5 sticks per day only. In the next 3 years he increased it
to half pack per day (which was an odd number). In the 5th year, his habits worsened to 1 pack per day. In the last year he stated that his daily
sticks consumption is 2 packs per day. Select the correct statement for mean, median and mode of number of sticks are

a. 16,10,15

b. 16,10,10

c. 24,15,15

d. 16,16,15

Solution. (c) 24,15,15

Ref:Read the text below

Sol:

http://cbt.damsdelhi.com/damscbtadmin/admin/index.php?pageName=test/export&id=1710001
xxxxxxxxxxxxxxxxxxxxxxxxxxxxxxxxxxxxxxxxxxxxxxxxxxxxxxxxxxxxxxxxxxxxxxxxxxxxxxxxxxxxxx 83/121
15 oct, 2017
10/25/2017 Test Information

Answer. c

237. Throat swab was done on a child & stool sample was also collected after the child had developed weakness in limbs. Child had no history of any
recent vaccination. Sewer sample were also positive for the virus. No wild strain was found in the community. Strain is:

a. Type 1

b. Type 2

c. Type 3

d. Non typable strain

Solution. (b) Type 2

Ref:Read the text below

Sol:

Answer. b

238. A study was conducted to assess relationship between height and osteoporosis. In this two participants were chosen with the following details. This
is:

http://cbt.damsdelhi.com/damscbtadmin/admin/index.php?pageName=test/export&id=1710001
xxxxxxxxxxxxxxxxxxxxxxxxxxxxxxxxxxxxxxxxxxxxxxxxxxxxxxxxxxxxxxxxxxxxxxxxxxxxxxxxxxxxxx 84/121
15 oct, 2017
10/25/2017 Test Information

a. Matched

b. Overmatched

c. Unmatched

d. Randomized

Solution. (b) Overmatched

Ref:Read the text below

Sol:

Process of selecting controls so that they are similar to cases with regard to certain variables

Dont match for the variable in interest as wont be able to get statistical results

Eliminates confounding

Done in: Case control >> Cohort

http://cbt.damsdelhi.com/damscbtadmin/admin/index.php?pageName=test/export&id=1710001
xxxxxxxxxxxxxxxxxxxxxxxxxxxxxxxxxxxxxxxxxxxxxxxxxxxxxxxxxxxxxxxxxxxxxxxxxxxxxxxxxxxxxx 85/121
15 oct, 2017
10/25/2017 Test Information

Answer. b

239. Which method is used to diagnose TB in HIV positive patients?

a. CXR

b. Smear examination

c. Both

d. CBNAAT

Solution. (d) CBNAAT

http://cbt.damsdelhi.com/damscbtadmin/admin/index.php?pageName=test/export&id=1710001
xxxxxxxxxxxxxxxxxxxxxxxxxxxxxxxxxxxxxxxxxxxxxxxxxxxxxxxxxxxxxxxxxxxxxxxxxxxxxxxxxxxxxx 86/121
15 oct, 2017
10/25/2017 Test Information
Ref:Read the text below

Sol:

- As per the new revised guidelines of RNTCP any patient with HIV positive status; the test to be performed is CBNAAT

Answer. d

240. Which study design does this diagram pertain to?

a. Case control

b. Cohort

c. Ecological

d. RCT

Solution. (c) Ecological

Ref:Read the text below

Sol:

The diagram provides country level estimates so pertains to ecological study

Answer. c

241. Which kind of epidemic curve will this diagram depict?

http://cbt.damsdelhi.com/damscbtadmin/admin/index.php?pageName=test/export&id=1710001
xxxxxxxxxxxxxxxxxxxxxxxxxxxxxxxxxxxxxxxxxxxxxxxxxxxxxxxxxxxxxxxxxxxxxxxxxxxxxxxxxxxxxx 87/121
15 oct, 2017
10/25/2017 Test Information

a. Common source point

b. Common source propagated

c. Mixed

d. Recurrent

Solution. (a) Common source point

Ref:Read the text below

Sol:

http://cbt.damsdelhi.com/damscbtadmin/admin/index.php?pageName=test/export&id=1710001
xxxxxxxxxxxxxxxxxxxxxxxxxxxxxxxxxxxxxxxxxxxxxxxxxxxxxxxxxxxxxxxxxxxxxxxxxxxxxxxxxxxxxx 88/121
15 oct, 2017
10/25/2017 Test Information

Answer. a

242. What is the median survival in this diagram?

a. 8 years

b. 10 years

c. 12 years

d. 6 years

Solution. (b) 10 years

Ref:Read the text below

Sol:

Answer. b

243. This diagram is the pictorial representation of which type of study design?

http://cbt.damsdelhi.com/damscbtadmin/admin/index.php?pageName=test/export&id=1710001
xxxxxxxxxxxxxxxxxxxxxxxxxxxxxxxxxxxxxxxxxxxxxxxxxxxxxxxxxxxxxxxxxxxxxxxxxxxxxxxxxxxxxx 89/121
15 oct, 2017
10/25/2017 Test Information

a. RCT

b. Cross sectional

c. Ecological

d. Any of these

Solution. (a) RCT

Ref:Read the text below

Sol:

Answer. a

244. In the cycle provided; which is correct?

http://cbt.damsdelhi.com/damscbtadmin/admin/index.php?pageName=test/export&id=1710001
xxxxxxxxxxxxxxxxxxxxxxxxxxxxxxxxxxxxxxxxxxxxxxxxxxxxxxxxxxxxxxxxxxxxxxxxxxxxxxxxxxxxxx 90/121
15 oct, 2017
10/25/2017 Test Information

a. Monitoring done before evaluation

b. Evaluation is done after monitoring

c. Both done simultaneously

d. Not done in this cycle at all

Solution. (a) Monitoring done before evaluation

Ref:Read the text below

Sol:

Answer. a

245. Which cut off would you prefer to take?

http://cbt.damsdelhi.com/damscbtadmin/admin/index.php?pageName=test/export&id=1710001
xxxxxxxxxxxxxxxxxxxxxxxxxxxxxxxxxxxxxxxxxxxxxxxxxxxxxxxxxxxxxxxxxxxxxxxxxxxxxxxxxxxxxx 91/121
15 oct, 2017
10/25/2017 Test Information

a. 3

b. 6

c. 15

d. 1.1

Solution. (b) 6

Ref:Read the text below

Sol:

This is a ROC/ Receiver Operator Curve which is used to select the cut offs

X- axis: 1 Specificity/ FP

Y- axis: Sensitivity/ TP

Types:

Straight line at 45o (a): no benefit by this cut off

(b) & (c): fair & good results

(d): perfect ROC

Excellent

100% sensitivity & specificity

http://cbt.damsdelhi.com/damscbtadmin/admin/index.php?pageName=test/export&id=1710001
xxxxxxxxxxxxxxxxxxxxxxxxxxxxxxxxxxxxxxxxxxxxxxxxxxxxxxxxxxxxxxxxxxxxxxxxxxxxxxxxxxxxxx 92/121
15 oct, 2017
10/25/2017 Test Information

Answer. b

246. A person died in emergency. The anaesthetist was resuscitating the patient but in middle his mask fell off and had contact with patients mucosal
secretion as patient had coughed out gastric contents. The anaesthetist died after 10 days. No unnatural reasons were found. On autopsy intra
cytoplasmic bodies were found. The most probable infectious agent is:

a. Rabies

b. Botulism

c. Anthrax

d. Cause Unknown

Solution. (a) Rabies

Ref:Read the text below

Sol:

The picture is that of intra cytoplasmic bodies classically seen in rabies.

Clinical history shows death in ten days which is a feature of rabies

Also there is might have been mucosal contact

Answer. a

247. The intraocular pressure(IOP) was measured in a population of 400 people above age of 65. The mean IOP was 25 & SD was 10. What is the range
that would contain IOP of 95% of population:

a. 23-27

b. 21-29

c. 24-26

d. 22-28

http://cbt.damsdelhi.com/damscbtadmin/admin/index.php?pageName=test/export&id=1710001
xxxxxxxxxxxxxxxxxxxxxxxxxxxxxxxxxxxxxxxxxxxxxxxxxxxxxxxxxxxxxxxxxxxxxxxxxxxxxxxxxxxxxx 93/121
15 oct, 2017
10/25/2017 Test Information
Solution. (c) 24-26

Ref:Read the text below

Sol:

N = 400; Mean = 25; SD = 10; 95% CI = ?????

SE = SD/ square root of N = 10/ square root 400 = 10/ 20 = 0.5

CI = Mean 2 SE = 25 2 * 0.5 = 24- 26

CAUTION: Dont forget to calculate SE whenever you calculate CI

Answer. c

248. True for Pap smear is:

a. Visual inspection of cervix is good enough in patient > 65 years old & regular PAP screening is not needed

b. In case of HPV testing negative test can be repeated once in 5 year

c. If PAP smear did not show any abnormality test can be repeated in 3 years

d. Screening to be done in age group of 25 and 40 years as it the most needed time

Solution. (d) Screening to be done in age group of 25 and 40 years as it the most needed time

Ref:Read the text below

Sol:

Prevention of Cervical Cancer


Primary Prevention: vaccinate 9 to 13 years with two doses of HPV vaccine
Delhi: 1st to start HPV vaccine for school kids of Class VI girls in government schools
Secondary Prevention: Using VIA (Visual Inspection with Acetic Acid) or HPV testing to screen women for cervical cancer prevention above 29
years of age (d)
Once a woman is screened to be HPV negative, she should not undergo re- screening for at least 5 year (c), but before 10 years (b)

Answer. d

249. In a centre number of nodes dissected during 20 modified radical mastectomy were plotted in form of curve & there are 3 markings on
curve below. Based on the distribution, which of following is correct?

a. Mean > median > mode

b. Mode > mean > median

c. Median > mode > mean

d. Mode > mean > median

Solution. (a) Mean > median > mode

Ref:Read the text below

http://cbt.damsdelhi.com/damscbtadmin/admin/index.php?pageName=test/export&id=1710001
xxxxxxxxxxxxxxxxxxxxxxxxxxxxxxxxxxxxxxxxxxxxxxxxxxxxxxxxxxxxxxxxxxxxxxxxxxxxxxxxxxxxxx 94/121
15 oct, 2017
10/25/2017 Test Information
Sol:

Answer. a

250. Match the following and what are the options?

a. 1 III; 2 II; 3 V; 4 IV; 5 - I

b. 1 I; 2 II; 3- III; 4 IV; 5 V

c. 1 III; 2 V; 3 II; 4 I; 5 IV

d. 1 I; 2 III; 3 II; 4 V; 5 - IV

Solution. (a) 1 III; 2 II; 3 V; 4 IV; 5 - I

Ref:Read the text below

Sol:

Answer. a

251. Which is not true for Reduvid Bug?

a. Also called kissing bug

b. Also called cone nose bug

c. Causes sleeping sickness


http://cbt.damsdelhi.com/damscbtadmin/admin/index.php?pageName=test/export&id=1710001
xxxxxxxxxxxxxxxxxxxxxxxxxxxxxxxxxxxxxxxxxxxxxxxxxxxxxxxxxxxxxxxxxxxxxxxxxxxxxxxxxxxxxx 95/121
15 oct, 2017
10/25/2017 Test Information

d. Causes African Trypnosomiasis

Solution. (d) Causes African Trypnosomiasis

Ref:Read the text below

Sol:

Tse- tse fly (Glossina palpalis): Sleeping sickness (African trypanosomiasis)

Reduvid bug (Triatominae/ Kissing/ Assassin/ Cone nose bug): Sleeping sickness (American trypanosomiasis (Chagas Disease)

Answer. d

252. Which of the following is false about the best indicators for disease burden?

a. Mortality indicator Proportional Mortality Rate

b. Morbidity indicator Prevalence

c. Health index DALY

d. Health index - HALE

Solution. (c) Health index DALY

Ref:Read the text below

Sol:

Answer. c

253. Which is not correct about the nodal heads for disaster management?

a. Head Prime Minister

b. Level CHC

c. Agency National Disaster Management Authority

d. Ministry - Home

Solution. (b) Level CHC

Ref:Read the text below

Sol:

http://cbt.damsdelhi.com/damscbtadmin/admin/index.php?pageName=test/export&id=1710001
xxxxxxxxxxxxxxxxxxxxxxxxxxxxxxxxxxxxxxxxxxxxxxxxxxxxxxxxxxxxxxxxxxxxxxxxxxxxxxxxxxxxxx 96/121
15 oct, 2017
10/25/2017 Test Information
Answer. b

254. For which vaccine is the open vial policy applicable?

a. Rota

b. JE

c. Yellow fever

d. IPV

Solution. (d) IPV

Ref:Read the text below

Sol:

Open Vial Policy

Reconstituted vaccines (B.C.G, MEASLES, J.E. & ROTAVIRUS) cant be reused

All vaccines which are not reconstituted can be reused for up to a maximum of 4 WEEKS/ 28 days if

Provided that all of the following conditions are met:

The expiry date has not passed; The vaccines are stored under appropriate cold chain conditions

The VVM has not been submerged in water

Aseptic technique has been used to withdraw all doses

VVM if attached, has not reached the discard point

Answer. d

255. Which is the highest level of learning?

a. Psychomotor

b. Cognitive

c. Affective

d. All are equal

Solution. (a) Psychomotor

Ref:Read the text below

Sol:

Answer. a

256. A 35 year old women suffering from recurrent major depression becomes very distressed when her supervisor asks her to revise a projects she has
been working on for weeks. I dont do anything right. I must be the most incompetent person in the firm. I will lose my job for sure she state, sobbing,
to her therapist. According to the cognitive model, her depression is a consequence of which of the following Problems?

a. Anger turned toward the self

b. Hopelessness

c. Worthlessness

http://cbt.damsdelhi.com/damscbtadmin/admin/index.php?pageName=test/export&id=1710001
xxxxxxxxxxxxxxxxxxxxxxxxxxxxxxxxxxxxxxxxxxxxxxxxxxxxxxxxxxxxxxxxxxxxxxxxxxxxxxxxxxxxxx 97/121
15 oct, 2017
10/25/2017 Test Information
d. Maladaptive negative beliefs

Solution. (d) Maladaptive negative beliefs

Ref:Read the text below

Sol:

The lady here is having cognitive distortion (also known as maladaptive assumptions or beliefs) that she has overgeneralised her problem when she
was asked to revise the project.

She did an arbitrary inference that I will lose my job for sure.

Answer. d

257. Which of the following is not true about KLEINE LEVIN SYNDROME?

a. Periodic hypersomnia followed by hyperphagia

b. Voracious eating

c. Self limiting in nature

d. Onset in 3rd decade of life

Solution. (d) Onset in 3rd decade of life

Ref:Read the text below

Sol:

Clinical features of kleine Levin syndrome

Periodic episodes of hypersomnia followed by hyperphagia

Episodes lasts for several weeks

1st attack between 10 -21 years of age

Self limiting

Intervening periods of normal sleep and waking

Other symptoms during hypersomnic episodes

Social withdrawal

Apathy

Irritability

Confusion

Voracious eating

Delusions/ hallucinations

Incoherent speech

Answer. d

258. Which of the following is not true about waveform seen in EEG below?

a. Seen in NREM 2 sleep

b. Disorder in this stage is bruxism

c. Rapid eye movements

http://cbt.damsdelhi.com/damscbtadmin/admin/index.php?pageName=test/export&id=1710001
xxxxxxxxxxxxxxxxxxxxxxxxxxxxxxxxxxxxxxxxxxxxxxxxxxxxxxxxxxxxxxxxxxxxxxxxxxxxxxxxxxxxxx 98/121
15 oct, 2017
10/25/2017 Test Information

d. Reduced EMG levels in this stage

Solution. (c) Rapid eye movements

Ref:Read the text below

Sol:

Sleep spindles and k complexesare seen in stage 2 nrem, with no rapid eye movements and reduced EMG levels.

Bruxism is associated with this stage.

Answer. c

259. An 18-year-old man is seen by a psychiatrist in the emergency room. During the history, the patient is asked to describe his mood. He answers the
following, My mood is flextitating, I am up and down. The patient is exhibiting which of the following thought disorders?

a. Clang association

b. Thought blocking

c. Tangentiality

d. Neologism

Solution. (d) Neologism

Ref:Read the text below

Sol:

A neologism is either the use of a completely made up word or phrase, or the use of an existing word or phrase in an idiosyncratic manner.

Clang associationsare thoughts which come out in a rhyming pattern, whether or not the verbalized sentence means anything logically.

Thought blocking is a sudden stoppage or blocking in the patients pattern of thought, so much so that speech is disrupted as well.

Tangentiality refersto a pattern of thought in which the patient answers a question with something that is related to the question, but does not answer it
directly. For example:

Question: How are you feeling?

Answer:This sofa is feeling particularly soft today.

Answer. d

260. A psychiatric resident is called to consult on the case of a 75-year-old woman who had undergone a hip replacement 2 days before. On
examination,the resident notes that the patient states the date as 1956, and she thinks she is at her sons house. These impairments best illustrate which
aspect of the mental status examination?

a. Concentration

b. Memory

c. Thought process

d. Orientation

Solution.

(d) Orientation

Ref: Read the text below

Sol:
Orientation refers to the state of awareness of the individual as to time and place, and to the awareness of the identity of oneself and others
in the environment.

This is the reason patients cognitive states are often referred to as oriented 3, meaning oriented to person, place, and time.

Answer. d

261. Psychotherapy of choice for Post Traumatic Stress Disorder?

http://cbt.damsdelhi.com/damscbtadmin/admin/index.php?pageName=test/export&id=1710001
xxxxxxxxxxxxxxxxxxxxxxxxxxxxxxxxxxxxxxxxxxxxxxxxxxxxxxxxxxxxxxxxxxxxxxxxxxxxxxxxxxxxxx 99/121
15 oct, 2017
10/25/2017 Test Information

a. Cognitive Behavioural Therapy

b. Eye movement desensitization and reprocessing

c. Supportive Psychotherapy

d. Crisis intervention

Solution.

(a) Cognitive Behavioural Therapy

Ref: Read the text below

Sol:

- Psychotherapy of choice is CBT and New psychotherapy in PTSD is EMDR.

Answer. a

262. Most Common Location of imagined defect in patients with Body Dysmorphic Disorder?

a. Hair

b. Nose

c. Skin

d. Eyes

Solution. (a) Hair

Ref:Read the text below

Sol:

- Prevalence wise Hair > Nose > Skin

Answer. a

263. A 23-year old man impulsively steals a pack of gum at a convenience store. He has never stolen anything previously, and almost immediately upon
exiting the store with the gum, he begins to feel extremely guilty.Which of the following concepts introduced by Freud is most likely responsible for this
mans emotional response to his theft?

a. Id

b. Ego

c. Superego

d. Preconscious function

Solution. (c) Superego

Ref:Read the text below

Sol:

In his structural theory of the mind, Freud divided the psychic apparatus into three agencies: the id,which contains the instinctual drives; the ego,
whose function is to find an equilibrium between gratification of the instinctual drives and the rules of society (and the demands of the superego); and the
superego, the agency that contains the internalized parental and societal rules and dictates to the ego what is not to be done.

Guilt is the consequence of transgressing the

superegos prohibitions.

Preconscious and conscious functions are dimensions of the ego. Logical and abstract thinking as well as verbal expression come from these functions.

Answer. c

264. A young man is often the object of his friends jokes because he drops to the floor whenever he is having a good laugh. Which of the following
isthis man most likely suffering from?

a. Cataplexy
http://cbt.damsdelhi.com/damscbtadmin/admin/index.php?pageName=test/export&id=1710001
xxxxxxxxxxxxxxxxxxxxxxxxxxxxxxxxxxxxxxxxxxxxxxxxxxxxxxxxxxxxxxxxxxxxxxxxxxxxxxxxxxxxxx 100/121
15 oct, 2017
10/25/2017 Test Information

b. Narcolepsy

c. Drop seizures

d. Histrionic personality

Solution. (b) Narcolepsy

Ref:Read the text below

Sol:

Cataplexy refersto a sudden loss of muscle tone (ranging in severity from weakness in the knee to a total loss of tone) triggered by strong emotions,
which takes place during full wakefulness.

Cataplexy is thought to be because of an abnormal intrusion of REM sleep phenomena in periods of wakefulness.

It is usually treated with medications that reduce REM sleep, such as antidepressants.

Cataplexy maybe a symptom of narcolepsy, another dyssomnia characterized by the irresistible urge to fall asleep regardless of the situation.

Answer. b

265. A 46-year-old man is being monitored in a sleep study laboratory.After he has been asleep for 90 minutes, his EEG shows low-voltage,
random fast activity with sawtooth waves. When awakened during this period,the patient reports that he was dreaming. Which of the following
sleep stages was this patient in when awakened?

a. Alpha waves

b. Theta waves

c. Delta waves

d. Rapid eye movement (REM)

Solution. (d) Rapid eye movement (REM)

Ref:Read the text below

Sol:

Dreaming is the main characteristic of REM sleep.

The EEG shows characteristic low-voltage waves that are random, fast, and saw toothed.S

Active eye movements are attributed to the individuals watching his or her dreams.

A lack of muscle tone during REM sleep prevents the individual from acting out his or her dreams.

REM sleep is also characterized by increased heart rate and blood pressure and penile or clitoral nocturnal erections.

Answer. d

266. 26 year old female with breast lump best investigation is

a. MRI

b. USG

c. Mammography

d. CT

Solution. (b) USG

Ref:Read the text below

Sol:

In a young woman with breast lump, USG is recommended.

Mammography will have limitations in sensitivity at her age because of a glandular breast which will be dense.

CT is not recommended for breast imaging.

http://cbt.damsdelhi.com/damscbtadmin/admin/index.php?pageName=test/export&id=1710001
xxxxxxxxxxxxxxxxxxxxxxxxxxxxxxxxxxxxxxxxxxxxxxxxxxxxxxxxxxxxxxxxxxxxxxxxxxxxxxxxxxxxxx 101/121
15 oct, 2017
10/25/2017 Test Information
MRI is recommended for screening for DCIS in high risk females and evaluation of breast implants. MRI also for differentiating operative scar from
recurrence.

Answer. b

267. Most sensitive Xray to detect pneumoperitoneum is :

a. CXR Erect PA view

b. CXR Erect AP view

c. CXR Erect Lateral View

d. Abdomen Erect View

Solution. (c) CXR Erect Lateral View.

Ref:Read the text below

Sol:

CXR erect is the classic xraywhich is considered the best Xray for detecting pneumoperitoneum and is considered better than abdominal erect.

However, in this question, all three choices mention CXR erect in different positions. AP & PA view have almost same sensitivity while a CXR erect
lateral projection has maximum sensitivity.

It is not often clinically done, but this is the answer. Also note CT Abdomen is most sensitive radiological investigation for pneumoperitoneum.

Answer. c

268. MR enterography oral contrast material used is

a. Mannitol

b. Water

c. Barium

d. Iodinated contrast

Solution. (a) Mannitol.

Ref:Read the text below

Sol:

MR enterography is a new non-invasive technique for diagnosis of small bowel disorders, like crohns disease.

Patients drink about 1-1.5 L of a 2.5% mannitol solution at regular intervals over a period of approximately 40 minutes prior to the study.

This solution acts as a hyperosmolar agent which draw fluid into the bowel and (biphasic) appears as low signal intensity on T1-weighted images
and high signal intensity on T2-weighted images

Answer. a

269. Which of them is correctly described as per the image shown

a. 4 is LA, 5 is RA, 1 is RVOT, 2 Aortic root, 3 SVC

http://cbt.damsdelhi.com/damscbtadmin/admin/index.php?pageName=test/export&id=1710001
xxxxxxxxxxxxxxxxxxxxxxxxxxxxxxxxxxxxxxxxxxxxxxxxxxxxxxxxxxxxxxxxxxxxxxxxxxxxxxxxxxxxxx 102/121
15 oct, 2017
10/25/2017 Test Information
b. 5 is LA, 4 is RA, 2 is RVOT, 1 Aortic root, 3 SVC

c. 2 is LA, 3 is RA, 1 is RVOT, 5 Aortic root,8 SVC

d. 1 is LA,3 is RA, 1 is RVOT, 8 Aortic root,2 SVC

Solution. (b) 5 is LA, 4 is RA, 2 is RVOT, 1 Aortic root, 3 SVC.

Ref:Read the text below

Sol:

Key to image

1 = root of aorta
2 = pulmonary outflow
3 = superior vena cava
4 = right atrium
5 = left atrium
6 = left pulmonary vein
7 = right pulmonary vein
8 = descending aorta

Answer. b

270. Identify the disease on this CXR

a. Coarctation of Aorta

b. Rheumatic Heart disease

c. TGA

d. Ebsteins anomaly

Solution. (b) Rheumatic Heart disease

Ref:Read the text below

Sol:

Note the carinal widening, and bulge of the left atrial appendage. Also notice the double atrial density evident on the right side. These are findings
consistent with LA enlargement seen in RHD due to mitral valvular disease.

See the findings in this annotated image: In yellow the boundaries of the enlarged LA are shown.

http://cbt.damsdelhi.com/damscbtadmin/admin/index.php?pageName=test/export&id=1710001
xxxxxxxxxxxxxxxxxxxxxxxxxxxxxxxxxxxxxxxxxxxxxxxxxxxxxxxxxxxxxxxxxxxxxxxxxxxxxxxxxxxxxx 103/121
15 oct, 2017
10/25/2017 Test Information

Ebsteins anomaly would show box shaped heart, with pulmonary oligaemia & predominately right atrial enlargement.

TGA (uncorrected) shows an egg on side/string appearance with narrow vascular pedicle.

Answer. b

271. Most sensitive investigation in abdominal trauma

a. CT

b. MRI

c. USG

d. PET

Solution. (a) CT.

Ref:Read the text below

Sol:

CECT is investigation of choice for blunt abdominal trauma.

FAST(focussed assessment with sonography in trauma) is the 1st step n evaluating a patient with blunt abdominal trauma.

Answer. a

272. Identify the investigation & structure marked in the image :

http://cbt.damsdelhi.com/damscbtadmin/admin/index.php?pageName=test/export&id=1710001
xxxxxxxxxxxxxxxxxxxxxxxxxxxxxxxxxxxxxxxxxxxxxxxxxxxxxxxxxxxxxxxxxxxxxxxxxxxxxxxxxxxxxx 104/121
15 oct, 2017
10/25/2017 Test Information

a. MRI showing lymphnodal mass

b. CT showing aortic dissection

c. CT showing pulmonary embolism

d. CT showing Thymus

Solution. (d) CT showing Thymus.

Ref:Read the text below

Sol:

- This is a CT scan of the thorax, note the white appearance of bones and note the triangular structure in anterior mediastinum,just anterior to great
vessels: thymus.

Answer. d

273. Which of the following is not correctly matched:

a. Osteoclastoma or giant cell tumour

b. Hyperparathyroidism

c. Aneurysmal Bone Cyst

http://cbt.damsdelhi.com/damscbtadmin/admin/index.php?pageName=test/export&id=1710001
xxxxxxxxxxxxxxxxxxxxxxxxxxxxxxxxxxxxxxxxxxxxxxxxxxxxxxxxxxxxxxxxxxxxxxxxxxxxxxxxxxxxxx 105/121
15 oct, 2017
10/25/2017 Test Information

d. Vertebral hemangioma

Solution. (c) Aneurysmal Bone Cyst.

Ref:Read the text below

Sol:

Aneurysmal bone cyst (not correctly matched)

Discussions

Choice C is not aneurysmal bone cyst. There is a lytic lesion in epiphysis of an immature skeleton, which is likely to be chondrobalstoma also called
as codmans tumour. Histological characterized by chicken wire calcification.

Choice Dis classic CT scan showing polka dot sign of the vertebral hemangioma

Choice Ais GCT of the lower end of radius in mature skeleton with multilocular/expansile appearance and subarticular location. ABC on the other
hand is seen in immature skeleton in subphyseal location

Choice B is subperiosteal resorption of the 2/3 digit on radial side, typical of hyperparathryoidism

Answer. c

274. The sentinel clot sign:

a. Helps to localize the source of a hemopertioneum.

b. Is due to the lower density of the clotted component of a hemoperitoneum.

c. Is rarely seen.

d. All of the above.

Solution. (a) Helps to localize the source of a hemopertioneum.

http://cbt.damsdelhi.com/damscbtadmin/admin/index.php?pageName=test/export&id=1710001
xxxxxxxxxxxxxxxxxxxxxxxxxxxxxxxxxxxxxxxxxxxxxxxxxxxxxxxxxxxxxxxxxxxxxxxxxxxxxxxxxxxxxx 106/121
15 oct, 2017
10/25/2017 Test Information
Ref:Read the text below

Sol:

The area of hemoperitoneum adjacent to a source of bleeding is most likely to clot first and may thus help localize the source of bleeding.

The clotted component is more dense than the unclotted blood, not less dense.

The sentinel clot sign is often seen.

Answer. a

275. An asymmetric breast density is seen on MLO view,but not on CC view. so next appropriate approach would be

a. Exaggerated CC view

b. Cleavage view

c. 90-degree mediolateral view

d. Rolled cc view

Solution. (c) 90-degree mediolateral view.

Ref:Read the text below

Sol:

The real lesion is unlikely to change its appearance, regardless of the angle of projection. A straight lateral view for an asymmetry seen only on a
mediolateral oblique (MLO) view and a rolled view for an asymmetry seen only on a craniocaudal (CC) view should be obtained.

If the asymmetryis maintained even after the angle of projection is changed, additional views in other projections should be obtained.

Answer. c

276. Sepsis increases metabolic needs by approximately what percentage above the normal value?

a. 25%

b. 50%

c. 75%

d. 100%

Solution. (b) 50%

Ref:Read the text below

Sol:

Sepsis increases the metabolic needs to approximately 150-160% of resting energy expenditure, or 50% above normal.

http://cbt.damsdelhi.com/damscbtadmin/admin/index.php?pageName=test/export&id=1710001
xxxxxxxxxxxxxxxxxxxxxxxxxxxxxxxxxxxxxxxxxxxxxxxxxxxxxxxxxxxxxxxxxxxxxxxxxxxxxxxxxxxxxx 107/121
15 oct, 2017
10/25/2017 Test Information

Answer. b

277. Which nutrient is proportionately increased in pulmonary failure enteral formula?

a. Carbohydrate

b. Protein

c. Fat

d. Vitamins

Solution. (c) Fat

Ref:Read the text below

Sol:

In pulmonary failure formulas, fat content is usually increased to 50% of the total calories, with a corresponding reduction in carbohydrate
content.

The goal is to reduce carbon dioxide production and alleviate ventilation burden for failing lungs.

Answer. c

278. A 70 kg man with a laceration to the femoral artery loses approximately 850 cc of blood. He presents to the emergency with a PR of 101/min.
Which class of hemorrhage would this represent?

a. Class 1

b. Class 2

c. Class 3

d. Class 4

Solution. (b) Class 2

Ref:Read the text below

Sol:

http://cbt.damsdelhi.com/damscbtadmin/admin/index.php?pageName=test/export&id=1710001
xxxxxxxxxxxxxxxxxxxxxxxxxxxxxxxxxxxxxxxxxxxxxxxxxxxxxxxxxxxxxxxxxxxxxxxxxxxxxxxxxxxxxx 108/121
15 oct, 2017
10/25/2017 Test Information

Answer. b

279. Surgical intervention in a patient with a lung abscess should be considered:

a. If the abscess is more than 3 cm in size

b. If there is slight decrease in the size after 2 weeks of antibiotic therapy

c. If the abscess is under tension

d. If there are bilateral lung abscesses

Solution. (c) If the abscess is under tension

Ref:Read the text below

Sol:

Answer. c

280. In a patient with symptomatic varicose veins where the GSV has reflux and a diameter of 9mm, the preferred surgical treatment is:

http://cbt.damsdelhi.com/damscbtadmin/admin/index.php?pageName=test/export&id=1710001
xxxxxxxxxxxxxxxxxxxxxxxxxxxxxxxxxxxxxxxxxxxxxxxxxxxxxxxxxxxxxxxxxxxxxxxxxxxxxxxxxxxxxx 109/121
15 oct, 2017
10/25/2017 Test Information
a. Sclerotherapy

b. RFA or laser ablation

c. Vein valvuloplasty

d. Flush ligation of SFJ with stripping of vein

Solution. (b) RFA or laser ablation

Ref:Read the text below

Sol:

RFA (radiofrequency ablation)is the best method to manage symptomatic varicose vein patients.

EVLT (endovenous laser therapy) is equally effective as RFA in the management of these patients

Sclerotherapyis used for patients with veins less than 3 mm in diameter and this technique is associated with a lot of complications

Open surgery (Trendenlenburg procedure) flush ligation of SFJ was done earlier but since the developmentof RFA/EVLT, this is not used.

Answer. b

281. Which of the following is not one of the five principles of surgical correction of GERD?

a. The fundoplication should be maintained in the abdomen by a crural repair

b. The operation should restore the pressure of the LES to 10 times the gastric pressure

c. An adequate amount of intra-abdominal esophagus should be obtained (approx. 2 cm)

d. The fundoplication should not increase the resistance above what the peristalsis of the esophagus can overcome

Solution. (b) The operation should restore the pressure of the LES to 10 times the gastric pressure

Ref:Read the text below

Sol:

http://cbt.damsdelhi.com/damscbtadmin/admin/index.php?pageName=test/export&id=1710001
xxxxxxxxxxxxxxxxxxxxxxxxxxxxxxxxxxxxxxxxxxxxxxxxxxxxxxxxxxxxxxxxxxxxxxxxxxxxxxxxxxxxxx 110/121
15 oct, 2017
10/25/2017 Test Information

Answer. b

282. In an attempt to teach medical students about a barium swallow, a volunteer undergoes this test and is incidentally discovered with the finding
shown in the image. How should this patient be managed?

a. Observation

b. Dilation

c. Incision of the structure

d. Resection with end to end anastomosis

Solution. (a) Observation

Ref:Read the text below

Sol:

This image shows a schatzkis ring, which is a thin submucosal ring in the lower esophagus close to the squamo-columnar junction.

Asymptomatic patientsdont require any treatment whereas symptomatic patients should be managed with dilation.

Incision should never be attempted

Answer. a

283.

Which of the following catheter materials is most suitable for long-term use?

http://cbt.damsdelhi.com/damscbtadmin/admin/index.php?pageName=test/export&id=1710001
xxxxxxxxxxxxxxxxxxxxxxxxxxxxxxxxxxxxxxxxxxxxxxxxxxxxxxxxxxxxxxxxxxxxxxxxxxxxxxxxxxxxxx 111/121
15 oct, 2017
10/25/2017 Test Information

a. Latex

b. Silicone

c. Rubber

d. Polyurethane

Solution. (b) Silicone

Ref:Read the text below

Sol:

Answer. b

284. What is the probable diagnosis of the patient with spinal cast presenting with bilious vomiting?

a. Acute dilation of stomach

b. Duodenal obstruction

c. Peritonitis

d. Acute pancreatitis

Solution. (b) Duodenal obstruction

Ref:Read the text below

Sol:

This is a case of duodenal obstruction secondary to superior mesenteric artery syndrome.

SMA syndrome or Wilkie syndrome

SMA arises from anterior aspect of aortaand crosses in front of the third portion of the duodenum to enter the root of mesentery.

Disease particularly affect young asthenic individual being more common in female.

Symptoms include profound nausea,vomiting, abdominal distension weight loss and post parandial abdominal pain.

Diagnosed by hypotonic duodenography

Treatment Conservative

- Duodenojejunostomy in severe cases

Answer. b

285. Which of the following preservative is used while packing catgut suture?

a. Isopropyl alcohol

b. Colloidal iodine

c. Glutaraldehyde

d. Hydrogen peroxide

Solution. (a) Isopropyl alcohol

Ref:Read the text below

Sol:

http://cbt.damsdelhi.com/damscbtadmin/admin/index.php?pageName=test/export&id=1710001
xxxxxxxxxxxxxxxxxxxxxxxxxxxxxxxxxxxxxxxxxxxxxxxxxxxxxxxxxxxxxxxxxxxxxxxxxxxxxxxxxxxxxx 112/121
15 oct, 2017
10/25/2017 Test Information
Catgut suture

- Biological, absorbable monofilament suture

- Derived from submucosa of the sheep intestine.

- It is the oldest suture lasts for 8-10 days

- Plain catgut suture lasts for 8-10 days.

- Catgut suture causes inflammatory cellular reaction with release of protease. It also carries the risk of prion transmission.

- On soaking in chromic acid (chromic catgut) promote early dissolution in body fluids.

Answer. a

286. The most pre-malignant lesion of the stomach is

a. Adenoma

b. Chronic gastric ulcer

c. Atrophic gastritis

d. Verrucous gastritis

Solution. (c) Atrophic gastritis

Ref:Read the text below

Sol:

Answer. c

287. This instrument is used in all of the following procedure except:

http://cbt.damsdelhi.com/damscbtadmin/admin/index.php?pageName=test/export&id=1710001
xxxxxxxxxxxxxxxxxxxxxxxxxxxxxxxxxxxxxxxxxxxxxxxxxxxxxxxxxxxxxxxxxxxxxxxxxxxxxxxxxxxxxx 113/121
15 oct, 2017
10/25/2017 Test Information

a. Hemorrhoid surgery

b. APR following rectal cancer surgery

c. LAR following rectal cancer surgery

d. Esophago-jejunostomy following total gastrectomy

Solution. (b) APR following rectal cancer surgery

Ref:Read the text below

Sol:

The image shows a circular stapler, which is used for Hemorrhoid surgery, LAR following rectal cancer surgery & Esophago-jejunostomy
following total gastrectomy

Answer. b

288. Which one of the following statements regarding breast cancer during pregnancy is not true?

a. It is one of the most common cancer during pregnancy

b. Breast conservation surgery can be done if the patient is diagnosed in the third trimester

c. Hormonal therapy can be given in the second trimester

d. The safest trimester to give chemotherapy is the second trimester

Solution.

(c) Hormonal therapy can be given in the second trimester

Ref: Read the text below

Sol:

Breast cancer is one of the commonest cancers during pregnancy.

http://cbt.damsdelhi.com/damscbtadmin/admin/index.php?pageName=test/export&id=1710001
xxxxxxxxxxxxxxxxxxxxxxxxxxxxxxxxxxxxxxxxxxxxxxxxxxxxxxxxxxxxxxxxxxxxxxxxxxxxxxxxxxxxxx 114/121
15 oct, 2017
10/25/2017 Test Information
Usually BCS is avoided during pregnancy because radiotherapy cannot be given, but if the patient is diagnosed in the third trimester, BCS can be
done and RT can be given after delivery.

Hormonal therapy and RT are contraindicated in all trimesters. 2nd trimester is the safest trimester to give chemotherapy in breast cancer
Answer. c

289.

Which flap has been marked in the image shown below?

a. S. anterior flap

b. L. dorsii flap

c. S. posterior inferior flap

d. Trapezius flap

Solution. (b) L. dorsii flap

Ref:Read the text below

Sol:

This image clearly highlights the marking of an L. dorsii flap, which is an axial flap based on the thoracodorsal pedicle.

This flap is used for the reconstruction of the chest wall and the breast.

Other axial flaps

PMMC flap - based on the pectoral branch of thoracoacromial artery.

TRAM flap - based on the superior or inferior epigastric artery.

Abbey estlander flap - based on the labial artery.

Forehead flap - based on the superficial temporal artery

Answer. b

290. Identify the knot shown in the image

a. Surgeons knot

http://cbt.damsdelhi.com/damscbtadmin/admin/index.php?pageName=test/export&id=1710001
xxxxxxxxxxxxxxxxxxxxxxxxxxxxxxxxxxxxxxxxxxxxxxxxxxxxxxxxxxxxxxxxxxxxxxxxxxxxxxxxxxxxxx 115/121
15 oct, 2017
10/25/2017 Test Information
b. Grannys knot

c. Reef knot

d. Aberdeens knot

Solution. (a) Surgeons knot

Ref:Read the text below

Sol:

Reef knot is also a secure knot

Answer. a

291. Most frequently used nerve in perioperative monitoring:

a. Ulnar

b. Radial

c. Facial

d. Median

Solution. (a) Ulnar

Ref:Read the text below

Sol:

In clinical practice ulnar nerve is the most common site for nerve stimulation monitoring.

Median; posterior tibial; common peroneal and facial nerve can also be used:

Role of nerve stimulating moinitoring:

- To judge the depth of anaesthesia

- To determine effect of muscle relaxant

- To determine the exact site of injury at the time of surgery (as in brachial plexus repair)

Answer. a

292.

In the immediate postopera ve period the common cause of respiratory insuciency could be because of the following, except:

a. Residual effect of muscle relaxant

b. Overdose of narcotic analgesic

c. Mild hypovolemia

d. Myocardial infarction

Solution. (c) Mild hypovolemia

Ref:Read the text below

Sol:

http://cbt.damsdelhi.com/damscbtadmin/admin/index.php?pageName=test/export&id=1710001
xxxxxxxxxxxxxxxxxxxxxxxxxxxxxxxxxxxxxxxxxxxxxxxxxxxxxxxxxxxxxxxxxxxxxxxxxxxxxxxxxxxxxx 116/121
15 oct, 2017
10/25/2017 Test Information
Mild hypovolemia is well compensatedby body and is usually asymptomatic

Infact respiratory insufficient isseen only in serve hypovolemic shock, when there would be acidosis secondary to impaired tissue perfusion.

Answer. c

293. Which one of the following is an indication for IVC filter in a patient with DVT?

a. 45 year old male with first episode of DVT

b. 80 year old male with first episode of DVT

c. 55 year old male with recent history of intra-cranial hge with DVT

d. 50 year old female with DVT in the post-partum period

Solution. (c) 55 year old male with recent history of intra-cranial hge with DVT

Ref:Read the text below

Sol:

Indications for IVC filter in patients with DVT

Recurrent DVT despite anticoagulation

Contraindication for anticoagulation in a patient with proximal DVT

Recurrent pulmonary embolisms despite anticoagulation

The commonly used filter is a Greenfield filter.

Answer. c

294. Which one of the following changes have been incorporated in the 8th AJCC classification for breast cancer?

a. Dimpling is now considered as skin involvement in breast cancer

b. Pagets disease is no longer classified in the AJCC staging

c. LCIS has been removed as an in-situ lesion from the latest staging and is now considered a benign condition

d. Involvement of p. major is now considered a chest wall involvement.

Solution.

(c) LCIS has been removed as an in-situ lesion from the latest staging and is now considered a benign condition

http://cbt.damsdelhi.com/damscbtadmin/admin/index.php?pageName=test/export&id=1710001
xxxxxxxxxxxxxxxxxxxxxxxxxxxxxxxxxxxxxxxxxxxxxxxxxxxxxxxxxxxxxxxxxxxxxxxxxxxxxxxxxxxxxx 117/121
15 oct, 2017
10/25/2017 Test Information
Ref: Read the text below

Sol:

According to the 8th AJCC Breast cancer classification, LCIS is no longer considered as an in-situ lesion. It has been labeled as a benign disease
with an increased risk of breast cancer.

Involvement of the pectoralis major muscle is not considered as chest wall involvement

Skin involvement is only when peaudorange, ulceration or satellite nodules are present

Dimpling and retrac on are not considered as skin involvement

Answer. c

295. Fine needle aspiration cytology is not suitable for diagnosing:

a. Tubercular lymphadenitis

b. Papillary carcinoma thyroid

c. Plasmacytoma

d. Aneurysmal bone cyst

Solution. (d) Aneurysmal bone cyst

Ref:Read the text below

Sol:

Aneurysmal bone cyst cannot be diagnosed by FNAC.

FNAC is the first investigationdone in thyroid, parotid, breast and lymph node enlargement.

But it cant differentiate follicular adenoma from follicular carcinoma of thyroid.

Answer. d

296. The most radiosensitive tumor among the following is :

a. Bronchogenic carcinoma

b. Carcinoma parotid

c. Dysgerminoma

d. Osteogenic sarcoma

Solution. (c) Dysgerminoma

Ref:Read the text below

Sol:

- Seminoma or its female counter part dysgerminoma is most radiosensitive tumor.

Answer. c

297. Which one of the following statements is not true regarding Fowlers position?

http://cbt.damsdelhi.com/damscbtadmin/admin/index.php?pageName=test/export&id=1710001
xxxxxxxxxxxxxxxxxxxxxxxxxxxxxxxxxxxxxxxxxxxxxxxxxxxxxxxxxxxxxxxxxxxxxxxxxxxxxxxxxxxxxx 118/121
15 oct, 2017
10/25/2017 Test Information

a. It reduces the risk of air embolism

b. It reduces the bleeding during CNS procedures

c. It is ideally suited for CNS procedures involving the posterior fossa

d. This position can also be used for breast reconstruction procedures

Solution. (a) It reduces the risk of air embolism

Ref:Read the text below

Sol:

Answer. a

298. The root cause of majority of wrong site surgeries result from:

a. Communication errors

b. Emergency surgery

c. Multiple procedures in the same sitting

d. Multiple surgeons

Solution.

(a) Communication errors

Ref: Read the text below

Sol:

All the reasons listed can lead to wrong site surgery but the most important cause is communication errors and to prevent this a WHO surgical safety
checklist is used.
http://cbt.damsdelhi.com/damscbtadmin/admin/index.php?pageName=test/export&id=1710001
xxxxxxxxxxxxxxxxxxxxxxxxxxxxxxxxxxxxxxxxxxxxxxxxxxxxxxxxxxxxxxxxxxxxxxxxxxxxxxxxxxxxxx 119/121
15 oct, 2017
10/25/2017 Test Information
The image below highlights the components of the WHO surgical safety checklist

Answer. a

299. Which operation is not advised for morbid obesity

a. Partial gastrectomy

b. Illeal transposition

c. Gastic bypass

d. Illeopancreatic diversion

Solution. (b) Illeal transposition

Ref:Read the text below

Sol:

SURGICAL THERAPY

According to NIH guidelines for bariatric surgery, Patients must have a BMI greater than 40 kg/m2 without associated comorbid medical
problem. They must have also failed dietary therapy.

VERTICAL BANDED GASTROPLASTY

Not used now because of poor long term weight loss, a high rate of late stenosis of the gastric outlet, and a tendency for patients to adopt a high-
calorie liquid diet, thereby leading to regain of weight.

ADJUSTABLE GASTRIC BANDING

In this procedure bands are applied on the proximal part of stomach and work on the principle of restriction of oral intake by limiting the volume of
the proximal part of the stomach. Their advantage over the traditional vertical banded gastroplasty is adjustability.

The pars flaccid technique has become the approach of choice for placing the adjustable band

Complications include band slippage (common) and erosion of band (rare but serious).

ROUX-EN-Y GASTRIC BYPASS

Most commonly performed bariatric operation in the United States

BILIOPANCREATIC DIVERSION

The intestinal tract is reconstructed to allow only a short so-called common channel of the distal 50-cm of terminal ileum for absorption of fat and
protein.

http://cbt.damsdelhi.com/damscbtadmin/admin/index.php?pageName=test/export&id=1710001
xxxxxxxxxxxxxxxxxxxxxxxxxxxxxxxxxxxxxxxxxxxxxxxxxxxxxxxxxxxxxxxxxxxxxxxxxxxxxxxxxxxxxx 120/121
15 oct, 2017
10/25/2017 Test Information
DUODENAL SWITCH/DS

Answer. b

300. In order to expose the celiac axis, left renal vein, superior mesenteric artery and abdominal aorta in a case of trauma.Choose the most appropriate
procedure?

a. Left medial visceral rotation

b. Right medial visceral

c. Cranial visceral rotation

d. Caudal visceral rotation

Solution. (a) Left medial visceral rotation

Ref:Read the text below

Sol:

The suprarenal aorta, celiac axis, proximal superior mesenteric , and left renal arteries can all be exposed by left medial visceral rotation.

Answer. a

Test Answer

1. (c) 2. (b) 3. (b) 4. (c) 5. (a) 6. (d) 7. (a) 8. (d) 9. (c) 10. (d)
11. (c) 12. (d) 13. (c) 14. (d) 15. (a) 16. (d) 17. (c) 18. (a) 19. (c) 20. (a)
21. (c) 22. (b) 23. (c) 24. (c) 25. (b) 26. (a) 27. (c) 28. (d) 29. (c) 30. (a)
31. (a) 32. (d) 33. (b) 34. (a) 35. (c) 36. (a) 37. (a) 38. (c) 39. (a) 40. (c)
41. (b) 42. (a) 43. (c) 44. (d) 45. (a) 46. (a) 47. (d) 48. (d) 49. (d) 50. (b)
51. (a) 52. (b) 53. (d) 54. (b) 55. (d) 56. (d) 57. (a) 58. (a) 59. (d) 60. (a)
61. (b) 62. (a) 63. (a) 64. (c) 65. (a) 66. (b) 67. (a) 68. (a) 69. (c) 70. (d)
71. (a) 72. (c) 73. (d) 74. (b) 75. (a) 76. (d) 77. (b) 78. (b) 79. (d) 80. (c)
81. (a) 82. (b) 83. (a) 84. (a) 85. (d) 86. (b) 87. (d) 88. (b) 89. (b) 90. (d)
91. (c) 92. (c) 93. (c) 94. (d) 95. (d) 96. (d) 97. (d) 98. (c) 99. (a) 100. (b)
101. (c) 102. (c) 103. (c) 104. (c) 105. (a) 106. (c) 107. (d) 108. (c) 109. (c) 110. (d)
111. (c) 112. (c) 113. (d) 114. (c) 115. (a) 116. (b) 117. (d) 118. (b) 119. (d) 120. (d)
121. (d) 122. (d) 123. (a) 124. (c) 125. (c) 126. (a) 127. (b) 128. (c) 129. (d) 130. (c)
131. (c) 132. (b) 133. (a) 134. (b) 135. (c) 136. (d) 137. (a) 138. (b) 139. (d) 140. (c)
141. (c) 142. (b) 143. (c) 144. (d) 145. (c) 146. (d) 147. (a) 148. (a) 149. (b) 150. (a)
151. (a) 152. (b) 153. (d) 154. (d) 155. (a) 156. (a) 157. (a) 158. (d) 159. (a) 160. (b)
161. (d) 162. (b) 163. (a) 164. (c) 165. (c) 166. (d) 167. (a) 168. (a) 169. (c) 170. (a)
171. (d) 172. (a) 173. (b) 174. (a) 175. (a) 176. (c) 177. (a) 178. (c) 179. (b) 180. (c)
181. (c) 182. (b) 183. (b) 184. (d) 185. (b) 186. (a) 187. (a) 188. (a) 189. (c) 190. (c)
191. (c) 192. (c) 193. (b) 194. (d) 195. (a) 196. (d) 197. (a) 198. (c) 199. (a) 200. (b)
201. (a) 202. (a) 203. (b) 204. (b) 205. (b) 206. (a) 207. (c) 208. (a) 209. (d) 210. (a)
211. (a) 212. (a) 213. (b) 214. (c) 215. (a) 216. (a) 217. (b) 218. (c) 219. (b) 220. (a)
221. (c) 222. (d) 223. (c) 224. (d) 225. (d) 226. (d) 227. (b) 228. (a) 229. (c) 230. (b)
231. (b) 232. (d) 233. (d) 234. (d) 235. (d) 236. (c) 237. (b) 238. (b) 239. (d) 240. (c)
241. (a) 242. (b) 243. (a) 244. (a) 245. (b) 246. (a) 247. (c) 248. (d) 249. (a) 250. (a)
251. (d) 252. (c) 253. (b) 254. (d) 255. (a) 256. (d) 257. (d) 258. (c) 259. (d) 260. (d)
261. (a) 262. (a) 263. (c) 264. (b) 265. (d) 266. (b) 267. (c) 268. (a) 269. (b) 270. (b)
271. (a) 272. (d) 273. (c) 274. (a) 275. (c) 276. (b) 277. (c) 278. (b) 279. (c) 280. (b)
281. (b) 282. (a) 283. (b) 284. (b) 285. (a) 286. (c) 287. (b) 288. (c) 289. (b) 290. (a)
291. (a) 292. (c) 293. (c) 294. (c) 295. (d) 296. (c) 297. (a) 298. (a) 299. (b) 300. (a)

http://cbt.damsdelhi.com/damscbtadmin/admin/index.php?pageName=test/export&id=1710001
xxxxxxxxxxxxxxxxxxxxxxxxxxxxxxxxxxxxxxxxxxxxxxxxxxxxxxxxxxxxxxxxxxxxxxxxxxxxxxxxxxxxxx 121/121

You might also like